Você está na página 1de 158

APRENDA MATEMÁTICA DE FORMA FÁCIL

PARA O ENEM
Curso aplicado de matemática básica para o Ensino
Médio

ATENÇÃO: No final deste ebook temos uma surpresa


para você!
editora
PESAFRA
UMÁRIO
• Conjuntos
• Lógica Matemática I
• Lógica Matemática II
• Lógica Matemática III
• Relações Binárias
• Funções I
• Funções II
• Funções III
• Funções IV
• Certa função
• Certa classe de funções
• Calcule o valor desta função
• Módulo I
• Módulo II
• Módulo III
• Produtos Notáveis
• Exercícios de Aritmética I
• Exercícios de Aritmética II
• Números congruentes
• Proporcionalidade entre grandezas
• Regra de três composta
• Cuidado com a regra de três
• MDC e MMC
• Sistema de numeração binário
• Sistema de numeração romano
• Uma contagem na sala de aula
• Vôlei, xadrez ou tênis?
• Potências e radicais - parte I
• Potências e radicais - parte II
• Três problemas na média
• Uma questão antiga em Catanduva
• Um trem de três vagões
• Sobre o conjunto vazio
• Subconjuntos de um conjunto finito
• 1 + 11 + 111 + 1111 + 11111 + ... + 11...11111
• De um jeito mais fácil
• Fatorial de 18
• Calcule o valor da função
• Análise do preço de uma torta de morango
• Coletânea de exercícios de funções
• Argumentos lógicos
• Um Problema interessante
• Domínio e conjunto imagem de uma função real de variável real
• Num Centro Esportivo
• Dois problemas, duas soluções e duas propostas imperdíveis
• Desen ROCK - se
• Contando elementos I
• Contando elementos II
• No reino da bicharada

1 A teoria avançada dos conjuntos foi desenvolvida por volta do ano 1872
pelo matemático alemão Georg Cantor (1845 / 1918) e aperfeiçoada no início
do século XX por outros matemáticos, entre eles, Ernst Zermelo (alemão -
1871/1956), Adolf Fraenkel (alemão - 1891/ 1965), Kurt Gödel (austríaco -
1906 /1978), Janos von Newman (húngaro - 1903 /1957), entre outros.
O que se estuda deste assunto ao nível do segundo grau e exigido em alguns
vestibulares, é tão somente uma introdução elementar à teoria dos conjuntos,
base para o desenvolvimento de temas futuros, a exemplo de relações,
funções, análise combinatória, probabilidades, etc.

2 - Conjunto: conceito primitivo; não necessita, portanto, de definição.


Exemplo: conjunto dos números pares positivos: P = {2, 4, 6 ,8 ,10 ,12, ... }.

Esta forma de representar um conjunto, pela enumeração dos seus


elementos, chama-se forma de listagem. O mesmo conjunto também poderia
ser representado por uma propriedade dos seus elementos ou seja, sendo x
um elemento qualquer do conjunto P acima, poderíamos escrever:
P = { x | x é par e positivo } = { 2,4,6, ... }.
2.1 - Relação de pertinência:

Sendo x um elemento do conjunto A , escrevemos x A,


onde o símbolo significa "pertence a".
Sendo y um elemento que não pertence ao conjunto A , indicamos esse fato
com a notação
y A.

O conjunto que não possui elementos , é denominado conjunto vazio e


representado por .
Com o mesmo raciocínio, e opostamente ao conjunto vazio, define-se o
conjunto ao qual pertencem todos os elementos, denominado conjunto
universo, representado pelo símbolo U.
Assim é que, pode-se escrever como exemplos:
= { x; x x} e U = {x; x = x}.
2.2 - Subconjunto

Se todo elemento de um conjunto A também pertence a um conjunto B, então


dizemos que
A é subconjunto de B e indicamos isto por A B.

Notas:
a) todo conjunto é subconjunto de si próprio. ( A A)
b) o conjunto vazio é subconjunto de qualquer conjunto. ( A)
m
c) se um conjunto A possui m elementos então ele possui 2 subconjuntos.
d) o conjunto formado por todos os subconjuntos de um conjunto A é
denominado
conjunto das partes de A e é indicado por P(A).
Assim, se A = {c, d} , o conjunto das partes de A é dado por P(A) = { , {c},
{d}, {c,d}}
e) um subconjunto de A é também denominado parte de A.
3 - Conjuntos numéricos fundamentais
Entendemos por conjunto numérico, qualquer conjunto cujos elementos são
números. Existem infinitos conjuntos numéricos, entre os quais, os chamados
conjuntos numéricos fundamentais, a saber:
3.1 - Conjunto dos números naturais

N = {0,1,2,3,4,5,6,... }
3.2 - Conjunto dos números inteiros

Z = {..., -4,-3,-2,-1,0,1,2,3,... }
Nota: é evidente que N Z.
3.3 - Conjunto dos números racionais

Q = {x | x = p/q com p Z,q Zeq 0 }. (o símbolo | lê-se como "tal


que").
Temos então que número racional é aquele que pode ser escrito na forma de
uma fração p/q onde p e q são números inteiros, com o denominador diferente
de zero.
Lembre-se que não existe divisão por zero!.
São exemplos de números racionais: 2/3, -3/7, 0,001=1/1000, 0,75=3/4,
0,333... = 1/3,
7 = 7/1, etc.

Notas:
a) é evidente que N Z Q.
b) toda dízima periódica é um número racional, pois é sempre possível
escrever uma dízima periódica na forma de uma fração.
Exemplo: 0,4444... = 4/9
3.4 - Conjunto dos números irracionais

Q' = {x | x é uma dízima não periódica}. (o símbolo | lê-se como "tal que").
Exemplos de números irracionais:
= 3,1415926... (número pi = razão entre o comprimento de qualquer
circunferência e o seu diâmetro)
2,01001000100001... (dízima não periódica)
3 = 1,732050807... (raiz não exata).

3.5 - Conjunto dos números reais

R = { x | x é racional ou x é irracional }.
Notas:
a) é óbvio que N Z Q R
b) Q' R
c) um número real é racional ou irracional; não existe outra hipótese!

4 - Intervalos numéricos

Dados dois números reais p e q, chama-se intervalo a todo conjunto de todos


números reais compreendidos entre p e q , podendo inclusive incluir p e q. Os
números p e q são os limites do
intervalo, sendo a diferença p - q , chamada amplitude do intervalo. ;
Se o intervalo incluir p e q , o intervalo é fechado e caso contrário, o intervalo
é dito aberto.
A tabela abaixo, define os diversos tipos de intervalos.
TIPOS REPRESENTAÇÃO OBSERVAÇÃO
INTERVALO FECHADO [p;q] = {x R; p inclui os limites p e q
x q}
INTERVALO ABERTO (p;q) = { x R; p exclui os limites p e q
x q}
INTERVALO FECHADO [p;q) = { x R; p inclui p e exclui q
A ESQUERDA x q}
INTERVALO FECHADO (p;q] = {x R; p exclui p e inclui q
À DIREITA x q}
INTERVALO SEMI- [p; ) = {x R; x valores maiores ou iguais a p.
FECHADO p}
INTERVALO SEMI- (- ; q] = { x R; valores menores ou iguais a q.
FECHADO x q}
INTERVALO SEMI- (- ; q) = { x R; valores menores do que q.
ABERTO x q}
INTERVALO SEMI- (p; )={x p} valores maiores do que p.
ABERTO

Nota: é fácil observar que o conjunto dos números reais, (o conjunto R) pode
ser representado na forma de intervalo como R = ( - ;+ ).
5 - Operações com conjuntos
5.1 - União ( )
Dados os conjuntos A e B, define-se o conjunto união A B = {x; x A ou x
B}.
Exemplo: {0,1,3} { 3,4,5 } = { 0,1,3,4,5}. Percebe-se facilmente que o
conjunto união contempla todos os elementos do conjunto A ou do conjunto B.
Propriedades imediatas:
a) A A=A
b) A =A
c) A B=B A (a união de conjuntos é uma operação comutativa)
d) A U = U , onde U é o conjunto universo.
5.2 - Interseção ( )
Dados os conjuntos A e B , define-se o conjunto interseção A B = {x; x
Aex B}.
Exemplo: {0,2,4,5} { 4,6,7} = {4}. Percebe-se facilmente que o conjunto
interseção contempla os elementos que são comuns aos conjuntos A e B.
Propriedades imediatas:
a) A A=A
b) A =
c) A B=B A ( a interseção é uma operação comutativa)
d) A U = A onde U é o conjunto universo.
São importantes também as seguintes propriedades:
P1. A (B C ) = (A B) (A C) (propriedade distributiva)
P2. A (B C ) = (A B) (A C) (propriedade distributiva)
P3. A (A B) = A (lei da absorção)
P4. A (A B) = A (lei da absorção)
Observação: Se A B= , então dizemos que os conjuntos A e B são
Disjuntos.
5.3 - Diferença: A - B = {x ; x Aex B}.
Observe que os elementos da diferença são aqueles que pertencem ao
primeiro conjunto, mas não pertencem ao segundo.
Exemplos:
{ 0,5,7} - {0,7,3} = {5}.
{1,2,3,4,5} - {1,2,3} = {4,5}.
Propriedades imediatas:
a) A - =A
b) -A=
c) A - A =
d) A - B B - A (a diferença de conjuntos não é uma operação comutativa).
5.3.1 - Complementar de um conjunto
Trata-se de um caso particular da diferença entre dois conjuntos. Assim é ,
que dados dois conjuntos A e B, com a condição de que B A , a diferença
A - B chama-se, neste caso, complementar de B em relação a A .
Simbologia: CAB = A - B.
Caso particular: O complementar de B em relação ao conjunto universo U, ou
seja , U - B ,é indicado pelo símbolo B' .Observe que o conjunto B' é formado
por todos os elementos que não pertencem ao conjunto B, ou seja:
B' = {x; x B}. É óbvio, então, que:
a) B B' =
b) B B' = U
c) ' U
d) U' =
6 - Partição de um conjunto
Seja A um conjunto não vazio. Define-se como partição de A, e representa-se
por part(A), qualquer subconjunto do conjunto das partes de A
(representado simbolicamente por P(A)), que satisfaz simultaneamente, às
seguintes condições:
1 - nenhuma dos elementos de part(A) é o conjunto vazio.
2 - a interseção de quaisquer dois elementos de part(A) é o conjunto vazio.
3 - a união de todos os elementos de part(A) é igual ao conjunto A.
Exemplo: Seja A = {2, 3, 5}
Os subconjuntos de A serão: {2}, {3}, {5}, {2,3}, {2,5}, {3,5}, {2,3,5}, e o
conjunto vazio - Ø.
Assim, o conjunto das partes de A será:
P(A) = { {2}, {3}, {5}, {2,3}, {2,5}, {3,5}, {2,3,5}, Ø }
Vamos tomar, por exemplo, o seguinte subconjunto de P(A):
X = { {2}, {3,5} }
Observe que X é uma partição de A - cuja simbologia é part(A) - pois:
a) nenhum dos elementos de X é Ø .
b) {2} { } Ø
c) {2} U { } = {2, 3, 5} = A
Sendo observadas as condições 1, 2 e 3 acima, o conjunto X é uma partição
do conjunto A.
Observe que Y = { {2,5}, {3} } ; W = { {5}, {2}, {3} }; S = { {3,2}, {5} } são outros
exemplos de partições do conjunto A.

Outro exemplo: o conjunto Y = { {0, 2, 4, 6, 8, ...}, {1, 3, 5, 7, ...} } é uma


partição do conjunto Z dos números inteiros, pois {0, 2, 4, 6, 8, ...} {1, 3, 5,
7, ...} = Ø e {0, 2, 4, 6, 8, ...} U {1, 3, 5, 7, ...} = Z

7 - Número de elementos da união de dois conjuntos


Sejam A e B dois conjuntos, tais que o número de elementos de A seja n(A) e
o número de elementos de B seja n(B).
Nota: o número de elementos de um conjunto, é também conhecido com
cardinal do conjunto.

Representando o número de elementos da interseção A B por n(A B) e


o número de elementos da união A B por n(A B) , podemos escrever a
seguinte fórmula:
n(A B) = n(A) + n(B) - n(A B)
8 - Exercícios propostos I:
1) USP-SP - Depois de n dias de férias, um estudante observa que:
a) choveu 7 vezes, de manhã ou à tarde;
b) quando chove de manhã não chove à tarde;
c) houve 5 tardes sem chuva;
d) houve 6 manhãs sem chuva.
Podemos afirmar então que n é igual a:
a)7
b)8
*c)9
d)10
e)11

2) 52 pessoas discutem a preferência por dois produtos A e B, entre outros e


conclui-se que o número de pessoas que gostavam de B era:
I - O quádruplo do número de pessoas que gostavam de A e B;
II - O dobro do número de pessoas que gostavam de A;
III - A metade do número de pessoas que não gostavam de A nem de B.
Nestas condições, o número de pessoas que não gostavam dos dois produtos
é igual a:
*a)48
b)35
c)36
d)47
e)37

3) UFBA - 35 estudantes estrangeiros vieram ao Brasil. 16 visitaram Manaus;


16, S. Paulo e 11, Salvador. Desses estudantes, 5 visitaram Manaus e
Salvador e , desses 5, 3 visitaram também São Paulo. O número de
estudantes que visitaram Manaus ou São Paulo foi:
*a) 29
b) 24
c) 11
d) 8
e) 5

4) FEI/SP - Um teste de literatura, com 5 alternativas em que uma única é


verdadeira, referindo-se à data de nascimento de um famoso escritor,
apresenta as seguintes alternativas:
a)século XIX
b)século XX
c)antes de 1860
d)depois de 1830
e)nenhuma das anteriores
Pode-se garantir que a resposta correta é:
a)a
b)b
*c)c
d)d
e)e

9 - Exercícios propostos II:


1 - Se um conjunto A possui 1024 subconjuntos, então o cardinal de A é igual
a:
a) 5
b) 6
c) 7
d) 9
*e)10
2 - Após um jantar, foram servidas as sobremesas X e Y. Sabe-se que das 10
pessoas presentes, 5 comeram a sobremesa X, 7 comeram a sobremesa Y
e 3 comeram as duas. Quantas não comeram nenhuma?
*a) 1
b) 2
c) 3
d) 4
e) 0
3) PUC-SP - Se A = eB={ }, então:
*a) A B
b) A B=
c) A = B
d) A B=B
e) B A
4) FGV-SP - Sejam A, B e C conjuntos finitos. O número de elementos de A
B é 30, o número de elementos de A C é 20 e o número de elementos
de A B C é 15.
Então o número de elementos de A (B C) é igual a:
*a)35
b)15
c)50
d)45
e)20
5) Sendo a e b números reais quaisquer, os números possíveis de elementos
do conjunto
A = {a, b, {a}, {b}, {a,b} } são:
*a)2 ou 5
b)3 ou 6
c)1 ou 5
d)2 ou 6
e)4 ou 5

1 - INTRODUÇÃO
A Lógica Matemática, em síntese, pode ser considerada como a ciência do
raciocínio e da demonstração. Este importante ramo da Matemática
desenvolveu-se no século XIX, sobretudo através das idéias de George Boole
, matemático inglês (1815 - 1864), criador da Álgebra Booleana, que utiliza
símbolos e operações algébricas para representar proposições e suas inter-
relações.
As idéias de Boole tornaram-se a base da Lógica Simbólica, cuja aplicação
estende-se por alguns ramos da eletricidade, da computação e da eletrônica.
A lógica matemática (ou lógica simbólica), trata do estudo das sentenças
declarativas também conhecidas como proposições , as quais devem
satisfazer aos dois princípios fundamentais seguintes:
Princípio do terceiro excluído: uma proposição só pode ser verdadeira ou falsa
, não havendo outra alternativa.
Princípio da não contradição: uma proposição não pode ser ao mesmo tempo
verdadeira e falsa.
Diz-se então que uma proposição verdadeira possui valor lógico V (verdade) e
uma proposição falsa possui valor lógico F (falso). Os valores lógicos também
costumam ser representados por 0 (zero) para proposições falsas ( 0 ou F) e
1 (um) para proposições verdadeiras ( 1 ou V ).
As proposições são indicadas pelas letras latinas minúsculas: p, q, r, s, t, u, ...
De acordo com as considerações acima, expressões do tipo, "O dia está
bonito" , "3 + 5" , "x é um número real" , "x + 2 = 7", etc., não são proposições
lógicas, uma vez que não poderemos associar a ela um valor lógico definido
(verdadeiro ou falso).
Exemplificamos a seguir algumas proposições, onde escreveremos ao lado de
cada uma delas, o seu valor lógico V ou F. Poderia ser também 1 ou 0.
p: " a soma dos ângulos internos de um triângulo é igual a 180º " ( V )
q: " 3 + 5 = 2 " ( F )
r: " 7 + 5 = 12" ( V)
s: " a soma dos ângulos internos de um polígono de n lados é dada por Si = (n
- 2) . 180º " ( V )
t: " O Sol é um planeta" ( F )
w: " Um pentágono é um polígono de dez lados " ( F )

2 - Símbolos utilizados na Lógica Matemática


não
e
ou
se ... então
se e somente se
tal que
implica
equivalente
existe
existe um e somente
um
qualquer que seja

3 - O Modificador Negação
Dada a proposição p, indicaremos a sua negação por ~p . (Lê-se " não p " ).
Ex.: p: Três pontos determinam um único plano ( V )
~p: Três pontos não determinam um único plano ( F )
Obs.: duas negações equivalem a uma afirmação ou seja, em termos
simbólicos: ~(~p) = p .

4 - Operações lógicas
As proposições lógicas podem ser combinadas através dos operadores
lógicos , , e , dando origem ao que conhecemos como
proposições compostas. Assim, sendo p e q duas proposições simples,
poderemos então formar as seguintes proposições compostas: p q,p q,
p q,p q (Os significados dos símbolos estão indicados na tabela
anterior).
Estas proposições compostas recebem designações particulares, conforme
veremos a seguir.
Conjunção: p q (lê-se "p e q " ).
Disjunção: p q (lê-se "p ou q ") .
Condicional: p q (lê-se "se p então q " ).
Bi-condicional: p q ( "p se e somente se q") .
Conhecendo-se os valores lógicos de duas proposições simples p e q , como
determinaremos os valores lógicos das proposições compostas acima? Ah!
caro vestibulando! Isto é conseguido através do uso da tabela a seguir,
também conhecida pelo sugestivo nome de TABELA VERDADE.
Sejam p e q duas proposições simples, cujos valores lógicos representaremos
por 0 quando falsa (F) e 1 quando verdadeira (V). Podemos construir a
seguinte tabela simplificada:

p q p p p p q
q q q
1 1 1 1 1 1
1 0 0 1 0 0
0 1 0 1 1 0
0 0 0 0 1 1

Da tabela acima, infere-se (deduz-se) que:


a conjunção é verdadeira somente quando ambas as proposições são
verdadeiras.
a disjunção é falsa somente quando ambas as proposições são falsas.
a condicional é falsa somente quando a primeira proposição é
verdadeira e a segunda falsa.
a bi-condicional é verdadeira somente quando as proposições possuem
valores lógicos iguais.
Ex.: Dadas às proposições simples:
p: O Sol não é uma estrela (valor lógico F ou 0)
q: 3 + 5 = 8 (valor lógico V ou 1)
Temos:
p q tem valor lógico F (ou 0)
p q tem valor lógico V (ou 1)
p q tem valor lógico V (ou 1)
p q tem valor lógico F (ou 0).
Assim, a proposição composta "Se o Sol não é uma estrela então 3 + 5 = 8" é
logicamente verdadeira, não obstante ao aspecto quase absurdo do contexto
da frase!
Não quero lhe assustar, mas o fato das proposições verdadeiras (valor lógico
1) ou falsas (valor lógico 0), não podem estar associadas à analogia de que
zero (0) pode significar um circuito elétrico desligado e um (1) pode significar
um circuito elétrico ligado? Isto lembra alguma coisa vinculada aos
computadores? Pois é, caros amigos, isto é uma verdade, e é a base lógica
da arquitetura dos computadores!
Seria demais imaginar que a proposição p q pode ser associada a um
circuito série e a proposição p q a um circuito em paralelo?
Pois, as analogias são válidas e talvez tenham sido elas que mudaram o
mundo!
Paulo Marques - Feira de Santana - BA

Lógica
Matemática II
Vimos no texto anterior, a tabela verdade - reproduzida abaixo - que permite
determinar o valor lógico de uma proposição composta, conhecendo-se os
valores lógicos das proposições simples que a compõem.

p q p q p q p q p q
1 1 1 1 1 1
1 0 0 1 0 0
0 1 0 1 1 0
0 0 0 0 1 1
Nota: valor lógico verdadeiro = 1 ou V
valor lógico falso = 0 ou F
Podemos observar que é muito fácil entender (e o nosso intelecto admitir) as
regras contidas na tabela acima para a conjunção, disjunção e equivalência,
ou seja:
a conjunção "p e q" só é verdadeira quando p e q forem ambas verdadeiras.
A disjunção "p ou q" só é falsa quando p e q forem ambas falsas.
A bi-condicional só e falsa quando p e q possuem valores lógicos opostos.
Quanto à condicional "se p então q" , vamos analisá-la separadamente, de
modo a facilitar o entendimento das regras ali contidas:

p q p q
V V V
V F F
F V V
F F V

O raciocínio a seguir, será a base da nossa análise:


Se é dada uma proposição p e é possível fazer-se um raciocínio válido que
nos conduza a outra proposição q, consideraremos que p q é verdadeira.
Visto isso, vamos analisar as quatro possibilidades contidas na tabela acima:
1º) p é V e q é V: somente através de um raciocínio válido é possível partir de
uma proposição verdadeira para outra também verdadeira. Logo, p qé
verdadeira.
2º) p é V e q é F: não existe raciocínio válido capaz de , partindo-se de uma
proposição verdadeira chegar-se a uma proposição falsa. Logo, neste caso,
p q é falsa.
3º) p é F e q é V: É possível partir de uma proposição falsa e chegar-se
através de um raciocínio válido, a uma proposição verdadeira. Isto é um
pouco difícil de entender, mas acompanhe o exemplo abaixo:
Sejam as proposições:
p: 10 = 5 (valor lógico F)
q: 15 = 15 (valor lógico V)
Através de um raciocínio válido, vamos mostrar que é possível a partir de p
(falsa), chegar a q(verdadeira). Com efeito, se 10 = 5, então podemos dizer
que 5 = 10. Somando membro a membro estas igualdades vem: 10+5 = 5+10
e, portanto 15 = 15. Portanto a partir de p FALSA foi possível, através de um
raciocínio válido chegar-se a q VERDADEIRA. Logo, p q é verdadeira.
4º) p é F e q é F: É possível partir de uma proposição falsa e chegar-se
através de um raciocínio válido, a uma proposição também falsa. Senão
vejamos:
Sejam as proposições:
p: 10 = 5 (valor lógico F)
q: 19 = 9 (valor lógico F)
Através de um raciocínio válido, vamos mostrar que é possível a partir de p
FALSA, chegarmos a q também FALSA. Com efeito, se 10 = 5, então,
subtraindo uma unidade em cada membro, obteremos 9 = 4. Somando agora
membro a membro estas duas igualdades, obtemos 10+9 = 5+4 e, portanto 19
= 9, que é a proposição q dada. Logo, p q é verdadeira (V).

Exercícios:
1) Sendo p uma proposição verdadeira e q uma proposição falsa, qual o valor
lógico da proposição composta r: (p q) q?
Solução: Teremos, substituindo os valores lógicos dados: p = V , q = F e ~q =
V.
r: (V V) F , logo, pelas tabelas acima vem: r: V F e portanto r é falsa.
Valor lógico F ou 0.

2) Qual das afirmações abaixo é falsa?


a) se Marte é um planeta então 3 = 7 - 4.
b) a soma de dois números pares é um número par e 72 = 49.
c) 3 = 5 se e somente se o urso é um animal invertebrado.
d) se 102 = 100 então todo número inteiro é natural.
e) 2 = 32 - 7 ou a Terra é plana.
Analisando os valores lógicos das proposições simples envolvidas e usando-
se as tabelas anteriores, concluiremos que apenas a proposição do item (d) é
falsa, uma vez que 102 = 100 é V e "todo número inteiro é natural" é F ( o
número negativo -3 por exemplo é inteiro, mas não é natural) . Portanto,
temos V F , que sabemos ser falsa. (Veja a segunda linha da tabela
verdade acima).

Lógica
Matemática III
1 - Tautologias e Contradições
Considere a proposição composta s: (p q) (p q) onde p e q são
proposições simples
lógicas quaisquer. Vamos construir a tabela verdade da proposição s :
Considerando-se o que já foi visto até aqui, teremos:

p q p q p q (p q) (p q)
V V V V V
V F F V V
F V F V V
F F F F V

Observe que quaisquer que sejam os valores lógicos das proposições simples
p e q, a proposição composta s é sempre logicamente verdadeira. Dizemos
então que s é uma TAUTOLOGIA.
Trazendo isto para a linguagem comum, considere as proposições: p: O Sol é
um planeta
(valor lógico falso - F) e q: A Terra é um planeta plano (valor lógico falso - F),
podemos concluir que a proposição composta "Se o Sol é um planeta e a
Terra é um planeta plano então o Sol é um planeta ou a Terra é um planeta
plano" é uma proposição logicamente verdadeira.
Opostamente, se ao construirmos uma tabela verdade para uma proposição
composta, verificarmos que ela é sempre falsa, diremos que ela é uma
CONTRADIÇÃO.
Ex.: A proposição composta t: p ~p é uma contradição, senão vejamos:

p ~p p ~p
V F F
F V F

NOTA: Se uma proposição composta é formada por n proposições simples, a


sua tabela verdade possuirá 2n linhas.
Ex.: Construa a tabela verdade da proposição composta t: (p q) r
Teremos:

p q r (p q) (p q) r
V V V V V
V V F V V
V F V F V
V F F F F
F V V F V
F V F F F
F F V F V
F F F F F

Observe que a proposição acima não é Tautologia nem Contradição.


Apresentaremos a seguir, exemplos de TAUTOLOGIAS, as quais você poderá
verifica-las, simplesmente construindo as respectivas tabelas verdades:
Sendo p e q duas proposições simples quaisquer, podemos dizer que as
seguintes proposições compostas, são TAUTOLOGIAS:

1) (p q) p
2) p (p q)
3) [p (p q)] q (esta tautologia recebe o nome particular de "modus
ponens")
4) [(p q) ~q] ~p (esta tautologia recebe o nome particular de "modus
tollens")
Você deverá construir as tabelas verdades para as proposições compostas
acima e comprovar que elas realmente são tautologias, ou seja, na última
coluna da tabela verdade teremos V V V V.
NOTAS:
a) as tautologias acima são também conhecidas como regras de inferência.
b) como uma tautologia é sempre verdadeira, podemos concluir que a
negação de uma tautologia é sempre falsa, ou seja, uma contradição.
2 - Álgebra das proposições
Sejam p, q e r três proposições simples quaisquer v uma proposição
verdadeira e f uma proposição falsa. São válidas as seguintes propriedades:
a) Leis idempotentes
p p=p
p p=p

b) Leis comutativas
p q=q p
p q=q p
c) Leis de identidade
p v=p
p f=f
p v=v
p f=p
d) Leis complementares
~(~p) = p (duas negações eqüivalem a uma afirmação)
p ~p = f
p ~p = v
~v = f
~f = v
e) Leis associativas
(p q) r=p (q r)
(p q) r=p (q r)

f) Leis distributivas
p (q r) = (p q) (p r)
p (q r) = (p q) (p r)

g) Leis de Augustus de Morgan


~(p q) = ~p ~q
~(p q) = ~p ~q
h) Negação da condicional
~(p q) = p ~q
Todas as propriedades acima podem ser verificadas com a construção das
tabelas verdades.
Vamos exemplificar verificando a propriedade do item (h):
Para isto, vamos construir as tabelas verdades de ~(p q) e de p ~q :

Tabela1:
p q p q ~(p q)
V V V F
V F F V
F V V F
F F V F
Tabela 2:
p q ~q p ~q
V V F F
V F V V
F V F F
F F V F

Observando as últimas colunas das tabelas verdades 1 e 2, percebemos que


elas são iguais, ou seja, ambas apresentam a seqüência F V F F, o que
significa que ~ (p q) = p ~q .
Exs.:
1) Qual a negação da proposição composta: "Eu estudo e aprendo"?
Do item (g) acima, concluímos que a negação procurada é: "Eu não estudo ou
não aprendo".
2) Qual a negação da proposição "O Brasil é um país ou a Bahia é um estado"
?
Do item (g) acima, concluímos que a negação é: "O Brasil não é um país e a
Bahia não é um estado".
3) Qual a negação da proposição: "Se eu estudo então eu aprendo" ?
Conforme a propriedade do item (h) acima, concluímos facilmente que a
negação procurada é: "Eu estudo e não aprendo"

Relações
Binárias
INTRODUÇÃO
Neste capítulo, vamos estudar apenas os tópicos necessários para um
perfeito entendimento do assunto que será abordado no capítulo seguinte:
Funções.
PAR ORDENADO: conjunto ordenado de dois elementos, representado pelo
símbolo (x; y) onde x e y são números reais, denominados respectivamente
de abscissa e ordenada.
Ex: Par ordenado (6; -3) : abscissas = 6 e ordenada = -3.
Propriedade: dois pares ordenados são iguais, quando são respectivamente
iguais as abscissas e as ordenadas. Em termos simbólicos:
(x;y) = (w;z) x=wey=z
Ex: (2x - 4; y) = (- x; 7) 2x - 4 = - x e y = 7 x = 4/3 e y = 7.
PLANO CARTESIANO: também conhecido como sistema de coordenadas
retangulares; Trata-se de um conceito introduzido no século XVII pelo
matemático e filósofo francês René Descartes, para representar graficamente
o par ordenado (xo;yo). Consiste basicamente de dois eixos orientados que se
interceptam segundo um angulo reto, num ponto denominado origem. O eixo
horizontal é denominado eixo das abscissas e o eixo vertical é denominado
eixo das ordenadas. Denominamos o ponto O de origem do plano cartesiano,
sendo nulas a sua abscissa e a sua ordenada, ou seja, O (0; 0).
Observe que o plano cartesiano pode ser subdividido em quatro regiões, que
são denominadas Quadrantes. Temos então o seguinte quadro resumo:
QUADRANTE ABCISSA ORDENADA
1º quadrante + +
2º quadrante - +
3º quadrante - -
4º quadrante + -

Obs:
1) a equação do eixo Ox é y = 0 e do eixo Oy é x = 0.
2) o gráfico de y = x é uma reta denominada bissetriz do primeiro quadrante.
3) o gráfico de y = -x é uma reta denominada bissetriz do segundo quadrante.
MÓDULO DE UM NÚMERO REAL : Entende-se por módulo ou valor absoluto
do número real a e
escreve-se a , o seguinte:
a = a se a 0
a = -a se a 0
Por esta definição, o módulo de um número positivo ou nulo (não negativo) é
o próprio número e o módulo de um número negativo é o simétrico desse
número.
Exs: 7 =7; -5 =5; 0 =0; 7 - 10 = -3 =3
São válidas as seguintes propriedades relativas às igualdades e
desigualdades modulares:
P1) w =0 w=0
P2) w = b , onde b 0 w = b ou w = - b
P3) w b , onde b 0 w b ou w -b
P4) w b , onde b 0 -b w b
PRODUTO CARTESIANO: Dados dois conjuntos A e B , definimos o produto
cartesiano de A por B , que indicamos pelo símbolo AxB , ao conjunto de
todos os pares ordenados (x;y)
onde x Aey B. Em termos simbólicos, podemos escrever:
AxB = { (x;y); x Aey B}
Ex: {0;2;3} x {5; 7} = { (0;5) , (0; 7) , (2;5) , (2;7) , (3;5}, (3;7) }
Obs.: Sendo A e B conjuntos quaisquer, temos:
a) o produto cartesiano de um conjunto A por ele mesmo, ou seja, AxA é
representado por A2 .
Assim, podemos escrever: A x A = A2.
b) A x B B x A (o produto cartesiano é uma operação não comutativa)
c) A x =
d) n(A x B) = n(A) . n(B) , onde n(A) e n(B) representam os números de
elementos de A e de B, respectivamente.
RELAÇÃO BINÁRIA
Dados dois conjuntos A e B , chama-se relação de A em B , a qualquer
subconjunto de AxB. Em termos simbólicos, sendo uma relação de A em B
, podemos escrever:
= { (x;y) AxB ; x y}
Ex: = { (0;3) , (2;5) , (3;0) } é uma relação de A = { 0;2;3;4} em B = {3;5;0}.
NOTAS:
1) AxB
2) o conjunto A é o conjunto de partida e B o conjunto de chegada ou
contradomínio.
3) se (x;y) , então dizemos que y é imagem de x , pela relação .
4) a expressão x y equivale a dizer que (x;y) .
5) dada uma relação = { (x;y) AxB ; x y } , o conjunto dos valores de
x chama- se domínio da relação e o conjunto dos valores de y chama-se
conjunto imagem da relação.
6 - o número de relações possíveis de A em B é dado por 2n(A).n(B) .
7 - Dada uma relação = { (x,y) AxB ; x y } , define-se a relação
-1
inversa como sendo:
-1
= { (y,x) BxA ; y x }.
Ex: F = { (0,2) , (3.5) , (4,8) , ( 5,5) }
F-1 = { (2,0) , (5,3) , (8,4) , (5,5) }.
Agora, tente resolver as questões a seguir.
1 - Sendo A = {x N; 1 x 4} e B = {x Z; 5 x 10}, o conjunto
imagem da relação
S = {(x, y) AXB; x + y = 9} é:
a) {4,5,6}
*b) {6,7}
c) {5,6,7}
d) {7}
e) {1}
2 - Sendo n(A) = 2 e n(B) = 3, então o número de elementos de p(A) X p(B) é:
a)4
b)8
c)16
*d)32
e)64
3 - UFBA - Sejam: A = { 1 , 5 } ; B = { -1 , 0 , 1 }; R = {(x , y) AxB } e
F = conjunto dos pontos do plano, simétricos aos pontos de R em relação à
primeira bissetriz. Dos conjuntos e relações dados, pode-se afirmar:
I) A imagem da relação inversa de R é o conjunto A.
II) O domínio de F é o conjunto B.
III) R tem 5 elementos.
IV) Em F há pontos pertencentes ao eixo Ox.
V) Existe um único ponto de R que pertence à primeira bissetriz.
São verdadeiras:
a) todas
b) nenhuma
c) III e IV
*d) I, II e V
e)somente I
4 - UEFS - Sendo A = { 1, 3 } e B = [-2 , 2], o gráfico cartesiano de AxB é
representado por:
a) 4 pontos
b) 4 retas
c)um retângulo
d)retas paralelas a Ox
*e) dois segmentos de reta
5 - Sabendo-se que n(AxB) = 48 , n(BxC) = 72 , n(p(A)) = 256, podemos
afirmar que n(AxC) é:
a)64
b)72
*c)96
d)128
e)192
6 - UFCE - Dado um conjunto C , denotemos por n(p(C)) o número de
elementos do conjunto das partes de C. Sejam A e B dois conjuntos não
vazios, tais que n(p(AxB)) = 128 e n(B) n(A). Calcule n(p(B)) / n(p(A)).
Resp: 64

Funções I
1 - Definição
Dados dois conjuntos A e B não vazios, chama-se função (ou aplicação) de A
em B, representada por
f:A B ; y = f(x) , a qualquer relação binária que associa a cada elemento
de A , um único elemento de B .
Veja o capítulo Relações Binárias
Portanto, para que uma relação de A em B seja uma função, exige-se que a
cada x A esteja associado um único y B, podendo, entretanto existir y
B que não esteja associado a nenhum elemento pertencente ao conjunto A.

Obs : na notação y = f(x) , entendemos que y é imagem de x pela função f, ou


seja:
y está associado a x através da função f.
Exemplo:

f(x) = 4x+3 ; então f(2) = 4.2 + 3 = 11 e portanto , 11 é imagem de 2 pela


função f ;
f(5) = 4.5 + 3 = 23 , portanto 23 é imagem de 5 pela função f , f(0) = 4.0 + 3 =
3, etc.
Para definir uma função, necessitamos de dois conjuntos (Domínio e
Contradomínio ) e de uma fórmula ou uma lei que relacione cada elemento
do domínio a um e somente um elemento do contradomínio .
Quando D(f) R e CD(f) R, sendo R o conjunto dos números reais ,
dizemos que a função f é uma função real de variável real . Na prática ,
costumamos considerar uma função real de variável real como sendo apenas
a lei y = f(x) que a define , sendo o conjunto dos valores possíveis para x ,
chamado de domínio e o conjunto dos valores possíveis para y , chamado de
conjunto imagem da função . Assim, por exemplo, para a função definida por
y = 1/x , temos que o seu domínio é D(f) = R* , ou seja o conjunto dos reais
diferentes de zero (lembre-se que não existe divisão por zero) , e o seu
conjunto imagem é também R* , já que se y = 1/x , então x = 1/y e portanto y
também não pode ser zero .
Dada uma função f : A B definida por y = f(x),
podemos representar os pares ordenados (x,y) f onde x Aey B
,num sistema de coordenadas cartesianas .
O gráfico obtido será o gráfico da função f .

Assim , por exemplo , sendo dado o gráfico cartesiano de uma função f ,


podemos dizer que:
a ) a projeção da curva sobre o eixo dos x , nos dá o domínio da função .

b ) a projeção da curva sobre o eixo dos y , nos dá o conjunto imagem da


função .

c ) toda reta vertical que passa por um ponto do domínio da função ,


intercepta o gráfico da função em no máximo um ponto .
Veja a figura abaixo:
2 -Tipos de funções
2.1 - Função sobrejetora
É aquela cujo conjunto imagem é igual ao contradomínio .
Exemplo:

2.2 - Função injetora


Uma função y = f(x) é injetora quando elementos distintos do seu domínio ,
possuem imagens distintas,
isto é:
x1 x2 f(x1) f(x2) .
Exemplo:
2.3 - Função bijetora
Uma função é dita bijetora , quando é ao mesmo tempo , injetora e sobrejetora
.
Exemplo:

Exercícios resolvidos:
1 - Considere três funções f, g e h, tais que:
A função f atribui a cada pessoa do mundo, a sua idade.
A função g atribui a cada país, a sua capital
A função h atribui a cada número natural, o seu dobro.
Podemos afirmar que, das funções dadas, são injetoras:
a) f, g e h
b) f e h
c) g e h
d) apenas h
e) nenhuma delas
Solução:

Sabemos que numa função injetora, elementos distintos do domínio, possuem


imagens distintas, ou seja:
x1 x2 f(x1) f(x2).

Logo, podemos concluir que:

f não é injetora, pois duas pessoas distintas podem ter a mesma idade.
g é injetora, pois não existem dois países distintos com a mesma capital.
h é injetora, pois dois números naturais distintos, possuem os seus dobros
também distintos.
Assim é que concluímos que a alternativa correta é a de letra C.
2 - Seja f uma função definida em R - conjunto dos números reais - tal que
f(x - 5) = 4x. Nestas condições, pede-se determinar f(x + 5).
Solução:

Vamos fazer uma mudança de variável em f(x - 5) = 4x, da seguinte forma:


x-5=u x=u+5

Substituindo agora (x - 5) pela nova variável u e x por (u + 5), vem:


f(u) = 4(u + 5) f(u) = 4u + 20
Ora, se f(u) = 4u + 20, teremos:
f(x + 5) = 4(x+5) + 20 f(x+5) = 4x + 40
Agora resolva este:

A função f em R é tal que f(2x) = 3x + 1. Determine 2. f(3x + 1).


Resp: 9x + 5
3 - Paridade das funções
3.1 - Função par
A função y = f(x) é par, quando x D(f) , f(- x ) = f(x) , ou seja, para todo
elemento do seu domínio,
f( x ) = f ( - x ). Portanto, numa função par, elementos simétricos possuem a
mesma imagem. Uma conseqüência desse fato é que os gráficos cartesianos
das funções pares são curvas simétricas em relação ao eixo dos y ou eixo das
ordenadas.
Exemplo:

y = x4 + 1 é uma função par, pois f(x) = f(-x), para todo x. Por exemplo,
f(2) = 24 + 1 = 17 e f(- 2) = (-2)4 + 1 = 17
O gráfico abaixo, é de uma função par.
4.2 - Função ímpar
A função y = f(x) é ímpar , quando x D(f) , f( - x ) = - f (x) , ou seja,
para todo elemento do seu domínio, f( - x) = - f( x ). Portanto, numa função
ímpar, elementos simétricos possuem imagens simétricas. Uma conseqüência
desse fato é que os gráficos cartesianos das funções ímpares são curvas
simétricas em relação ao ponto (0,0), origem do sistema de eixos cartesianos.
Exemplo:

y = x3 é uma função ímpar pois para todo x, teremos f(- x) = - f(x).


Por exemplo, f( - 2) = (- 2)3 = - 8 e - f( x) = - ( 23 ) = - 8.
O gráfico abaixo é de uma função ímpar:
Nota: se uma função y = f(x) não é par nem ímpar, dizemos que ela não
possui paridade.

Exemplo:

O gráfico abaixo, representa uma função que não possui paridade, pois a
curva não é simétrica em relação ao eixo dos x e, não é simétrica em relação
à origem.

Funções
II
1 - FUNÇÃO INVERSA
Dada uma função f: A B, se f é bijetora, então se define a função inversa f
-1
como sendo a função de B em A, tal que f -1 (y) = x.
Veja a representação a seguir:

É óbvio então que:


a) para obter a função inversa , basta permutar as variáveis x e y .
b) o domínio de f -1 é igual ao conjunto imagem de f .
c) o conjunto imagem de f -1 é igual ao domínio de f .
d) os gráficos de f e de f -1 são curvas simétricas em relação à reta y = x ou
seja , à bissetriz do primeiro quadrante .
Exemplo:
Determine a INVERSA da função definida por y = 2x + 3.
Permutando as variáveis x e y, fica: x = 2y + 3
Explicitando y em função de x, vem:
2y = x - 3 y = (x - 3) / 2, que define a função inversa da função dada.
O gráfico abaixo, representa uma função e a sua inversa.
Observe que as curvas representativas de f e de f-1, são simétricas em relação
à reta
y = x, bissetriz do primeiro e terceiro quadrantes.

Exercício resolvido:
A função f: R R, definida por f(x) = x2:
a) é inversível e sua inversa é f -1 (x) = x
b) é inversível e sua inversa é f -1(x) = - x
c) não é inversível
d) é injetora
e) é bijetora.
SOLUÇÃO:
Já sabemos que somente as funções bijetoras são inversíveis, ou seja,
admitem função inversa.
Ora, a função f(x) = x2, definida em R - conjunto dos números reais - não é
injetora, pois elementos distintos possuem a mesma imagem. Por exemplo,
f(3) = f(-3) = 9. Somente por este motivo, a função não é bijetora e, em
conseqüência, não é inversível.
Observe também que a função dada não é sobrejetora, pois o conjunto
imagem da função f(x) = x2 é o conjunto R + dos números reais não negativos,
o qual não coincide com o contradomínio dado que é
igual a R. A alternativa correta é a letra C.
2 - FUNÇÃO COMPOSTA
Chama-se função composta ( ou função de função ) à função obtida
substituindo-se a variável independente x , por uma função.
Simbologia : fog (x) = f(g(x)) ou gof (x) = g(f(x)) .
Veja o esquema a seguir:

Obs.: atente para o fato de que fog gof , ou seja, a operação " composição
de funções " não é comutativa .
Exemplo:
Dadas às funções f(x) = 2x + 3 e g(x) = 5x, pede-se determinar gof(x) e fog(x).
Teremos:
gof(x) = g[f(x)] = g(2x + 3) = 5(2x + 3) = 10x + 15
fog(x) = f[g(x)] = f(5x) = 2(5x) + 3 = 10x + 3
Observe que fog gof .
Exercícios resolvidos:
1 - Sendo f e g duas funções tais que: f(x) = ax + b e g(x) = cx + d . Podemos
afirmar que a igualdade gof(x) = fog(x) ocorrerá se e somente se:
a) b(1 - c) = d(1 - a)
b) a(1 - b) = d(1 - c)
c) ab = cd
d) ad = bc
e) a = bc
SOLUÇÃO:
Teremos:
fog(x) = f[g(x)] = f(cx + d) = a(cx + d) + b fog(x) = acx + ad + b
gof(x) = g[f(x)] = g(ax + b) = c(ax + b) + d gof(x) = cax + cb + d
Como o problema exige que gof = fog, fica:
acx + ad + b = cax + cb + d
Simplificando, vem:
ad + b = cb + d
ad - d = cb - b d(a - 1) = b(c - 1), que é equivalente a d(a - 1) = b(c - 1), o
que nos leva a concluir que a alternativa correta é a letra A.
2 - Sendo f e g duas funções tais que fog(x) = 2x + 1 e g(x) = 2 - x então f(x) é:
a) 2 - 2x
b) 3 - 3x
c) 2x - 5
*d) 5 - 2x
e) uma função par.
SOLUÇÃO:
Sendo fog(x) = 2x + 1, temos: f[g(x)] = 2x + 1
Substituindo g(x) pelo seu valor, fica: f(2 - x) = 2x + 1
Fazendo uma mudança de variável, podemos escrever 2 - x = u, sendo u a
nova variável. Portanto, x = 2 - u.
Substituindo, fica:
f(u) = 2(2 - u) + 1 f(u) = 5 - 2u
Portanto, f(x) = 5 - 2x, o que nos leva à alternativa D.
Agora resolva esta:
Dadas às funções f(x) = 4x + 5 e g(x) = 2x - 5k ocorrerá gof(x) = fog(x) se e
somente se k for igual a:
*a) -1/3
b) 1/3
c) 0
d) 1
e) -1

Funções
III
Tipos particulares de funções
1 FUNÇÃO CONSTANTE
Uma função é dita constante quando é do tipo f(x) = k , onde k não depende
de x .
Exemplos:
a) f(x) = 5
b) f(x) = -3
Nota : o gráfico de uma função constante é uma reta paralela ao eixo dos x .
Veja o gráfico a seguir:

2 FUNÇÃO DO 1º GRAU
Uma função é dita do 1º grau , quando é do tipo y = ax + b , onde a 0.
Exemplos :
f(x) = 3x + 12 ( a = 3 ; b = 12 )
f(x) = -3x + 1 (a = -3; b = 1).
Propriedades da função do 1º grau:
1) o gráfico de uma função do 1º grau é sempre uma reta.

2) na função f(x) = ax + b, se b = 0, f é dita função linear e se b 0 f é dita


função afim.
Nota: consta que o termo AFIM foi introduzido por Leonhard Euler (pronuncia-
se óiler) - excepcional matemático suíço - 1701/1783).
3) o gráfico intercepta o eixo dos x na raiz da equação f(x) = 0 e, portanto, no
ponto de
abscissa x = - b/a .
4) o gráfico intercepta o eixo dos y no ponto (0 , b) , onde b é chamado
coeficiente linear .
5) o valor a é chamado coeficiente angular e dá a inclinação da reta .
6) se a 0, então f é crescente.
7) se a 0, então f é decrescente.
8) quando a função é linear, ou seja, y = f(x) = ax, o gráfico é uma reta que
sempre passa na origem.
Exercício resolvido:
1 - Determine a função f(x) = ax + b, sabendo-se que f(2) = 5 e f(3) = -10.
SOLUÇÃO:
Podemos escrever:
5 = 2.a + b
-10 = 3.a + b
Subtraindo membro a membro, vem:
5 - (- 10) = 2.a + b - (3.a + b)
15 = - a a = - 15
Substituindo o valor de a na primeira equação (poderia ser na segunda), fica:
5 = 2.(- 15) + b b = 35.
Logo, a função procurada é: y = - 15x + 35.
Agora resolva esta:
A função f é definida por f(x) = ax + b. Sabe-se que f(-1) = 3 e f(3) = 1, então
podemos afirmar que f(1) é
igual a:
*a) 2
b) -2
c) 0
d) 3
e) -3
3 FUNÇÃO DO 2º GRAU
Uma função é dita do 2º grau quando é do tipo f(x) = ax2 + bx + c , com a
0.
Exemplos: f(x) = x2 - 2x + 1 ( a = 1 , b = -2 , c = 1 ) ;
y = - x2 ( a = -1 , b = 0 , c = 0 )
Gráfico da função do 2º grau y = ax2 + bx + c: é sempre uma parábola de
eixo vertical .
Propriedades do gráfico de y = ax2 + bx + c:
1) se a 0 a parábola tem um ponto de mínimo.
2) se a 0 a parábola tem um ponto de máximo
3) o vértice da parábola é o ponto V(xv, yv) onde:
xv = - b/2a
yv = - /4a, onde = b2 - 4ac
4) a parábola intercepta o eixo dos x nos pontos de abscissa x' e x'' , que são
as raízes da
equação ax2 + bx + c = 0 .
5) a parábola intercepta o eixo dos y no ponto (0 , c) .
6) o eixo de simetria da parábola é uma reta vertical de equação x = - b/2a.
7) ymax = - / 4a ( a 0)
8) ymin = - /4a ( a 0)
9) Im(f) = { y R;y - /4a } ( a 0)
10) Im(f) = { y R;y - /4a} ( a 0)
11) Forma fatorada : sendo x1 e x2 as raízes da de f(x) = ax2 + bx + c , então
ela pode ser escrita na forma fatorada a seguir :
y = a(x - x1).(x - x2)
Exercícios Resolvidos
1 - UCSal - Sabe-se que -2 e 3 são raízes de uma função quadrática. Se o
ponto
(-1 , 8) pertence ao gráfico dessa função, então:
a) o seu valor máximo é 1,25
b) o seu valor mínimo é 1,25
c) o seu valor máximo é 0,25
d) o seu valor mínimo é 12,5
*e) o seu valor máximo é 12,5.
SOLUÇÃO:
Sabemos que a função quadrática, pode ser escrita na forma fatorada:
y = a(x - x1)(x - x2) , onde x1 e x2, são os zeros ou raízes da função.
Portanto, poderemos escrever:
y = a[x - (- 2 )](x - 3) = a(x + 2)(x - 3)
y = a(x + 2)(x - 3)
Como o ponto (-1,8) pertence ao gráfico da função, vem:
8 = a(-1 + 2)(-1 - 3)
8 = a(1)(-4) = - 4.a
Daí vem: a = - 2
A função é, então: y = -2(x + 2)(x - 3) , ou y = (-2x -4)(x - 3)
y = -2x2 + 6x - 4x + 12
y = -2x2 + 2x + 12
Temos então: a = -2, b = 2 e c = 12.
Como a é negativo, concluímos que a função possui um valor máximo.
Isto já elimina as alternativas B e D.
Vamos então, calcular o valor máximo da função.
= b2 - 4ac = 22 - 4 .(-2).12 = 4+96 = 100
Portanto, yv = - 100/4(-2) = 100/8 = 12,5
Logo, a alternativa correta é a letra E.
2 - Que número excede o seu quadrado o máximo possível?
*a) 1/2
b) 2
c) 1
d) 4
e) -1/2
SOLUÇÃO:
Seja x o número procurado.
O quadrado de x é x2 .
O número x excede o seu quadrado , logo: x - x2.
Ora, a expressão anterior é uma função quadrática y = x - x2 .
Podemos escrever:
y = - x2 + x onde a = -1, b = 1 e c = 0.
O valor procurado de x, será o xv (abscissa do vértice da função).
Assim,
xv = - b / 2.a = - 1 / 2(-1) = 1 / 2
Logo, a alternativa correta é a letra A .
Agora resolva estes similares:
1 - A diferença entre dois números é 8. Para que o produto seja o menor
possível, um deles deve ser:
a) 16
b) 8
*c) 4
d) -4
e) -16
2 - A diferença entre dois números é 8. O menor valor que se pode obter para
o produto é:
a) 16
b) 8
c) 4
d) -4
*e) -16

Funções
IV
Exercícios resolvidos e propostos
1 - Se f(x) = 1/[x(x+1)] com x 0ex -1, então o valor de S = f(1) + f(2) +
f(3) + ... + f(100) é:
a)100
b) 101
c) 100/101
d) 101/100
e) 1
SOLUÇÃO:
Temos:

Portanto,
f(1) = 1/1 - 1/2
f(2) = 1/2 - 1/3
f(3) = 1/3 - 1/4
f(4) = 1/4 - 1/5
f(5) = 1/5 - 1/6
.........................
..........................
...........................
f(99) = 1/99 - 1/100
f(100) = 1/100 - 1/101
Somando membro a membro as igualdades acima (observe que os termos
simétricos se anulam entre si), vem:
f(1) + f(2) + f(3) + ... + f(100) = 1 - 1/101 = 100/101, o que nos leva à
alternativa C.
2 - UCSal - Sejam f e g funções de R em R, sendo R o conjunto dos números
reais, dadas por f(x) = 2x - 3 e f(g(x)) = -4x + 1. Nestas condições, g(-1) é igual
a:
a) -5
b) -4
c) 0
*d) 4
e) 5
SOLUÇÃO:
Como f(x) = 2x -3, podemos escrever: f[g(x)] = 2.g(x) - 3 = - 4x + 1
Logo, 2.g(x) = - 4x +4 g(x) = -2x + 2
Assim, g(-1) = -2(-1) + 2 = 4.
Logo, a alternativa correta é a letra D.
3 - O conjunto imagem da função y = 1 / (x - 1) é o conjunto:
a) R - { 1 }
b) [0,2]
c) R - {0}
d) [0,2)
e) (-2 ,2]
SOLUÇÃO:
Se y = 1 / (x - 1), então x - 1 = 1 / y.
Como o conjunto imagem é o conjunto dos valores de y, percebemos que y
não pode ser nulo, pois não existe divisão por zero.
Logo, o conjunto imagem é R - {0}, o que nos leva à alternativa C.
4 - Determine o domínio da função y = (x+1) / (x - 2).
SOLUÇÃO:
Como não existe divisão por zero, vem imediatamente que: x - 2 0 x
2.
Logo, o domínio da função será D = R - {2}, onde R é o conjunto dos números
reais.
Agora resolva estes:
1 - UFBA - Se f (g (x) ) = 5x - 2 e f (x) = 5x + 4 , então g(x) é igual a:
a) x - 2
b) x - 6
c) x - 6/5
d) 5x - 2
e) 5x + 2
Resp: C
2 - A função f é tal que f(2x + 3) = 3x + 2. Nestas condições, f(3x + 2) é igual
a:
a) 2x + 3
b) 3x + 2
c) (2x + 3) / 2
d) (9x + 1) /2
e) (9x - 1) / 3
Resp: D
3 - Qual o domínio da função y = (x - 4)1/4 ?
Resp: D = [4, ).
4 - Qual o conjunto imagem da função y = 1/x?
Resp: Im = R - {0}.
5 - Qual o domínio da função y = (senx)/x ?
Resp: D = R - {0}.
6 - Sendo f(x) = senx e g(x) = logx, pede-se determinar o valor de g[f( /2)].
Resp: 0
7 - Elabore o gráfico da função y = [x] , de domínio R, onde [x] significa o
maior inteiro contido em x, assim definido:
[x] = maior inteiro que não supera x.
Exemplos:
[2] = 2
[2,01] = 2
[0,833...] = 0
[-3,67...] = -4
[-1,34...] = -2, etc.
Resp:

UMA CERTA
FUNÇÃO
Seja f uma função definida para todo x real, satisfazendo as condições:

Então, f(–3) vale:


a) –6 b) 0 c) ½ d) 2 e) –1

Solução:

Podemos escrever, usando as definições dadas no enunciado:


Para x = -3:
f(-3 + 3) = f(-3).f(3) ou f(0) = f(-3).2
Podemos também escrever:
Para x = 0:
f(0 + 3) = f(0).f(3) ou f(3) = f(0).f(3), de onde concluímos que o valor de f(0) é:
f(0) = f(3)/f(3) = 2/2 = 1.
Daí, vem, por substituição, lembrando que f(0) = f(-3).2 e que f(0) = 1:
1 = f(-3).2, de onde concluímos imediatamente f(-3) = 1/2, o que nos leva à
alternativa C.
Agora resolva este:
PUC-RS - Se f é uma função tal que f(1) = a, f( ) = b e f(x + y) = f(x) . f(y),
x, y R, então f(2 + ) é igual a:

a) a b) b c) a2b d) ab2 e) a + b

Resposta: alternativa C.
Simbologia:
- qualquer que seja, para todo.
- pertence a
- número irracional pi, cujo valor aproximado é 3,1416.

Uma certa classe


de funções
Determine todas as funções f tais que

quaisquer que sejam os números reais x, y.


Solução:
Fazendo x = y = 0, já que todas as funções f que satisfazem à condição dada,
pelo enunciado, estão definidas para todo x e y real, vem:
f(02) – f(02) + 2.0 + 1 = f(0 + 0).f(0 – 0)
Daí, vem:
f(0) – f(0) + 1 = f(0).f(0) = [f(0)] 2 .
Como f(0) - f(0) = 0, vem:
0 + 1 = [f(0)] 2
1 = [f(0)] 2, de onde vem: f(0) = 1.
Pelo conceito de função , o elemento 0 não poderá ter duas imagens (1 e –1),
e, portanto, apenas um desses valores deve ser válido.
Fazendo y = x na igualdade dada no problema, vem:
f(x2) – f(x2) + 2x + 1 = f(x + x) . f(x – x)
Como f(x2) = f(x2), vem da igualdade acima:
2x + 1 = f(2x).f(0)
Fazendo uma mudança de variável, colocando 2x = u, vem:
u + 1 = f(u).f(0)
Supondo f(0) = 1 (do resultado obtido acima), fica:
f(u) = u + 1
Supondo f(0) = -1 (também do resultado obtido acima), fica:
f(u) = - (u + 1)
Como é indiferente usar o símbolo u ou x, teremos:
f(x) = x + 1 ou f(x) = - (x + 1).
Seriam estas duas funções, a solução do problema proposto.
Mas, como é dito que f é uma função, f(0) não pode ter duas imagens (1 e –1),
conforme já foi relatado anteriormente.
Temos então que verificar os dois resultados, para saber qual a que satisfaz
ao problema proposto.
Consideremos que y = f(x) = x + 1, seja uma solução procurada.
Como, já sabemos do enunciado que:

Vem,
f(x) = x + 1
f(x2) = x2 + 1
y = f(x) y2 = [f(x)]2 = (x + 1)2
f(y2) = y2 + 1 = (x +1)2 + 1
f(x + y) = f[x + (x +1)] = f(2x + 1) = (2x + 1) + 1 = 2x + 2
f(x – y) = f[x – (x + 1)] = f(-1) = -1 + 1 = 0
Substituindo, vem:
x2 + 1 – [(x +1)2 + 1]+ 2x + 1 = (2x + 1).0
x2 + 1 –(x2 + 2x + 1 + 1)+ 2x + 1 = 0
x2 + 1 – x2 – 2x – 2 + 2x + 1 = 0
Simplificando, vem 0 = 0, e, portanto, a função y = f(x) = x + 1, satisfaz ao
problema.
Por extensão, sabendo que f é uma função, é razoável supor que o valor de
f(0) (que deve ser único, pelo conceito de função ) é igual a f(0) = 1 e que o
resultado f(0) = -1, não serve.
Deixamos como exercício para o visitante, verificar que f(x) = - (x+1), não
satisfaz ao problema proposto. Isto é fácil; basta seguir os passos indicados
acima para f(x) = y = x + 1.

Portanto, a única função que obedece ao critério do enunciado do problema


proposto, é a função y = x + 1.

Resp: Só existe uma função que satisfaz à condição dada no enunciado e


esta função é y = f(x) = x + 1.
Nota: esta questão apareceu na prova da 9ª Olimpíada de Matemática do
Cone Sul, realizada no ano de 1998, na cidade de Salvador - BA.

Calcule o valor da
função
Seja f uma função tal que f(n + 1) = [(2.f(n) + 1)] / 2 para todo n
inteiro positivo e
f(1) = 2. Nestas condições, o valor de f(101) é:

(a) 102
(b) 101
(c) 86
(d) 76
(e) 52

Solução:

Teremos, fazendo n = 1, 2, 3, 4, ... na expressão f(n+1) = [(2.f(n) +


1) / 2:

n=1 f(1 + 1) = f(2) = [2.f(1) + 1] / 2 = [2.2 + 1] / 2 = 5 / 2


n=2 f(2 + 1) = f(3) = [2.f(2) + 1] / 2 = [2.(5 / 2) + 1] / 2 = 3
n=3 f(3 + 1) = f(4) = [2.f(3) + 1] / 2 = [2.3 + 1] / 2 = 7 / 2
n=4 f(4 + 1) = f(5) = [2.f(4) + 1] / 2 = [2.(7 / 2) + 1] / 2 = 4
...........................................................................................................
...........................................................................................................

Vamos resumir os valores obtidos acima:

f(1) = 2 = 4 / 2
f(2) = 5 / 2
f(3) = 3 = 6 / 2
f(4) = 7 / 2
f(5) = 4 = 8 / 2
........................
........................

Observe que o denominador é sempre 2 e o numerador é o valor de


n acrescido de 3 unidades, pois:

f(1) = 4 / 2 e 4 = 1 + 3
f(2) = 5 / 2 e 5 = 2 + 3
f(3) = 6 / 2 e 6 = 3 + 3
f(4) = 7 / 2 e 7 = 4 + 3
f(5) = 8 / 2 e 8 = 5 + 3
.......................................
.......................................

Observe que a lei de formação para um n inteiro positivo qualquer


será então
f(n) = (n + 3) / 2
Portanto, o valor de f(101) será obtido fazendo n = 101, o que
resulta:

f(101) = (101 + 3) / 2 = 104 / 2 = 52

Agora resolva este:

Seja f uma função tal que f(n + 1) = [(2.f(n) + 1)] / 2 para todo n
inteiro positivo e
f(1) = 2. Nestas condições, determine o valor de f(105) + f(109).

Resposta: 110

Módulo
I
Módulo ou valor absoluto
“O valor positivo do número real, desprezando-se o sinal. Escreve-se x .
Por exemplo: 3 = 3; -4 = 4,
e 0 = 0".

1 - INTRODUÇÃO
Genericamente, podemos dizer que o módulo de um número real, é o número
sem o seu sinal. Assim, o módulo de -7 é 7, o módulo de -5 é 5, ... , etc.
Para representar o módulo de um número real a , usamos a notação a ,
que lê-se módulo de a.
Podemos dizer que módulo é a operação de apagar o sinal, conforme pode-se
perceber nos exemplos acima.
2 - GENERALIDADES
2.1 - Seja x um número real qualquer. Das considerações do item (1)
acima, seria correto dizer que x = x ?. Claro que não! Senão vejamos:
Suponha x = -3; teremos: -3 = 3 = -(-3) = - x. Portanto para x negativo,
vale a igualdade x = -x. Não se esqueça do fato que se x é negativo,
então -x é positivo.
Somente para x positivo ou nulo é que vale a igualdade x = x.
Das considerações acima podemos concluir que o módulo ou valor absoluto
de um número real qualquer é sempre positivo ou nulo. Lembre-se que 0
= 0.
Exercícios resolvidos.

1 - Qual
o conjunto solução da equação x+1 + x-1 = 10 ?
Solução: Considere a reta numerada abaixo onde -1 e +1 são os valores que
anulam as expressões entre módulo:

Temos que considerar 3 casos:


1º caso: x -1: neste caso, tanto x -1 como x+1 são negativos, e portanto:
x-1 = -(x-1) e x+1 = -(x+1) . Assim, substituindo as expressões em
módulo pelos seus valores válidos nesse intervalo, vem:
-(x-1) + [-(x+1)] = 10 \ -x + 1 -x -1 = 10 e, portanto x = -5.
2º caso: -1 x 1: neste caso, x + 1 é positivo e x -1 é negativo, e,
portanto:
x+1 = x+1 e x-1 = -(x - 1). Assim, substituindo as expressões em
módulo pelos seus valores válidos nesse intervalo, vem:
x + 1 + [-(x - 1)] = 10 e, logo chegamos à igualdade 0.x = 8 que é impossível,
pois não existe divisão por zero. Logo, nesse intervalo, a equação não tem
solução.
3º caso: x 1 : nesse caso, tanto x + 1 quanto x - 1 são positivos e, portanto,
teremos:
x-1 =x-1e x+1 = x + 1; substituindo as expressões em módulo
pelos seus valores válidos nesse intervalo, vem:
x - 1 + x + 1 = 10 2x = 10 e, logo x = 5. Portanto, o conjunto solução da
equação dada é: S = { -5, 5 }.
2 - Agora você deve resolver a equação: 2x + 6 + 2x - 6 = 80.
Resp: x = -20 ou x = 20 ou S = { -20, 20 }.
2
3 - Resolva a equação: x - 10 x + 16 = 0.
Solução: Temos de considerar dois casos:
1º caso: x 0 : neste caso, já sabemos que x = -x. Substituindo as
expressões em módulo pelos seus valores válidos nesse intervalo, vem:
(-x)2 - ( - 10x ) + 16 = 0 x2 + 10x + 16 = 0, que é uma equação do 2º grau
de raízes -8 e -2 (verifique).
2º caso: x 0 : nesse caso, sabemos que x = x . Logo, substituindo,
vem:
x2 - 10x + 16 = 0, que é uma equação do 2º grau de raízes 2 e 8 (verifique).
Logo, o conjunto solução da equação dada é: S = { - 8, - 2, 2, 8 }.
2
4 - Resolva a equação: x - 20 x + 64 = 0.
Resp: S = { -16, -4, 4, 16 }
EXERCÍCIOS PROPOSTOS
1 - Sendo y = x-5 + 3x - 21 + 12 - 3x , se 4 x 5,
podemos afirmar que:
a) y =14 - x
b) y = x - 14
c) y = 7x + 38
d) y = 0
e) y = 14x
2 - Resolva as seguintes equações modulares em R, conjunto dos números
reais:
a) 2x - 3 =5
b) 3x = x+2
2
c) x -4 =5
Resp:
a) S = {-1, 4}
b) S = {-1/2, 1}
c) S = {-3, 3}
3 - UCSal/BA - O maior valor assumido pela função y = 2 - x-2 é:
a) 1
*b) 2
c) 3
d) 4
e) 5
4 - UCSal/BA - O gráfico da função f de R em R, dada por f(x) = 1-x -
2, intercepta o eixo das abscissas nos pontos (a,b) e (c,d), com a c. Nestas
condições o valor de d + c - b - a é:
*a) 4
b) -4
c) 5
d) -5
e) 0

Módulo
II
1 – Definição

Das considerações da aula anterior, sabemos que o módulo de um número


real é sempre positivo ou nulo.

Exemplos:
-6 =6, 3 =3, 0 = 0 , etc.

Considere x = -10. Sabemos que x = -10 = 10.


Observe que sendo x = -10 um número negativo, o módulo é igual a 10, que é
exatamente o simétrico de –10, ou seja -10 = -(-10) = 10.

Fica fácil portanto, entender a definição genérica de módulo de um número


real apresentada a seguir:

Dado um número real x , define-se:


x = x para x 0
x = -x para x 0
Exemplos:

a) Seja y = x-3

Para x = 3, temos x – 3 = 0 e portanto y =0


Para x 3, temos x – 3 0 e portanto y = x –3
Para x 3, temos x – 3 0 e portanto y = - (x – 3) = -x + 3 = 3 – x

b) Seja y = 2-x

Para x = 2, temos 2 – x = 0 e portanto y =0


Para x 2, temos 2 – x 0 e portanto y = - (2 – x) = -2 + x = x – 2
Para x 2, temos 2 – x 0 e portanto y =2–x

c) Simplifique a expressão y = 2x - 6 + x- 3 , para o caso


particular de x 3.
SOLUÇÃO:

Ora, se x 3 então 2x – 6 0 e, portanto 2x - 6 = - (2x – 6) = 6 – 2x


Analogamente, se x 3 então x – 3 0 e, portanto x-3 = - (x – 3) =
3-x
Portanto, teremos finalmente:
y = 6 – 2x + 3 – x = 9 – 3x
Ou seja, y = 9 – 3x para x 3.

Faça agora o mesmo problema para o caso de x 3.


Resposta: y = 3x – 9
2 – Outra definição importante para o módulo de um número real x é:

Exemplo: Resolva a equação a seguir:

SOLUÇÃO:

Pela definição vem: 2x-6 = 12 2x-6=12 ou 2x-6= -12


Portanto, x = 9 ou x = -3.
Agora tente resolver as duas questões a seguir:
1 - PUC/SP – O número de soluções da equação x -1 = 1, no
universo R é:
a) 0
b) 1
c) 2
d) 3
e) 4
2
2 - VUNESP/SP – As raízes da equação x + x - 6 = 0:
a) são positivas
b) tem soma igual a zero
c) tem soma igual a um
d) tem produto igual a seis
e) tem produto igual a menos seis
Resp: 1D ; 2B

Módulo III -
Inequações Modulares
1 - Introdução
Já sabemos que o módulo de um número real é um número positivo ou nulo, o
que nos leva a interpretar que o módulo de um número real está diretamente
associado à noção de distância.
Assim, dado um número real x, o módulo de x - representado por |x| - é igual
geometricamente, à distancia à origem O, do ponto P, representativo do
número x na reta real - também conhecida como reta numerada.
Consideremos por exemplo a reta numerada a seguir:

Teremos: |x| = distancia de P a O = dP, O


Vamos utilizar a interpretação geométrica do módulo vista acima, para
resolver as desigualdades ou inequações modulares.
Inequações do tipo |y| b, com b 0.
Resolver a inequação acima, significa determinar quais os números reais cuja
distancia à origem O, são menores ou iguais ao número positivo b.
Da figura abaixo, infere-se imediatamente que os números procurados estão
situados no intervalo fechado [-b, b].

Assim, podemos escrever a seguinte regra geral:


|y| b -b y b, para b real e positivo.
Inequações do tipo |y| b, com b 0.
Resolver a inequação acima, significa determinar quais os números reais cuja
distancia à origem O, são maiores ou iguais ao número positivo b.
Utilizando o mesmo raciocínio anterior, concluímos imediatamente que os
números procurados estão situados nos intervalos (- , - b] ou [b, + ).
Assim, podemos escrever a seguinte regra geral:
|y| b y b ou y - b, para b real e positivo.
Veja a figura abaixo, a qual lhe ajudará no entendimento da importante
propriedade vista acima.

2 - Exercícios resolvidos
Resolva em R - conjunto dos números reais - as seguintes inequações
modulares:
|2x + 5| 11
SOLUÇÃO:
Vem imediatamente que: -11 2x + 5 11
Somando -5 a todos os membros, fica: -16 2x 6
Daí, então, dividindo tudo por 2, concluímos finalmente: -8 x 3.
Logo, o conjunto solução da inequação dada será o conjunto S dado por:
S = {x R; -8 x 3} , que, representado na forma de um intervalo real,
seria indicado por S = [-8, 3].
Graficamente, teríamos:

Observe que no conjunto R dos números reais, o conjunto solução da


inequação dada é um conjunto infinito formado por todos os números reais a
partir de - 8 até +3.
Se, por exemplo, fosse pedido o conjunto solução da mesma inequação no
conjunto Z dos números inteiros, o conjunto solução seria FINITO, e igual a:
S = {-8, -7, -6, -5, -4, -3, -2, -1, 0, 1, 2, 3}.
Se, por exemplo, fosse pedido o conjunto solução da mesma inequação no
conjunto N dos números naturais, o conjunto solução seria FINITO, e igual a:
S = {0, 1, 2, 3}.
É muito importante estar atento ao conjunto universo adotado na
questão proposta.
Caso não seja feita nenhuma referencia, deveremos considerar que o
conjunto universo adotado é sempre R - conjunto dos números reais.

|x - 1| 5
SOLUÇÃO:
Teremos: x - 1 5 OU x - 1 -5
Portanto, x 6 OU x - 4.
O conjunto solução em R, será então: S = {x R; x 6 ou x - 4} .
Na forma de intervalo, teremos: S = (- , -4) (6, ).
Graficamente, teríamos:

Observe que, utilizando o conceito de diferença de conjuntos, poderemos


exprimir o conjunto solução S também na forma:
S = R - [-4, 6], onde R é o conjunto dos números reais.

|x + 3| + |2x - 8| 20
SOLUÇÃO:
Como não podemos somar diretamente os módulos, vamos considerar o que
segue:
Observe que as expressões entre módulo, se anulam para x = -3 e x = 4.
Temos tres casos a considerar:
1º caso: x -3
Neste caso, teremos:
x+3 0 |x + 3| = - (x + 3)
2x - 8 0 |2x - 8| = - (2x - 8)
Assim, a inequação dada será equivalente a:
- (x + 3) - (2x - 8) ³ 20 ou, eliminando os parênteses:
- x - 3 - 2x + 8 ³ 20
Portanto, teremos que determinar os valores de x que satisfaçam à dupla
desigualdade:
x - 3 e - x - 3 - 2x + 8 20.
Resolvendo este sistema de inequações, vem:
x -3ex - 5, que é equivalente a x - 5.
Então, a primeira parte da solução do problema é o conjunto
S1 = {x R; x - 5} = ( - , -5].
2º caso: - 3 x 4
Neste caso, teremos:
x+3 0 |x + 3| = x + 3
2x - 8 0 |2x - 8| = - (2x - 8)
Assim, a inequação dada será equivalente a:
x + 3 - (2x - 8) 20
Portanto, teremos que determinar os valores de x que satisfaçam à dupla
desigualdade:
-3 x 4 e x + 3 - (2x - 8) 20
Resolvendo este sistema de inequações, vem:
-3 x 4ex -9
Percebemos da figura abaixo, que a interseção é vazia, portanto, S2 = .
3º caso: x 4
Neste caso teremos:
x+3 0 |x + 3| = x + 3
2x - 8 0 |2x - 8| = 2x - 8
Assim, a inequação dada será equivalente a:
x + 3 + 2x - 8 20
Portanto, teremos que determinar os valores de x que satisfaçam à dupla
desigualdade:
x 4 e x + 3 + 2x - 8 20
Resolvendo este sistema de inequações, vem:
x 4ex 25/3, que é equivalente a x 25/3, conforme podemos
observar na figura abaixo.

Portanto, a terceira solução parcial será: S3 = {x R; x 25/3} = [25/3, )


A solução geral da inequação dada será então:
S = S1 S2 S3 = ( - , -5] [25/3, )=(- , -5] [25/3, )
S=(- , -5] [25/3, ).

SOLUÇÃO:
Observando que x2 - 10x + 25 = (x - 5)2 e lembrando que a2 = |a|, vem:
Dificuldade? Procure revisar módulo.

Portanto, |x - 5| 1 -1 x-5 1 -1 + 5 x-5+5


1+5 4 x 6.
Logo, o conjunto solução da inequação dada, será o intervalo real:
S = [4, 6].
|3x - 9| 2x
SOLUÇÃO:
Observe que a expressão entre módulo, se anula para x = 3.
Teremos então dois casos a considerar:
1º caso: x 3 3x - 9 0 |3x - 9| = -(3x - 9) = - 3x + 9
Portanto, a inequação fica: -3x + 9 2x
Teremos então de resolver a dupla desigualdade:
x 3 e -3x + 9 2x
Vem, x 3e9 5x x 3 e 9 /5 x 9 /5 x 3
Portanto, para o primeiro caso, teremos o conjunto solução parcial
S1 = [9/5, 3)
2º caso: x 3 3x - 9 0 |3x - 9| = 3x - 9
Portanto, a inequação fica: 3x - 9 2x
Teremos então de resolver a dupla desigualdade:
x 3 e 3x - 9 2x
Vem, x 3e-9 -x 3 xe9 x 3 xex 9 3 x 9
Portanto, para o segundo caso, teremos o conjunto solução parcial
S2 = [3, 9]
A solução geral da inequação proposta será então:
S = S1 S2 = [9/5, 3) [3, 9] = [9/5, 9].
S = [9/5, 9].
Graficamente, teríamos na reta real:

Agora, resolva esta:


|x - 3| + |x| + |x + 3| 9
Resposta: S = {x R; -3 x 3} = [-3, 3].

Produtos
Notáveis
Vamos relembrar aqui, identidades especiais, conhecidas particularmente
como Produtos Notáveis.
1 – Quadrado da soma e da diferença
(a + b)2 = a2 + 2ab + b2
(a – b)2 = a2 – 2ab + b2

Das duas anteriores, poderemos concluir que também é válido que:


(a+b)2 + (a-b)2 = 2(a2+b2) ou escrevendo de uma forma conveniente:

2 – Diferença de quadrados
(a + b).(a – b) = a2 – b2
3 – Cubo de uma soma e de uma diferença
(a + b)3 = a3 + 3.a2.b + 3.a.b2 + b3

Para determinar o cubo da diferença, basta substituir na identidade acima, b


por -b, obtendo:
(a – b)3 = a3 – 3.a2.b + 3.a.b2 – b3
Uma forma mais conveniente de apresentar o cubo de soma, pode ser obtida
fatorando-se a expressão como segue:

(a + b)3 = a3 + 3.a.b(a+b) + b3

Ou:

(a + b)3 = a3 + b3 + 3ab(a + b)

Esta forma de apresentação, é bastante útil.


Exemplos:

1 – A soma de dois números é igual a 10 e a soma dos seus cubos é igual a


100. Qual o valor do produto desses números?
SOLUÇÃO:
Temos: a + b = 10 e a3 + b3 = 100. Substituindo diretamente na fórmula
anterior, fica:
103 = 100 + 3ab(10) de onde tiramos 1000 = 100 + 30.ab
Daí, vem: 900 = 30.ab, de onde concluímos finalmente que ab = 30, que é a
resposta solicitada.

Nota: os números a e b que satisfazem à condição do problema acima, não


são números reais e sim, números complexos. Você pode verificar isto,
resolvendo o sistema formado pelas igualdades a+b = 10 e ab = 30. Verifique
como exercício!
Alerto para o fato de que é muito trabalhoso. Mas, vá lá, faça! É um bom
treinamento sobre as operações com números complexos. Pelo menos, fica
caracterizada a importância de saber a fórmula acima. Sem ela, a solução
DESTE PROBLEMA SIMPLES, seria bastante penosa!
2 - Calcule o valor de F na expressão abaixo, para:
a = -700, b = - 33 , x = 23,48 e y = 9,14345.

SOLUÇÃO: Com a substituição direta dos valores dados, os cálculos seriam


tantos que seria inviável! Vamos desenvolver os produtos notáveis indicados:

Se você observar CUIDADOSAMENTE a expressão acima, verá que o


numerador e o denominador da fração são IGUAIS, e, portanto, F = 1,
INDEPENDENTE dos valores de a, b, x e y.
Portanto, a resposta é igual a 1, independente dos valores atribuídos às
variáveis a, b, x e y.
Resposta : 1

Exercícios de
Aritmética I
1 – Um reservatório é alimentado por duas torneiras A e B: a primeira possui
uma vazão de 38 litros por minuto e a segunda 47 litros por minuto. A saída
da água dá-se através de um orifício que deixa passar 21 litros por minuto.
Deixando abertas as duas torneiras e a saída da água, o reservatório se
enche em 680 minutos. Qual o volume do reservatório?
Solução:

É fácil perceber que a cada minuto:


a) entram 38 litros da torneira A
b) entram 47 litros da torneira B
c) saem 21 litros do reservatório.

Portanto: 38 + 47 – 21 = 64 litros/min, é o saldo líquido da água que abastece


o reservatório.
Ora, se em 1 minuto são preenchidos 64 litros do reservatório, nos 680
minutos, teremos:
680x64 = 43520 litros, que é o volume do reservatório.

2 – Um filho sai correndo e quando deu 200 passos o pai parte ao seu
encalço. Enquanto o pai dá 3 passos, o filho dá 11 passos, porém 2 passos do
pai valem 9 do filho. Quantos passos deverá dar o pai para alcançar o filho?
Solução:

Temos:
2 passos do pai = 9 passos do filho. Daí, é claro que:
1 passo do pai = 4,5 passos do filho
3 passos do pai = 3x4,5 = 13,5 passos do filho
Em cada 3 passos, o pai se aproxima 13,5 – 11 = 2,5 passos do filho.
Como a distancia entre eles é de 200 passos, o pai, para vencer a distancia,
deverá dar
200/2,5 = 80 "seqüências" de 3 passos. Como cada "seqüência" é constituída
de 3 passos, teremos finalmente: 80x3 = 240 passos, que é a resposta do
problema.
NOTA: resolvi este probleminha, quando cursava a 1ª série ginasial. Como o
tempo passa depressa! Achei em minhas anotações, e resolvi publicar aqui,
como uma lembrança no tempo!
3 - Um floricultor possui 100 rosas brancas e 60 rosas vermelhas e pretende
fazer o maior número possível de ramalhetes iguais entre si. Quantos serão
os ramalhetes e quantas rosas de cada cor deve ter cada um deles?
Solução:

O número máximo de ramalhetes nas condições indicadas, será igual ao


Máximo Divisor Comum - MDC dos números 100 e 60.
Vamos então, calcular o MDC(100,60):
Sendo D(n) o conjunto dos divisores positivos de n , vem:
D(100) = {1, 2, 4, 5, 10, 20, 50, 100}
D(60) = {1, 2, 4, 5, 12, 15, 20, 30, 60}
Portanto, o máximo divisor comum será: MDC(100,60) = 20
Logo, serão 20 ramalhetes.
Para calcular o número de rosas conforme a cor, em cada um dos 20
ramalhetes, basta efetuar:
100/20 = 5 rosas brancas e 60/20 = 3 rosas vermelhas.
Resp: 20 ramalhetes, contendo cada um, 5 rosas brancas e 3 rosas
vermelhas.
4 – Numa corrida de automóveis, o primeiro piloto dá a volta completa na pista
em 10 segundos, o segundo em 11 segundos e o terceiro em 12 segundos.
Mantendo-se o mesmo tempo, no final de quantos segundos os três pilotos
passarão juntos pela primeira vez pela linha de partida e quantas voltas terão
dado cada um nesse tempo?
Solução:

Basta calcular o mínimo múltiplo comum – MMC(10, 11, 12).


Sendo M(n) o conjunto dos múltiplos positivos de n, vem:
M(10) = {10, 20, 30, 40, 50, 60, ... , 660, ...}
M(11) = {11, 22, 33, 44, 55, 66, ... , 660, ...}
M(12) = {12, 24, 36, 48, 60, 72, ... , 660, ...}
Temos: MMC(10, 11, 12) = 660
Portanto, os 3 pilotos passarão pela primeira vez no ponto de partida, após
660 segundos
(ou 660/60 = 11 minutos).
Cada piloto terá dado então:
1º piloto: 660 / 10 = 66 voltas
2º piloto: 660 / 11 = 60 voltas
3º piloto: 660 / 12 = 55 voltas
NOTA: a determinação do MMC acima, também poderia ser feita pelo método
tradicional, ou seja:

Portanto MMC(10,12,11) = 2x2x3x5x11 = 22x3x5x11 = 660


5 – Converta a velocidade de 20 m/s em km/h.
Solução:

NOTA: 1 hora = 60 min = 60.60 = 3600 segundos 1h = 3600s e portanto,


1s = (1/3600)h.
Exercícios propostos
1 - Um gato persegue um rato; enquanto o rato dá 5 pulos, o gato dá 3, porém
1 pulo do gato equivale a 2 pulos do rato. O rato leva uma dianteira
equivalente a 50 pulos do gato. Quantos pulos o gato deverá dar para
alcançar o rato?
Resp: O gato deverá dar 300 pulos.
2 - Pretende-se dividir dois rolos de arame de 36 metros e 48 metros de
comprimento, em partes iguais e de maior tamanho possível. Qual deverá ser
o comprimento de cada uma destas partes?
Resp: 12 metros
3 - Três despertadores são ajustados da seguinte maneira: o primeiro para
despertar de 3 em 3 horas; o segundo de 2 em 2 horas e o terceiro de 5 em 5
horas. Depois da primeira vez em que os três relógios despertarem ao mesmo
tempo, após quantas horas isto voltará a ocorrer?
Resp: 30 horas
4 - Converta a velocidade v = 144 km/h em m/s.
Resp: 40 m/s

Exercícios de
Aritmética II
1 – Um carpinteiro deve cortar três tábuas de madeira com 2,40m; 2,70m e 3m
respectivamente, em pedaços iguais e de maior comprimento possível. Qual deve ser
o comprimento de cada parte?

SOLUÇÃO:

Transformando as medidas em centímetros, vem: 240, 270 e 300 cm.


Agora, basta calcular o MDC (máximo divisor comum) entre estes números. Teremos,
então:
MDC(240,270,300) = 30.
Logo, o carpinteiro deverá cortar pedaços de madeira de 30 cm de comprimento.
2 – Sabe-se que o MDC (máximo divisor comum) de dois números é igual a 6 e o
MMC(mínimo múltiplo comum) desses mesmos números é igual a 60. Calcule o produto
desses números.

SOLUÇÃO:

Uma propriedade bastante conhecida é:


Dados dois números inteiros e positivos a e b , é válido que:
MMC(a,b) x MDC(a,b) = a x b
Daí, vem imediatamente que:
a x b = MMC(a,b) x MDC(a,b) = 6 x 60 = 360

3 – Dois cometas aparecem, um a cada 20 anos e outro a cada 30 anos. Se em 1920


tivessem ambos aparecido, pergunta-se quantas novas coincidências irão ocorrer até o
ano 2500?

SOLUÇÃO:

Trata-se de um clássico problema de MMC.


MMC(20,30) = 60. Logo:
A cada 60 anos haverá uma coincidência de aparições.
Portanto elas ocorrerão nos anos: (a partir de 1920)
1980, 2040, 2100, 2160, 2220, 2280, 2340, 2400, 2460, 2520, 2580, ...
Portanto, até o ano 2500, ocorrerão 09 (nove) aparições.

4 – Qual o número de divisores positivos de 320?

SOLUÇÃO:

Fatorando o número 320, vem:


320 = 26 x 51
Portanto, o número de divisores de 320 será igual a:
Nd = (6+1) x (1+1) = 7x2 = 14
Nota: o número de divisores positivos de am x bn é dado pelo produto (m + 1).(n + 1)

5 – Quantos divisores positivos o número 2000 possui?

SOLUÇÃO:

Fatorando o número 2000, vem:


2000 = 24 x 53
Portanto, o número de divisores positivos de 2000 será:
Nd = (4+1) x (3+1) = 5 x 4 = 20 divisores.
São eles: 1, 2, 4, 5, 8, 10, 16, 20, 25, 40, 50, 80, 100, 125, 200, 250, 400, 500, 1000
2000, ou seja, 20 divisores positivos.
Para saber mais, reveja Número de divisores positivos

Números
Congruentes
1 - Introdução

O conceito de números inteiros congruentes é devido a Gauss - Karl Friedrick


Gauss - físico, matemático e astrônomo alemão (17777 - 1855), um dos
estudiosos da Teoria dos Números.

A Teoria dos Números estuda as propriedades dos números inteiros, usando


métodos avançados. É uma disciplina obrigatória nos cursos regulares de
Matemática, na Universidade.

Entre outros precursores dos estudos da Teoria dos Números, podemos citar:

Diophantus - 210 d.C. /290 d.C. - matemático grego


Pierre de Fermat - 1601/1665 - matemático francês
Leonhard Euler - 1707/1783 - matemático suiço.
Adrien Marie Legendre - 1752/1833 - matemático francês

A Teoria dos Números, entretanto, tem os seus primórdios no mundo antigo.


Por exemplo, Erathostenes (276 a.C./194 a.C - matemático norte africano -
nascido em Cyrene, uma colônia grega do norte da África na época) e
Euclides (325 a.C./265 a.C. - matemático grego, autor da célebre obra "Os
elementos"), já tinham desenvolvido estudos sobre os números primos,
assunto que é objeto da atenção especial dos matemáticos, até os dias de
hoje.
2 - Definição

Sejam a e b dois números inteiros. Diremos que o número a é congruente ao


número b módulo m, onde m é um número inteiro não nulo, se e somente se,
a diferença a - b for divisível por m 0.

A congruência dos números a e b módulo m, será indicada pelo símbolo a


b (mod m).
Teremos, pela definição:
a b (mod m) a - b = k . m , onde k e m são números inteiros, com m não
nulo.
Podemos dizer que os números a e b são côngruos ou congruentes segundo
o módulo m, ou simplesmente congruentes módulo m.
Exemplos:

a) 10 2 (mod 4) porque 10 - 2 = 8, e 8 é divisível por 4.


b) 35 10 (mod 5) porque 35 - 10 = 25 e 25 é divisível por 5.
c) 12 2 (mod 5) porque 12 - 2 = 10 e 10 é divisível por 5.
3 - A congruência é uma relação de equivalência em Z

Vamos inicialmente, revisar o conceito de relação entre conjuntos.


Sejam A e B dois conjuntos quaisquer, não vazios. Chama-se Relação F de A
em B, a qualquer subconjunto do produto cartesiano AxB.

Assim é que, por exemplo, sendo A = {2, 3, 5} e B = {5, 7}, teremos


AxB = {(2,5), (2,7), (3,5), (3,7), (5,5), (5,7)}.

Podemos dizer como exemplo, que:


F1 = {(2,5), (2,7), (3,5)} é uma relação de A em B
F2 = {(2,7), (3,5), (3,7), (5,5)} é uma relação de A em B. F3 = {(3,5), (5,5)} é
uma relação de A em B.
Sendo B = A, podemos dizer que qualquer subconjunto de AxA, é uma
relação de A em A, ou simplesmente, uma relação em A.

Uma relação F de A em A, é dita uma relação de eqüivalência em A, se


satisfaz as seguintes propriedades:
Propriedade Reflexiva: Para todo a A, (a, a) F.
Propriedade Simétrica: Se (a,b) F então (b, a) F, para quaisquer a e b
A.
Propriedade Transitiva: Se (a,b) F e (b, c) F, então (a, c) F.
Exemplo:

A relação F = {(2,2), (3,5), (5,2), (3,2), (3,3), (5,5), (2,5), (5,3),(2,3)} é uma
relação de eqüivalência em A = {2, 3, 5}, pois as três propriedades acima são
satisfeitas, conforme você pode observar facilmente.
Vamos considerar agora uma relação R em Z - conjunto dos números inteiros
- assim definida:
(x, y) R x y (mod m), onde x y (mod m) significa
x é congruente a y módulo m, conforme vimos acima.
Podemos verificar que a relação R acima é uma relação de eqüivalência em
Z, pois são satisfeitas as três propriedades vistas anteriormente.
Vejamos:

1 - REFLEXIVA : para todo a Z, a a (mod m), pois a - a = 0 e 0 é


divisível por m, com m não nulo.
2 - SIMÉTRICA : se a b (mod m) então a - b = k.m . Ora se a - b é divisível
por m, então b - a também será divisível por m.
3 - TRANSITIVA : se a b (mod m) e b c (mod m) então a c (mod m) .
Com efeito,
a b (mod m) a - b = k.m
b c (mod m) b - c = k1.m
Somando membro a membro, vem: (a - b) + (b - c) = k.m + k1.m. Logo,
simplificando, fica:
a - c = (k + k1).m e, sendo k + k1 = k2 também um número inteiro, concluímos
pela transitividade da relação dada.
Podemos dizer então, que a relação de congruência em Z é uma relação
de equivalência.
4 - Uma aplicação prática da relação de congruência: os calendários

Vamos considerar, por exemplo, o calendário do mês de Janeiro do ano


2000:
D S T Q Q S S
1
2 3 4 5 6 7 8
9 10 11 12 13 14 15
16 17 18 19 20 21 22
23 24 25 26 27 28 29
30 31
Observe que em cada coluna, estão dispostos números que são congruentes,
segundo o módulo 7.

No Domingo, estão os números congruentes com 2, módulo 7.


Na Segunda, estão os números congruentes com 3, módulo 7.
Na Terça, estão os números congruentes com 4, módulo 7.
Na Quarta, estão os números congruentes com 5, módulo 7.
Na Quinta, estão os números congruentes com 6, módulo 7.
Na Sexta, estão os números congruentes com 7, módulo 7.
No Sábado, estão os números congruentes com 1, módulo 7
Por exemplo, em que dia da semana vai cair o dia 25/01/2000, sem
consultar o calendário acima?

Basta procurarmos um número congruente com 25, módulo 7.


Dividindo 25 por 7 dá 3 e resto 4.
Logo, 25 4(mod 7) e como 4 corresponde a uma Terça-feira, concluímos
que o dia 25/01/2000 cairá numa Terça-feira.
Exercício Resolvido
Resolva a seguinte equação de congruência em Z.
Obs.: Z = conjunto dos números inteiros.

5x 4 (mod 3)
SOLUÇÃO:

Teremos: 5x - 4 = 3.k onde k é um número inteiro.


5x = 3k + 4 x = (3k + 4) / 5, com a condição de 3k + 4 ser múltiplo de 5 e k
inteiro.
Logo, como os múltiplos de 5 são 0, 5, 10, 15, 20, ... , vem:
3k + 4 = 0 k = -4/3 (não serve pois k tem de ser inteiro).
3k + 4 = 5 k = 1/3 (não serve pois k tem de ser inteiro).
3k + 4 = -5 k = -3
3k + 4 = 10 k=2
3k + 4 = -10 k = -14/3 (não serve pois k tem de ser inteiro).
3k + 4 = 15 k = 11/3 (não serve pois k tem de ser inteiro).
3k + 4 = -15 k = -19/3 (não serve pois k tem de ser inteiro).
3k + 4 = 20 k = 16/3 (não serve pois k tem de ser inteiro).
3k + 4 = -20 k=-8
3k + 4 = 25 k=7
3k + 4 = -25 k = -29/3 (não serve pois k tem de ser inteiro).
3k + 4 = 30 k = 26/3 (não serve pois k tem de ser inteiro).
3k + 4 = -30 k = -34/3 (não serve pois k tem de ser inteiro).
3k + 4 = 35 k = 31/3 (não serve pois k tem de ser inteiro).
3k + 4 = -35 k = -13
.......................................
.......................................
Observe que os valores de k que satisfazem ao problema, são:
k = -3, 2, -8, 7, -13, ...

Podemos inferir que:


Valores positivos de k: 2, 7, 12, 17, ...
Valores negativos de k: -3, -8, -13, -18, ...
Então, as soluções da equação dada serão:
x = (3k + 4) / 5
k=2 x=2
k=7 x=5
k = 12 x=8
k = 17 x = 11
..........................
..........................
k = -3 x = -1
k = -8 x = -4
k = -13 x = -7
..........................
..........................
Os números que satisfazem à equação dada, escritos em ordem crescente,
são:
... , -7, -4, -1, 2, 5, 8, 11, ...
Portanto, o conjunto solução da equação dada é o conjunto
S = {..., -7, - 4, -1, 2, 5, 8, 11, ... }
Observe que S é um conjunto infinito.
Agora, resolva a seguinte equação de congruência em Z:
x 2(mod 4)

Resposta: S = {..., -6, -2, 2, 6, 10, 14, ...}

Proporcionalidade
entre grandezas
1 - O que é uma grandeza?
Entende-se por grandeza, como sendo qualquer entidade susceptível de ser
medida.
As grandezas classificam-se em dois tipos fundamentais:
Grandezas escalares - aquelas que ficam perfeitamente caracterizadas
apenas pelo conhecimento de um número que expresse a sua medida numa
determinada unidade.
Exemplos: massa, 20 kg ; volume, 12 m3 ; comprimento, 50 m ; tempo, 60 s,
etc.
Grandezas vetoriais - aquelas que para ficarem perfeitamente caracterizadas,
necessitam além de um número que expresse a sua medida numa
determinada unidade (o seu módulo), que sejam especificados o sentido e a
direção. São representadas através Vetores.

Exemplos: força, velocidade, aceleração, intensidade de campo elétrico, etc.


Nota: no que se segue, poderemos nos referir as grandezas vetoriais, sem
levar em conta o seu aspecto vetorial. Explico: ao nos referirmos a uma
velocidade (grandeza vetorial) de 80 km/h, por exemplo, não estaremos
interessados , na sua direção ou no seu sentido, e sim unicamente no seu
módulo, ou seja 80 km/h. O tratamento vetorial da velocidade, interessaria, se
estivéssemos dando uma abordagem do ponto de vista da Física. Para uma
abordagem de proporcionalidade, como nos propomos aqui, não
necessitamos de tal enfoque.
É conveniente ressaltar de passagem, que ao nos referirmos à velocidade, por
exemplo, estaremos nos referindo sempre à velocidade média, uma vez que a
velocidade instantânea de um móvel no tempo t = t0, teria que ser calculada
usando-se Derivadas.
2 - Proporcionalidade direta
Sejam G1 e G2, duas grandezas dependentes das variáveis X e W,
respectivamente, que assumem valores conforme tabela abaixo:

G1 X1 X2 X3 X4 ... Xn
G2 W1 W2 W3 W4 ... Wn

Dizemos que G1 e G2 estão em proporção direta quando,

Onde k é denominado constante de proporcionalidade.


Das igualdades acima, podemos inferir que genericamente, teremos X / W = k,
de onde vem,
X = k . W, sendo k a constante de proporcionalidade.
Dizemos então, que a variável X é diretamente proporcional à variável W,
segundo a constante k.
NOTA: se Y é diretamente proporcional a X, indicamos simbolicamente isto
por:
Y X.( = alfa , primeira letra do alfabeto grego).
Exemplo:
Considerando que um CD custa $0,80 é razoável supor que:

Quantidade 2 5 8 10 20 30 50 100
Preço total ( 1,60 4,00 6,40 8,00 16,00 24,00 40,00 80,00
$)

Observamos que as variáveis PREÇO e QUANTIDADE, são diretamente


proporcionais, pois:
1,60/2 = 4,00/5 = 6,40/8 = 8,00/10 = 16,00/20 = 24,00/30 = ... = 0,80, que, no
caso é a constante de proporcionalidade.
Podemos então concluir que a Quantidade Q e o Preço P, no exemplo acima,
estão relacionados pela sentença P = 0,80 . Q . Assim, conhecido Q,
determinaríamos o valor de P usando a fórmula anterior. Exemplo: 200 CDs
custariam $160,00.
Cabe aqui, entretanto, um comentário:
E se fossem 1.000.000.000.000 (um trilhão de CDs?). Pela fórmula,
chegaríamos a:
P = 1.000.000.000.000 x 0,80 = $800.000.000.000, ou seja 800 bilhões! De sã
consciência, você pagaria 800 bilhões por 1 trilhão de CDs?
Acho que não! Primeiro, porque 800 bilhões, são 800 bilhões e segundo,
porque eu acho que nem existe
1 trilhão de CDs no mundo!
Portanto, é conveniente lembrar que ao aplicarmos um modelo matemático
para traduzir um determinado problema, temos de estar atentos aos limites de
validade do modêlo. No exemplo acima, por exemplo, poderíamos considerar
que talvez 1000 CDs fosse o nosso limite (talvez um pouco mais), o que nos
levaria a interpretar o nosso modêlo, ou seja, a equação P = 0,80.Q com as
limitações
Q 1000 e P 800.
3 - Proporcionalidade inversa
Sejam G1 e G2 , duas grandezas dependentes das variáveis X e W,
respectivamente, que assumem valores conforme tabela abaixo:

G1 X1 X2 X3 X4 ... Xn
G2 W1 W2 W3 W4 ... Wn

Dizemos que G1 e G2 estão em proporção inversa quando,


X1.W1 = X2.W2 = X3.W3 = X4.W4 = ... = Xn.Wn = k = constante
Onde k é a constante de proporcionalidade.
Das igualdades acima, podemos inferir que genericamente, teremos X . W = k,
sendo k a constante de proporcionalidade.
Dizemos então, que as variáveis X e Y são inversamente proporcionais,
segundo a constante k.
Exemplo:
Considerando que um carro terá de percorrer a distancia de 240 km entre
duas cidades, é razoável supor que:

Velocidade (km/h) 20 40 50 80 100 125 200 240


Tempo de duração 12 6 4,8 3 2,4 1,92 1,2 1
(h)

Observamos que as variáveis VELOCIDADE e TEMPO, são inversamente


proporcionais, pois:
20.12 = 40.6 = 50.4,8 = 80.3 = 100.2,4 = 125.1,92 = 200.1,2 = 240.1 = k , onde
k no caso é a constante de proporcionalidade.
Podemos então concluir que, no exemplo acima, a VELOCIDADE V e o
TEMPO T, estão relacionados pela sentença V.T = 240. Assim, conhecido V,
determinaríamos o valor de T usando a fórmula anterior.
Aqui, também, vale a observação do item anterior. Por exemplo, se a
velocidade fosse 100.000 km/h, obteríamos um tempo igual a 0,0024h = 8,64
segundos! Ora, isto é um absurdo no mundo material!
Portanto, é conveniente relembrar que ao aplicarmos um modêlo matemático
para traduzir um determinado problema, temos de estar atentos aos limites de
validade do modêlo. No exemplo acima, por exemplo, poderíamos considerar
que talvez 200 km/h fosse o nosso limite (talvez um pouco mais), o que nos
levaria a interpretar o nosso modêlo, ou seja, a equação V.T = 240 com as
limitações
V 200 e T 1,2h.
Notas:
Se Y é diretamente proporcional a X, indicamos simbolicamente: Y X.
Se Y é inversamente proporcional a X, podemos dizer que Y é diretamente
proporcional a 1/X e indicamos :
Y (1 / X).
4 - Resolvendo problemas
14 trabalhadores, trabalhando 10 dias de 8 horas, conseguem
fazer 56000 metros de certo tecido. Quantos dias de 6 horas
serão necessários a 9 trabalhadores para fazerem 32400
metros do mesmo tecido?
Solução:
Sejam:
T = número de trabalhadores
D = número de dias
H = número de horas de trabalho por dia
L = comprimento de tecido
é plausível supor que:
D aumentando, T diminui, portanto D 1/T
D aumentando, H diminui, portanto D 1/H
D aumentando, L aumenta, portanto, D L
Assim, é que poderemos escrever:
D = k.(1 / T).(1 / H) . L = k.L / T.H, ou seja: D = k . L / T . H
Para determinar o valor da constante k, substituamos D, T, H e L pelos valores
conhecidos:
10 = k.56000 / 14.8 . Daí tiramos k = 10.14.8 / 56000 = 0,02
Portanto, a fórmula em vermelho acima, fica:
D = 0,02.L / T.H
Logo, usando os valores do enunciado, poderemos escrever:
D = 0,02. 32400 / 9.6 = 648 / 54 = 12
Portanto, serão necessários 12 dias.
Agora resolva este:
20 trabalhadores, em 10 dias de 8 horas, conseguem fazer
16000 metros de certo tecido. Quantos dias de 10 horas seriam
necessários para 10 trabalhadores, fazerem 32000 metros do
mesmo tecido?
Resposta: 32 dias
NOTA: observe que não usamos aqui o conceito de regra de três (rule of three
- em inglês) e nem falamos dela, e, entretanto, conseguimos resolver os
problemas.
Encontrei as seguintes preciosidades, na revista RPM n.º 9 / 2º
semestre 1986, publicação da Sociedade Brasileira de Matemática -
num artigo do Prof. Geraldo Ávila - pág. 8, as quais reproduzo
sintetizadas e adaptadas:
1. a regra de três, ao que tudo indica, surgiu na Índia e entrou na
Europa, através dos árabes.
2. ela foi muito usada no comércio, por vários séculos, porém
como simples regra.
3. Brahmagupta - matemático indiano - já no século VII da nossa
era - usava as técnicas da regra de três.
4. os livros modernos de Matemática, americanos, já não incorrem
no mesmo arcaísmo de abordagem, ainda presentes nos livros
brasileiros e, não usam nem mesmo, a expressão "rule of three"
(regra de três).
5. A propósito, recentemente o Prof. Ralph P. Boas, da
"Northwestern University" , celebrou este fato em versos
publicados na revista "American Mathematical Monthly" - 02/86
- pág. 115 - e que reproduzimos a seguir:
What has become of the rule of the three,
Simple or double, once popular pair?
Students today no longer see
Alligation, or tret and tare.
O que aconteceu com a regra de três,
Simples ou composta, outrora um par tão popular?
Os estudantes de hoje não mais reconhecem
"Alligation" ou "tret" e "tare" .

"Alligation" , "tret" e "tare" , em inglês, referem-se, de uma certa


forma, a um antigo processo de resolução de problemas, nos EUA.

Com estes versos deixamos aqui nossa sugestão de que o nome


"regra de tres" seja abolida também entre nós.

Prof. Geraldo Ávila


Uma forma prática de resolver problemas de
regra de três composta.
1 – Introdução
No capítulo sobre Proporcionalidade entre grandezas, demos um tratamento
técnico à questão. Aqui, entretanto, daremos um enfoque prático,
apresentando um método infalível, para resolver qualquer problema de regra
de três composta que possa aparecer na sua vida!.
Recomendo enfaticamente, que você revise o arquivo Proporcionalidade entre
grandezas, clicando no link acima.
A forma prática consiste em:

a) escrever em coluna as variáveis do mesmo tipo, ou seja, aquelas


expressas na mesma unidade de medida, tendo o cuidado de escrever o valor
desconhecido (x) sempre na segunda linha, conforme esquema mostrado no
item (c) abaixo.

b) Identificar aquelas que variam num mesmo sentido (grandezas diretamente


proporcionais) e aquelas que variam em sentidos opostos
(grandezas inversamente proporcionais), marcando-as com setas no mesmo
sentido ou sentidos opostos, conforme o caso.

c) A incógnita x será obtida da forma sugerida no esquema abaixo, dada


como exemplo de caráter geral.
Sejam as grandezas A, B, C e D, que assumem os valores indicados abaixo, e
supondo-se, por exemplo, que a grandeza A seja diretamente proporcional à
grandeza B, inversamente proporcional à grandeza C e inversamente
proporcional à grandeza D, podemos montar o esquema a seguir:

Neste caso, o valor da incógnita x será dado por:


Observem que para as grandezas que variam no mesmo sentido,
multiplicamos x pelos valores invertidos e para as grandezas que variam em
sentidos opostos, multiplicamos pelos valores como aparecem no esquema.
Exemplo:

STA CASA – SP – Sabe-se que 4 máquinas, operando 4 horas por dia,


durante 4 dias, produzem 4 toneladas de certo produto. Quantas toneladas do
mesmo produto seriam produzidas por 6 máquinas daquele tipo, operando 6
horas por dia, durante 6 dias?
a) 8 b) 15 c) 10,5 d) 13,5
Se você tentar usar a metodologia indicada no capítulo Proporcionalidade
entre grandezas , não obstante ser um método mais rigoroso e até mais
bonito, você perderia mais tempo na resolução.
Vejamos a solução:
Observe que a produção em toneladas é diretamente proporcional ao número
de máquinas, ao número de dias e ao número de horas/dia.
Portanto:

Portanto, seriam produzidas 13,5 toneladas do produto, sendo D a alternativa


correta.
2 – Exercícios resolvidos e propostos
2.1 – Vinte e cinco teares trabalhando oito horas por dia, durante 10 dias,
fizeram 1200 metros de certo tecido. Vinte teares trabalhando nove horas por
dia durante dezoito dias, produzirão quantos metros do mesmo tecido?

Nota: Tear – máquina destinada a tecer fios, transformando-os em pano ou


tecido. Plural: teares.
SOLUÇÃO:
Observe que o comprimento do tecido é diretamente proporcional ao número
de teares, ao número de dias e ao número de horas/dia.
Portanto:

Resp: 1944 m
2.2 – Em uma fábrica, vinte e cinco máquinas produzem 15000 peças de
automóvel em doze dias, trabalhando 10 horas por dia.
Quantas horas por dia, deverão trabalhar 30 máquinas, para produzirem
18000 peças em 15 dias?
Solução:

Observe que:
Aumentando o número de horas/dia, aumenta o número de peças, diminui o
número de dias necessários e diminui o número de máquinas necessárias.
Portanto:

Resp: 8 h
2.3 – Certo trabalho é executado por 15 máquinas iguais, em 12 dias de 10
horas. Havendo defeito em três das máquinas, quantos dias de 8 horas
deverão trabalhar as demais, para realizar o dobro do trabalho anterior?
Solução:
Aumentando o número de dias, diminui o número de horas/dia necessários e
diminui o número de máquinas necessárias.
Podemos também dizer que para realizar o dobro do trabalho, o número de
dias deve aumentar.
Portanto, podemos montar o seguinte esquema:

Logo,

Resp: 37,5 dias


Agora resolva estes dois:
1 - Em uma residência, no mês de fevereiro de um ano não bissexto, ficaram
acesas, em média, 16 lâmpadas elétricas durante 5 horas por dia e houve
uma despesa de R$ 14,00. Qual foi a despesa em março, quando 20
lâmpadas iguais às anteriores ficaram acesas durante 4 horas por dia,
supondo-se que a tarifa de energia não teve aumento?
Resp: R$15,50
2 - Um livro está impresso em 285 páginas de 34 linhas cada uma com 56
letras em cada linha. Quantas páginas seriam necessárias para reimprimir
esse livro com 38 linhas por página, cada uma com 60 letras?
Resp: 238 páginas

Cuidado com a
regra de três!

Um trabalhador recebeu a incumbência de fazer a capinação de um terreno


circular de 3 metros de raio, cobrando pelo trabalho o valor de $10,00.
Qual seria o preço justo a ser cobrado para capinar um terreno semelhante,
porém com 6 metros de raio?

Solução:

Vamos por partes:

1 – Capinação
Ação de capinar; retirar do solo, a planta gramínea conhecida como capim.

2 – Alguns mais desavisados, seriam compelidos a afirmar imediatamente e


equivocadamente, que deveria ser cobrado $20,00, uma vez que 6 metros é o
dobro de 3 metros. Ledo engano!.

3 – Observe que a área capinada pelo trabalhador é igual a


S= .r2, onde r é o raio do círculo capinado.
Sendo r = 3m, vem S = .32 = (9 ) m2.

NOTAS:
1 – a área S de um círculo de raio r é igual a S = r2.
2 - m2 = metro quadrado

Na capinação de uma área circular de 6 metros de raio, ele teria capinado


uma área S’ = .(r’)2 = .62 = (36. ) m2.

Formamos agora, a seguinte regra de três simples e direta:

ÁREA(m2) PREÇO ($)


9 10
36 x

Como o preço a ser cobrado, deve ser diretamente proporcional ao trabalho


realizado, vem imediatamente que:

x = 10.36 /9 = 10.4 = $40

Em resumo:

Se for cobrado $10 para capinar um terreno circular de 3 metros de raio,


então, o valor justo a ser cobrado para capinar um terreno circular de 6
metros de raio, deve ser igual a $40.

Muitas pessoas achariam $20 um valor correto, o que não é verdadeiro!

MDC e
MMC
1 - MDC - MÁXIMO DIVISOR COMUM
Definição: dados dois números inteiros a e b não nulos, define-se o máximo
divisor comum - MDC, como sendo o maior inteiro que divide
simultaneamente a e b.
O MDC de dois números será indicado por MDC (a, b).
Óbvio que se tivermos o MDC de n números inteiros a1, a2, a3, ... , an,
indicaremos por
MDC (a1, a2, a3, ... , an)
Exemplos:
1 - Determine o MDC dos inteiros 10 e 14.
Os divisores positivos de 10 são: 1, 2, 5, 10.
Os divisores positivos de 14 são: 1, 2, 7, 14.
Os divisores comuns, são, portanto: 1 e 2.
Portanto, o máximo divisor comum é igual a 2 e, indicamos: MDC(10,14) = 2.
2 - Determine MDC (4, 10, 14, 60)
Os divisores positivos de 4 são: 1, 2, 4
Os divisores positivos de 10 são: 1, 2, 5, 10
Os divisores positivos de 14 são: 1, 2, 7, 14
Os divisores positivos de 60 são: 1, 2, 3, 4, 5, 6, 10, 12, 15, 60
Os divisores comuns são, portanto: 1 e 2.
Portanto o MDC é igual a 2, ou seja: MDC (4, 10, 14, 60) = 2
Notas:
1.1 - um número inteiro positivo p 1 é denominado número primo, se e
somente se os seus divisores positivos são 1 e p. Pode-se provar que o
conjunto dos números primos é um conjunto infinito.

Sendo P o conjunto dos números primos, podemos escrever:


P = {2, 3, 5, 7, 11, 13, 17, 19, 23, 29, 31, 37, 41, 43, 47, 53, 61, ... }
Observa-se que 2 é o único número par que é primo.
1.2 -Todo número inteiro maior do que 1, que não é primo, pode ser
decomposto num produto único de fatores primos. Esta afirmação é conhecida
como o Teorema Fundamental da Aritmética - TFA.
Exemplos:

15 = 5.3
40 = 5.8 = 5.2.2.2 = 5.23
120 = 40.3 = 5.2.2.2.3 = 5.23.3
240 = 2.120 = 2.5.2.2.2.3 = 5.24.3
Na prática, podemos usar o seguinte esquema:
Seja o caso de 240 acima. Teremos:
240 |2
120 |2
60 |2
30 |2
15 |3
5 |5
1|
Então: 240 = 2.2.2.2.3.5 = 24.3.5
A decomposição de um número em fatores primos, é conhecida também
como fatoração, já que o número é decomposto em fatores de uma
multiplicação.
Usando o dispositivo prático acima, vamos fatorar o número 408.
Teremos:
408 |2
204 |2
102 |2
51 |3
17 |17
1|
Então: 408 = 2.2.2.3.17 = 23.3.17
1.3 - O método de decomposição de um número num produto de fatores
primos, sugere uma nova forma para o cálculo do MDC de dois números
inteiros não nulos, a e b, ou seja, para o cálculo de MDC (a, b).

Assim, seja calcular o MDC de 408 e 240.


Como já vimos acima, temos:
408 = 2.2.2.3.17 = 23.3.17
240 = 2.2.2.2.3.5 = 24.3.5
Tomando os fatores comuns elevados aos menores expoentes, teremos:
MDC (408, 240) = 23.3 = 8.3 = 24 , que é o MDC procurado.
Portanto, MDC (408, 240) = 24.
1.4 - o MDC do exemplo anterior, poderia ser também determinado pelo
método das divisões sucessivas, cujo dispositivo prático é mostrado a seguir:
1 1 2 3
408 240 168 72
24
| | | |

168
72| 24| 0
|

Para entender o dispositivo prático acima, basta observar que:


408:240 = 1 com resto 168
240:168 = 1 com resto 72
168:72 = 2 com resto 24
72:24 = 3 com resto zero.
Portanto o MDC procurado é igual a 24, conforme já tínhamos visto antes.
1.5 - se o MDC de dois números inteiros a e b for igual à unidade, ou seja,
MDC (a, b) = 1, dizemos que a e b são primos entre si, ou que a e b são co-
primos.
Ou seja:
MDC (a, b) = 1 a e b são primos entre si (co-primos).
a e b são primos entre si (co-primos).
Exemplo: MDC (7, 5) = 1 5 e 7 são primos entre si.
2 - MMC - MÍNIMO MÚLTIPLO COMUM
Definição: dados dois números inteiros a e b não nulos, define-se o mínimo
múltiplo comum - MMC, indicado por MMC (a, b) , como sendo o menor inteiro
positivo, múltiplo comum de a e b.
Exemplo:

Determine o MMC dos inteiros 10 e 14.


Os múltiplo positivos de 10 são: 10, 20, 30, 40, 50, 60, 70, 80, 90, 100, 110, ...
Os múltiplos positivos de 14 são: 14, 28, 42, 56, 70, 84, 98, 112, 126, 140, ...
Portanto, o mínimo múltiplo comum é igual a 70 e, indicamos: MMC(10,14) =
70.

Dos exemplos anteriores, vimos que: MDC (10,14) = 2 e MMC(10,14) = 70.


Observe que:
10.14 = 2.70 = 140 = MDC(10,14) . MMC (10,14)
Pode-se provar que, dados dois números inteiros positivos a e b, teremos
sempre que o produto desses números é igual ao produto do MDC pelo MMC
desses números, ou seja:
MDC (a,b) . MMC (a,b) = a . b
Observe que se dois números inteiros positivos a e b são primos entre si
(co-primos), o MDC entre eles é igual a 1, ou seja MDC (a, b) = 1 e, portanto,
teremos:
1.MMC(a,b) = a . b MMC (a, b) = a . b , ou seja:
O Mínimo Múltiplo Comum de dois números primos entre si é
igual ao produto deles.
Exemplos:

MMC(3, 5) = 3.5 = 15
MMC(7, 5, 3) = 7.5.3 = 105
Dois exercícios simples:
1 - O máximo divisor de dois números é igual a 10 e o mínimo múltiplo comum
deles é igual a 210. Se um deles é igual a 70, qual o outro?
Solução:

Ora, pelo que vimos acima, 10.210 = 70.n n = 30.


2 - Encontre um par ordenado (m, n) de números inteiros, que verifique a
relação
MDC(180, 1200) = 180m + 1200n.

Solução:

Inicialmente, vamos determinar o MDC entre 180 e 1200:


Os divisores positivos de 180 são:
1, 2, 3, 6, 9, 10, 12, 15, 18, 20, 30, 60, 90 , 180.
Os divisores positivos de 1200 são:
1, 2, 3, 4, 6, 8, 10, 12, 15, 20, 25, 30, 40, 50, 60, 80, 100, 120, 150, 200, 300,
400, 600, 1200.
Portanto, o máximo divisor comum - MDC - de 180 e 1200 é igual a 60, ou
seja:
MDC(180, 1200) = 60
Nota: poderíamos, é claro, determinar o MDC por qualquer um dos métodos
indicados neste texto.
Observe agora, que:

1200 = 180.7 - 60
1200 - 180.7 = - 60
Multiplicando ambos os membros por ( - 1), fica:
- 1200 + 180. 7 = 60
180.7 - 1200 = 60

180.7 + 1200( - 1) = 60
Comparando com os dados do enunciado da questão, teremos:
MDC (180, 1200) = 180m + 1200n = 60
Logo, vem imediatamente que m = 7 e n = -1, e portanto, o par ordenado (7,
-1) é uma solução inteira da equação 180m + 1200n = 60.
.Agora resolva este:
Se MDC (210, 1225) = 210a + 1225b, pede-se determinar um
par (a,b) de números inteiros, que satisfaça a igualdade acima.
Resp: a = 6 e b = - 1.
Veja mais exercícios de MDC e MMC , visitando os arquivos Exercícios de
Aritmética I e Exercícios de Aritmética II.

Sistema de
numeração binário

1 – O Sistema de numeração decimal

Já conhecemos o sistema de numeração decimal ou de base 10, que utiliza


os 10 algarismos 0, 1, 2, 3, 4, 5, 6, 7, 8 e 9 para representação dos números
reais.

Um aspecto muito importante da representação de um número, ou seja, do


seu numeral, é o
VALOR POSICIONAL dos algarismos que o compõe. Assim, por exemplo, no
número 234 – duzentos e trinta e quatro – o algarismo 2 possui valor
posicional 200, o algarismo 3 possui valor posicional 30 e o algarismo 4,
possui valor posicional 4.
Podemos escrever:
234 = 200 + 30 + 4
234 = 2.100 + 3.10 + 4.1
234 = 2.102 + 3.101 + 4.100

Analogamente, poderemos citar outros exemplos:

6542 = 6000 + 500 + 40 + 2


6542 = 6.1000 + 5.100 + 4.10 + 2
6542 = 6.103 + 5.102 + 4.101 + 2.100

508 = 500 + 0 + 8
508 = 5.100 + 0.10 + 8
508 = 5.102 + 0.101 + 8.100

Nota: cabe aqui distinguir número, numeral e algarismo.

NÚMERO = exprime a idéia de quantidade.


NUMERAL = símbolo utilizado para representar o número.
ALGARISMO = numerais de 0 a 9, no sistema de numeração decimal – base
10, letras I,V,X,L,C,D e M no sistema de algarismos romanos, etc.

Exemplo: O número trinta é representado no sistema de numeração decimal


pelo numeral 30, no qual foram utilizados os algarismos 3 e 0.

Voltando à questão do sistema de numeração decimal, um número de


numeral (abcd...j) composto por n algarismos
a, b, c, ..., j pode ser representado genericamente por:

(abcd...j) = a.10n-1 + b.10n-2 + c.10n-3 + ... + j.100


onde (abcd...j) possui n algarismos.

Exemplos:

A) Seja o número duzentos e cinqüenta e oito, cujo numeral no sistema


decimal é 258. Poderemos escrever:
258 = 2.100 + 5.10 + 8.1 = 2.102 + 5.101 + 8.100
B) Seja o número vinte e cinco mil e duzentos, cujo numeral é 25200.
Poderemos escrever:
25200 = 2.104 + 5.103 + 2.102 + 0.101 + 0.100

C) Seja o número treze milhões duzentos e quarenta e três mil trezentos e


vinte e cinco, cujo numeral no sistema decimal é 13 243 325. Poderemos
escrever:
13243325 = 1.107 + 3.106 + 2.105 + 4.104 + 3.103 + 3.102 + 2.101 + 5.100
2 – O Sistema de numeração binário

Analogamente ao sistema de numeração decimal, que usa os dez algarismos


0,1,2,3,4,5,6,7,8 e 9 para representar os números na base 10, podemos
considerar o sistema de numeração binário, que utiliza os dois algarismos 0 e
1 para representar os números na base 2.

Nota: De uma forma genérica, um sistema de numeração de base b,


com b maior ou igual a 2, será aquele sistema que usará os algarismos
0,1,2,3, ..., b – 1.

Exemplos:

Sistema de base 10: são usados os algarismos 0,1,2,3,...,9.

Sistema de base 8: são usados os algarismos 0,1,2,3,..7. Este sistema


também é conhecido como sistema octal.

Sistema de base 2: são usados os algarismos 0 e 1.


Este sistema também é conhecido como sistema binário.

Sistema de base 16: são usados os algarismos 0,1,2,3,4,5,6,7,8,9 e os


símbolos A,b,C,d,E,F para representar os numerais 10,11,12,13,14 e 15,
respectivamente.

Como este sistema utiliza as letras A,b,C,d,E,F para representar os numerais


10,11,12,13,14 e 15 respectivamente, alguns numerais escritos na base 16
podem possuir somente letras.
Exemplos:
O numeral 64218 escrito na base 16 fica: FAdA
O numeral 12237514 escrito na base 16 fica: bAbACA
O numeral 186 escrito na base 16 fica: bA
Convido o leitor a comprovar os exemplos acima, após a leitura integral deste
texto.
Este sistema também é conhecido como sistema hexadecimal.

Retornando à questão do sistema binário – objetivo deste arquivo - e, sem


maiores delongas, usando uma analogia com o sistema de base 10,
poderemos escrever o numeral de um número de base 2 na base 10, da
seguinte forma:
Dado o número de numeral (abcd...j) composto por n algarismos (todos iguais
a 0 ou 1), escrito na base 2, ou seja, utilizando apenas os algarismos ou
dígitos 0 e 1, poderemos convertê-lo para a base 10, da seguinte forma:

(abcd...j)(2 = (a.2n-1 + b.2n-2 + ... + j.20) na base 10.


Nota: (abcd...j)(2 representa um numeral escrito na base 2, onde a,b,c,d, ...,j
são iguais a 0 ou 1.

Exemplos:

101(2 = 101 na base 2 = 1.22 + 0.21 + 1.20 = 5 (5 escrito na base 10).

111(2 = 111 na base 2 = 1.22 + 1.21 + 1.20 = 7 (7 escrito na base 10).

1010(2 = 1.23 + 0.22 + 1.21 + 0.20 = 10 (10 escrito na base 10).

10001(2 = 1.24 + 0.23 + 0.22 + 0.21 + 1.20 = 17 (17 escrito na base 10).

E para converter um número dado na base 10 para a base 2?

Bem, neste caso, deveremos usar o seguinte algoritmo:

Dividir sucessivamente o número por 2, ...


Para justificar este algoritmo, vamos inicialmente, considerar a base 10.

Seja o numeral 248.


Dividindo 248 sucessivamente por 10, vem, usando a igualdade
Dividendo = Quociente x Divisor + Resto:

248 = 24 x 10 + 8
24 = 2 x 10 + 4
2 = 0 x 10 + 2
Observe os algarismos de baixo para cima: 248

Seja o numeral 1346.


Dividindo sucessivamente por 10, vem, usando a igualdade
Dividendo = Quociente x Divisor + Resto:
1346 = 134 x 10 + 6
134 = 13 x 10 + 4
13 = 1 x 10 + 3
1 = 0 x 10 + 1
Observe os algarismos de baixo para cima: 1346

Analogamente, teríamos para escrever um numeral na base 2, ou seja,


utilizando-se somente os algarismos 0 e 1:

Exemplo I) Seja o numeral 7 na base 10.

Dividindo sucessivamente por 2, vem, usando a igualdade


Dividendo = Quociente x Divisor + Resto:
7=3x2+1
3=1x2+1
1=0x2+1
Observe os algarismos de baixo para cima: 111
Portanto, 7 na base 10 é representado por 111 na base 2.

Exemplo II) Seja o numeral 15 na base 10.

Dividindo sucessivamente por 2, vem, usando a igualdade


Dividendo = Quociente x Divisor + Resto:
15 = 7 x 2 + 1
7=3x2+1
3=1x2+1
1=0x2+1
Observe os algarismos de baixo para cima: 1111
Portanto, 15 na base 10 é representado por 1111 na base 2.

Exemplo III) Seja o numeral 12 na base 10:

Dividindo sucessivamente por 2, vem, usando a igualdade


Dividendo = Quociente x Divisor + Resto:
12 = 6 x 2 + 0
6=3x2+0
3=1x2+1
1=0x2+1
Observe os algarismos de baixo para cima: 1100
Portanto, 12 na base 10 é representado por 1100 na base 2.

Exemplo IV) Seja o numeral 260 na base 10:


Dividindo sucessivamente por 2, vem, usando a igualdade
Dividendo = Quociente x Divisor + Resto:
260 = 130 x 2 + 0
130 = 65 x 2 + 0
65 = 32 x 2 + 1
32 = 16 x 2 + 0
16 = 8x2+0
8= 4x2+0
4= 2x2+0
2= 1x2+0
1= 0x2+1

Observe os algarismos de baixo para cima: 100000100


Portanto, 260 na base 10 é representado por 1100000100 na base 2.

Exemplo V) Seja o numeral 35:


Dividindo sucessivamente por 2, vem, usando a igualdade
Dividendo = Quociente x Divisor + Resto:
35 = 17 x 2 + 1
17 = 8 x 2 + 1
8= 4x2+0
4= 2x2+0
2= 1x2+0
1= 0x2+1
Observe os algarismos de baixo para cima: 100011
Portanto, 35 na base 10 é representado por 100011 na base 2.

Agora, como exercício, converta os seguintes numerais escritos na base 10,


para a base 2:
a) 100
b) 26
c) 9
d) 225

Respostas:
a) 1100100
b) 11010
c) 1001
d) 11100001

Nota: os computadores digitais utilizam o sistema de numeração na base 2,


daí a sua importância estratégica.

Sistema de
numeração romano
1 – Introdução

O sistema de numeração romano existe há um pouco mais de 2000 anos


e utiliza os numerais I, V, X, C, D e M para representação dos números
inteiros positivos. Hoje, trata-se de um sistema praticamente em desuso,
valendo entretanto a sua abordagem, pelo seu valor histórico. O uso do
sistema romano atualmente, restringe-se tão somente a algumas aplicações
tais como a numeração de capítulos de livros, a ordem cronológica dos papas,
a ordem cronológica de reis e rainhas, mostradores de alguns tipos de
relógios, designação dos séculos etc.

Exemplos:
Século XXI, Papa Pio XII, Papa João Paulo II, Dom Pedro II, Papa Bento
XVI, etc.

Numeral Valor absoluto


I 1
V 5
X 10
L 50
C 100
D 500
M 1000

2 – Regras básicas para escrita dos numerais romanos

2.1 – somente as letras I, X, C e M podem ser repetidas e, no máximo, três


vezes consecutivas.

2.2 – as letras V, L e D não podem ser repetidas.

2.3 – letras repetidas adicionam-se.


Exemplos: III = 3, CCC = 300, MMM = 3000, CC = 200 etc.

2.4 – letra à esquerda de outra de maior valor absoluto, subtrai-se; à direita,


soma-se.
Exemplos:
XL = 40 (L = 50 – X = 10)
LX = 60 (L = 50 + X = 10)
IV = 4
VI = 6
MI = 1001
XIV = 14
XVI = 16

2.5 – letra encimada por um traço horizontal, vale 1000 vezes o seu valor
absoluto. A única exceção é o numeral I, para o qual não se aplica esta
regra.Cada traço adicional, multiplica o valor anterior por 1000.

Exemplos:

Alguns exemplos aleatórios de numerais hindu-arábicos expressos como


numerais romanos:

356 = CCCLVI
1254 = MCCLIV
78 = LXXVIII
2001 = MMI
1952 = MCMLII
3.674.690 = MMM DCLXXIV DCXC

A data oficial do descobrimento do Brasil seria: XXII.IV.MD


A data da proclamação da república no Brasil seria: XV.XI.MDCCCLXXXIX

NOTAS:

1 – não existe representação para o zero neste sistema de numerais romanos.


Aliás, o zero, tal como conhecemos hoje, foi introduzido pelos hindus por volta
do ano 500 depois de Cristo. Apenas para ajudar a situar-se no tempo, a
queda do Império Romano do Ocidente deu-se no ano 476 depois de Cristo.
2 – O sistema de numeração hindu-arábico (o sistema atual,que utiliza os
algarismos 0, 1, 2, 3, 4, 5, 6, 7, 8, e 9) somente começou a ter uso rotineiro na
Europa, por volta do ano 976 depois de Cristo.

3 – O sistema de numeração romano não usa o valor posicional dos


algarismos, o que de uma certa forma, limitou e tornou proibitivo o uso dos
mesmos nos cálculos.

Uma contagem na
sala de aula
Numa sala de aula com 60 alunos, 11 jogam xadrez, 31 são homens ou jogam
xadrez e 3 mulheres jogam xadrez. Calcule o número de homens que não
jogam xadrez, o número de homens que jogam xadrez e o número de
mulheres que não jogam xadrez.

SOLUÇÃO:
Sejam:
H = conjunto dos homens
M = conjunto das mulheres
J = conjunto daqueles que jogam xadrez
Da figura acima, poderemos escrever:
x + y + z + w = 60 ......................... (a)
y + z = 11 ................................. (b)
x + y + z = 31 ............................. (c)
z = 3 ...................................... (d)
Vamos resolver o sistema acima:
Substituindo (b) em (c), vem:
x + 11 = 31, logo, x = 20.
Como x + y + z = 31, vem 20 + y + 3 = 31, logo, y = 8.
Da igualdade (a), vem imediatamente que:
31 + w = 60 e, portanto w = 29.
Portanto,
x = 20
y=8
z=3
w = 29
Observe que x é exatamente o número dos homens que não jogam xadrez,
sendo portanto a resposta procurada.

Podemos resumir então:


x = 20 homens não jogam xadrez.
y = 8 homens jogam xadrez.
z = 3 mulheres jogam xadrez.
w = 29 mulheres não jogam xadrez.

Vôlei, xadrez
ou tênis?
Num grupo de 99 esportistas , 40 jogam vôlei , 20 jogam vôlei e xadrez ,
22 jogam xadrez e tênis , 18 jogam vôlei e tênis e 11 jogam as 3
modalidades .O número de pessoas que jogam xadrez é igual ao número
de pessoas que jogam tênis. Sabendo-se que cada esportista pratica
pelo menos um dos esportes, pergunta-se:
a) Quantos esportistas jogam tênis e não jogam vôlei ?
b) Quantos jogam xadrez ou tênis e não jogam vôlei ?
c) Quantos jogam vôlei e não jogam xadrez ?
d) quantos praticam apenas um esporte?
e) quantos praticam pelo menos dois esportes?
SOLUÇÃO:
Temos, pelo enunciado:
n(V) = 40
n(V X) = 20
n(X T) = 22
n(V T) = 18
n(V X T) = 11
n(X) = n(T)
Podemos escrever:
n(V U X U T) = n(V) + n(X) + n(T) - n(V X) - n(X T) - n(V T) + n(V
X T)
Substituindo os valores conhecidos, fica:
99 = 40 + 2.n(X) – 20 – 22 – 18 + 11
Daí, vem:
99 = 2.n(X) – 9
2.n(X) = 108 e, portanto, n(X) = n(T) = 54.
Podemos então desenhar o seguinte diagrama de Venn:

Onde temos por simples contagem, a partir do diagrama acima:


Quantos esportistas jogam tênis e não jogam vôlei ?
N = 25 + 11 = 36
Quantos jogam xadrez ou tênis e não jogam vôlei ?
M = 23 + 11 + 25 = 59
Quantos jogam vôlei e não jogam xadrez ?
S = 13 + 7 = 20
Quantos praticam apenas um esporte?
R = 13 + 23 + 25 = 61
Quantos praticam pelo menos dois esportes?
W = 7 + 11 + 11 = 29

Potências e
radicais - parte I
1 – Potência de expoente natural

Sendo a um número real e n um número natural maior ou igual a 2, definimos


a n-ésima (enésima) potência de a como sendo:
an = a.a.a.a.a. … .a

onde o fator a é repetido n vezes, ou seja, o produto possui n fatores.

Denominamos o fator a de base e n de expoente; an é a n-ésima potência


de a.

Portanto, potência é um produto de n fatores iguais.


A operação através da qual se obtém uma potência, é denominada
potenciação.
Exemplos:

72 = 7.7 = 49
25 = 2.2.2.2.2 = 32
63 = 6.6.6 = 216
107 = 10.10.10.10.10.10.10 = 10000000 (dez milhões)
106 = 10.10.10.10.10.10 = 1000000 (um milhão)

Nota:
Observe que a potência 10n é igual a 1 seguido de n zeros.
Assim, por exemplo, 1010 = 10000000000 (dez bilhões).

1.1 – Convenções:

a) potência de expoente zero


a0 = 1
Exemplos: 45670 = 1; 2430 = 1; (- 2001)0 = 1
b) potência de expoente unitário
a1 = a
Exemplos: 231 = 23; 20011 = 2001.

1.2 – As potências de expoente 2 e 3 recebem nomes especiais, a saber:


a2 = a.a, é lido como a ao quadrado.
a3 = a.a.a, é lido como a ao cubo.

1.3 – Propriedades das potências

São válidas as seguintes propriedades das potências de expoentes naturais,


facilmente demonstráveis:

P1) am . an = am+n
Exemplo: 25.23 = 25+3 = 28 = 2.2.2.2.2.2.2.2 = 256

P2) am : an = am-n
Exemplo: 57:54 = 57-4 = 53 = 5.5.5 = 125

P3) (am)n = am.n


Exemplo: (42)3 = 42.3 = 46 = 4.4.4.4.4.4 = 4096

P4) am.bm = (a.b)m


Exemplo: 23.43 = (2.4)3 = 83 = 8.8.8 = 512

P5) am:bm = (a:b)m


Exemplo: 124:34 = (12:3)4 = 44 = 4.4.4.4 = 256

P6) a-n = 1/an


Exemplo: 5-2 = 1/52 = 1/5.5 = 1/25

Esta propriedade decorre de P2, ou seja: a-n = a0/an = a0-n = a-n.

Nota: estas propriedades também são válidas para expoentes reais.

Exercício:

Calcule o valor da expressão a seguir:


A = {[(23.24 : 43)]5}-2

Desenvolvimento:

A = {[(27 : (22)3)]5}-2 = {[27 : 26]5}-2 = {[21]5}-2 = 2-10 = 1/210 = 1/1024


2 - Radicais
A forma mais genérica de um radical é:

onde c = coeficiente, n = índice e A = radicando.


O radical acima é lido como: c raiz n-ésima (enésima) de A.
Se n = 2, costuma-se não representar o número 2 e lê-se como c raiz
quadrada de A.
Se n = 3, lê-se o radical como c raiz cúbica de A.

Exemplos:

que é lido com 5 raiz cúbica de 25, onde 5 é o coeficiente, 3 é o índice


e 25, o radicando.

3 10 que é lido como 3 raiz quadrada de 10, onde 3 é o coeficiente, 2 (não


indicado, por convenção) é o índice e 10, o radicando.
2.1 - Potência de expoente fracionário

Exemplo:

A propriedade acima decorre de:


Seja x = am/n . Podemos escrever: xn = (am/n)n e, daí, xn = am de onde vem,
extraindo-se a raiz n-ésima de ambos os membros:
2.2 – Introduzindo o coeficiente num radical
Uma importante propriedade dos radicais é a seguinte:

Exemplo:

Portanto, para introduzir um coeficiente num radical, basta elevar este


coeficiente a um expoente igual ao seu índice.
Esta propriedade é bastante útil também, para a simplificação de radicais, pois
às vezes, a depender do tipo de problema que está sendo abordado, pode
tornar-se necessário percorrer o caminho inverso. Assim, por exemplo,

2.3 – Raiz de raiz


Outra propriedade muito importante dos radicais é a que segue:

Exemplo:

A operação com radicais é denominada RADICIAÇÃO e, esta operação é a


inversa da POTENCIAÇÃO.

Isto decorre de:


Exemplos:

Como 2 elevado a 4 é igual a 16, dizemos que 2 é uma raiz quarta de 16.
Como 3 elevado a 2 é igual a 9, dizemos que 3 é uma raiz quadrada de 9.
Como 5 elevado a 3 é igual a 125, dizemos que 5 é uma raiz cúbica de 125,
etc

Potências e
radicais - parte II
Racionalização de denominadores

1 – O que significa?

Como o próprio nome indica, racionalizar o denominador de uma fração,


significa escreve-la
de uma forma equivalente, mantendo o seu valor inicial, sem que ela
contenha, entretanto, um número irracional no denominador. É importante
ressaltar, que a racionalização do denominador de uma fração, não a torna
racional mas, apenas, elimina o termo irracional do seu denominador, o que
de uma certa forma facilita os cálculos necessários à análise de um
determinado problema.

Apenas de passagem, lembramos que os números irracionais são todas as


dízimas não periódicas ou as raízes não exatas de números reais.
Podemos citar como exemplos de números irracionais:

A = 1,01001000100001... (dízima não periódica)


B = 0,123567012432756284... (dízima não periódica)
C= = 3,14159... (dízima não periódica)
D= 2 = 1,414... (raiz não exata e também uma dízima não periódica)
E= 3 = 1,732... (raiz não exata e também uma dízima não periódica)
F = 2 3 (raiz não exata e também uma dízima não periódica) etc

Observe que números como 0,34343434... , 0,2222222..., 3,1717171717...


etc., por serem dízimas periódicas, não são números irracionais e sim,
números racionais.

É importante ressaltar uma questão importante:


Um número racional pode sempre ser escrito como uma fração da forma a/b
com a e b sendo números inteiros, com b diferente de zero e os números
irracionais não podem ser escritos na
forma de fração a/b.

Por exemplo, 0,333333... que é uma dízima periódica, pode ser escrito como
1/3 pois 1/3 = 0,3333... e, portanto, é um número racional.
Outros exemplos de dízimas periódicas ou seja, números racionais:
0,21212121... = 21/99
0,342342342... = 342/999
0,888888... = 8/9 etc.

Já o número 2, por exemplo, que é uma raiz não exata, nunca poderá ser
escrito como uma fração do tipo a/b com a e b sendo números inteiros, com b
diferente de zero.

Voltemos então à questão da racionalização de denominadores.

2 – Racionalização de denominadores

Como regra geral, para racionalizar o denominador de uma fração, basta


multiplicar o numerador e denominador desta, por um termo conveniente
denominado Fator Racionalizante.

Saber escolher o fator racionalizante corretamente caracteriza-se como a


parte mais importante da solução do problema. Vejamos alguns exemplos
elucidativos:
a)

Observe que o fator racionalizante no caso acima foi 2.


b)

c)
Agora racionalize a seguinte fração:

Resposta:

Agora resolva o seguinte desafio:

Racionalize a fração:

Três problemas
de média
Média Aritmética Simples - MAS: Dados n valores x1, x2, x3, ... , xn , chama-se
média aritmética destes n valores, ao valor M = (x1 + x2 + x3 + ... + xn) / n

Exemplo: qual a média aritmética de 5, 7, 6 e 8?


M = (5 + 7 + 6 + 8) / 4 = 6,5.

Média Aritmética Ponderada - MAP: Às vezes, é importante atribuir-se pesos


diferenciados a cada valor, para o cálculo da média e, neste caso, a média
recebe o nome de média ponderada.
Assim, dados n valores x1, x2, x3, ... , xn aos quais são atribuídos os pesos k1,
k2, k3, ... , kn respectivamente, a média ponderada destes n valores será dada
por:

Mp = (x1.k1 + x2.k2 + x3.k3 + ... xn.kn) / (k1 + k2 + k3 + ... + kn)


Exemplo: Se os valores 10, 8 e 6 possuem pesos 4, 3 e 2 respectivamente, a
média ponderada destes valores será igual a:
Mp = (10.4 + 8.3 + 6.2) / (4 + 3 + 2) = 76 / 9 = 8,44.

Nota: Fazendo k1 = k2 = k3 = ... = k n = 1 na fórmula acima, obteremos:


Mp = (x1.1 + x2.1 + x3.1 + ... + xn.1) / (1 + 1 + 1 + ... + 1 )
Ora, no denominador temos o peso 1 somado n vezes e, portanto, igual a n.
Assim, teremos:
Mp = (x1 + x2 + x3 + ... + xn) / n, que é a fórmula da média aritmética. Portanto,
a média aritmética simples é um caso particular da média aritmética
ponderada, onde os pesos dos valores x1, x2,... xn , são unitários, ou seja,
igual à unidade.

1 – FGV – SP) Em uma classe com 20 rapazes e 30 moças, foi realizada uma
prova; a média dos rapazes foi 8 e a das moças, 7.
A média da classe foi:
a) 7,5
b) 7,4
c) 7,6
d) 7,55
e) 7,45

Solução:

Sejam r1, r2, r3, ... , r20 as notas dos rapazes e m1, m2, m3, ... , m30 as notas das
moças.
Podemos escrever para as médias:

Rapazes: (r1 + r2 + r3 + ... + r20) / 20 = 8 r1 + r2 + r3 + ... + r20 = 20.8 = 160


Moças: (m1 + m2 + m3 + ... + m30) / 30 = 7 m1 + m2 + m3 + ... + m30 = 30.7 =
210

Nota: Lembre-se que se a / b = k então a = b.k, para b diferente de zero.

A média global M da classe será obtida, dividindo-se a soma de todas as


notas (rapazes mais moças), dividido pelo número total de pessoas (20 + 30 =
50).

Logo, teremos:
M = [(r1 + r2 + r3 + ... + r20) + (m1 + m2 + m3 + ... + m30)] / (20 + 30)
Substituindo os valores calculados acima, vem:
M = (160 + 210) / 50 = 370 / 50 = 37 / 5 = 7,4.
Portanto, (b) é a alternativa correta.

2 – U. F. Santa Maria – RS) A velocidade média em km/h é definida como o


quociente do espaço percorrido, em quilômetros, pelo tempo gasto para
percorre-lo, em horas. Um automóvel percorreu a distância entre duas
cidades, com velocidade média de 60 km/h e fez a viagem de regresso com
velocidade média de 40 km/h. Então, pode-se afirmar que a velocidade média
do percurso total, ida e volta, foi de:
a) 48 km/h
b) 50 km/h
c) 52 km/h
d) 60 km/h
e) 100 km/h

Solução:

Se um móvel percorrer a distância s num intervalo de tempo t a sua


velocidade média será dada por Vm = s / t.
Na ida, teremos: 60 = s / t1 onde t1 é o tempo gasto no percurso.
Na volta, teremos: 40 = s / t2 onde t2 é o tempo gasto no percurso.

Da primeira equação acima, tiramos: t1 = s / 60


Da segunda equação acima, tiramos: t2 = s / 40

Para o cálculo da velocidade média no percurso total de ida e volta, teremos


que dividir o percurso total pelo tempo total ou seja:

Percurso total = s + s = 2s (ida e volta).


Tempo total = t1 + t2 = (s / 60) + (s / 40) (tempo de ida mais o tempo de
volta).

Logo, a velocidade média Vm será dada por:

Vm = (2s) / [(s / 60) + (s / 40)]

Efetuando a soma do denominador, vem:


s / 60 + s / 40 = 2s / 120 + 3s / 120 = 5s / 120 = s / 24
Substituindo, vem:

Vm = (2s) / (s / 24) = 2s . 24 / s = 48.

Portanto, a velocidade média no percurso foi igual a 48 km/h e, portanto,


alternativa A.

3 – PUC – MG) No concurso para o Tribunal de Alçada, os candidatos fizeram


provas de Português, Conhecimentos Gerais e Direito, respectivamente com
pesos 2, 4 e 6. Sabendo-se que cada prova teve o valor de 100 pontos, o
candidato que obteve 68 em Português, 80 em Conhecimentos Gerais e 50
em Direito, teve média:
a) 53
b) 56
c) 63
d) 66
e) 72

Solução:

A solução é imediata:
Mp = (68.2 + 80.4 + 50.6) / (2 + 4 + 6) = (136 + 320 + 300) / 12 = 63
Portanto, a alternativa correta é a de letra C.

Um problema antigo
em Catanduva
Faculdade de Medicina de Catanduva – SP – 1971
Um móvel percorre uma trajetória retilínea AB, onde M é o
ponto médio, sempre no mesmo sentido e com movimento
uniforme em cada um dos trechos AM e MB. A velocidade
no trecho AM é 3,0 m/s e no trecho MB é 7,0 m/s. A
velocidade média entre os pontos A e B vale:
a)2,1 m/s
b)3,3 m/s
c)4,2 m/s
d)5,0 m/s
e)nenhuma das respostas anteriores
A resposta correta é a de letra (c). Vejamos o porquê.
Solução:
É conveniente relembrar os seguintes conceitos:
1 – Movimento uniforme – aquele que se dá com velocidade
escalar constante. Neste tipo de movimento, distâncias
iguais são percorridas em intervalos de tempo iguais.
2 – Ponto médio – o ponto M que divide o segmento de reta
AB em dois segmentos congruentes ou seja, de mesma
medida.
3 – Velocidade média – em qualquer caso, a velocidade
média de um corpo, será obtida dividindo-se a distancia total
percorrida, pelo tempo total gasto para percorre-la.
Teremos então:

Seja x a distância de A a M.
Naturalmente, a distância de M a B será também igual a x,
já que M é o ponto médio de AB.
A distância total percorrida será igual a x + x = 2x.

Sabemos que distância = d = velocidade . tempo = v. t


e, portanto,
tempo = distância / velocidade = d/v
Portanto, o tempo gasto no percurso AM de medida x será:
t1 = x/3
Analogamente, no percurso MB também de medida x será:
t2 = x/7

O tempo total será então igual a t1 + t2 = x/3 + x/7 = 7x/21 +


3x/21 = 10x/21

Conforme item (3) acima, a velocidade média vm será igual


a:

vm = distância total / tempo total = 2.x / [(x/3) + (x/7)]


vm = 2.x / (10.x/21) = 2x.(21/10x) = 42/10 = 4,2

Efetuando as operações indicadas, encontraremos vm = 4,2


m/s.
Faça as contas você também!

Um trem de três vagões


UEFS 2002.1 – Um trem de três vagões, com 30 lugares
cada, foi fretado para uma excursão.
A empresa exigiu de cada passageiro R$800,00 mais
R$20,00 por cada lugar não ocupado.
Nessas condições, o número de passageiros necessários
para que essa empresa tenha rentabilidade máxima é igual a

a) 60
b) 65
c) 80
d) 85
e) 90

Solução:

Seja x o número de lugares ocupados, o que significa neste


caso, o número de passageiros.
Nestas condições, restarão 90 – x lugares não ocupados.

Pelo enunciado da questão e indicando por y o custo total do


frete do trem, podemos escrever, considerando-se os x
lugares ocupados:

y = [800 + 20(90 – x)] . x = [800 + 1800 – 20 x] . x


y = (2600 – 20x) . x = 2600 x – 20 x2

Esta função quadrática possui um valor extremante que é


máximo e este máximo será alcançado na abscissa do
vértice, ou seja, como a = - 20 e b = 2600, fica:

xv = - b / 2 . a = - 2600 / 2. ( - 20 ) = - 2600 / ( - 40 ) = 65

Sobre o conjunto vazio


Um conjunto, na linguagem comum e intuitiva, é uma classe, coleção ou
agrupamento de objetos denominados elementos. Na realidade, conjunto é
um conceito primitivo da Matemática, sendo aceito, portanto, sem definição.
Assim, estabelecendo-se uma propriedade a ser obedecida pelos elementos x
de um determinado conjunto, poderemos formar tantos conjuntos quanto
queiramos, a exemplo de:

A = {x; x é vogal do alfabeto latino} = {a, e, i, o, u}


B = {x; x é um satélite natural da Terra} = {Lua}
C = {x; x é uma estrela que permite a existência de vida na Terra} = {Sol}
D = {x; x é o planeta no qual vivemos} = {Terra}
E = {x; x é um número inteiro maior do que 7} = {8, 9, 10, 11, ... , 205, 206, ... }

Se a propriedade que define os elementos x for contraditória, os elementos


não existirão e, diremos, por extensão, que temos um conjunto sem
elementos, o qual é denominado conjunto vazio, representado pelo símbolo
.

Notas:

1- é uma letra de origem escandinava, ou seja, da Escandinávia.


2 – Escandinávia – região situada ao norte da Europa, abrangendo a
Dinamarca, Noruega, Suécia, Finlândia e Islândia.

Assim, poderemos exemplificar:

= {x; x é diferente de x}
= {x; x é um osso de borboleta}
= {x; x é uma asa de elefante}
= {x; x é um mês de 32 dias}
= {x; x é um número racional e irracional}

É válido afirmar que o conjunto vazio é subconjunto de qualquer conjunto


dado, ou seja: A, A,
que se lê: qualquer que seja o conjunto A, o conjunto vazio está contido em A
(ou é subconjunto de A).
Isto pode ser demonstrado como segue:

Suponhamos que A, ou seja, que o conjunto vazio não seja subconjunto


de um dado conjunto A. Se isto fosse verdadeiro, então deveria existir pelo
menos um elemento no conjunto , que não pertenceria ao conjunto A. Ora,
isto é um absurdo, pois já sabemos que o conjunto vazio não possui
elementos. Então, a afirmativa A é FALSA, o que nos leva a concluir
que A é VERDADEIRA.

Algumas observações sobre o conjunto vazio:

1 – O cardinal de é igual a zero, ou seja, o número de elementos do


conjunto vazio é zero.

2 – O conjunto vazio é subconjunto de si próprio, ou seja, .

3 – Não confunda o conjunto vazio , com o conjunto unitário { }, cujo


único elemento é o conjunto vazio, ou seja: = { } é FALSO.
O correto seria escrever uma das seguintes condições, ambas verdadeiras:
{ } ou { }. A primeira afirmação é verdadeira, pois o conjunto
vazio é subconjunto de todo conjunto e a segunda é verdadeira, pois o
conjunto { } realmente possui o como elemento e, portanto, é correto
também afirmar que
pertence a { }, indicado simbolicamente por { }.

4 – Não confunda o conjunto vazio com o conjunto unitário {0}, cujo único
elemento é o zero.
Portanto, = {0} é FALSO. O correto seria escrever {0}.

Exercício resolvido:
UFBA 73 – O conjunto dos valores de n para os quais a inequação
nx2 – 2(n+1)x + n + 1 > 0 se verifica qualquer que seja x R, é:
A) {n R; n > -1}
B) {n R; n < 0}
C) {n R; -1 < n < 0}
D)
E) nenhuma das respostas anteriores

Solução: Uma condição para que a função quadrática do primeiro membro da


expressão dada seja sempre positiva é expressada na figura seguinte, onde
está representada uma função quadrática
do tipo y = ax2 + bx + c :

Deveremos então ter as seguintes condições, para satisfazer ao enunciado da


questão:
1) n > 0
2) <0
Se necessário, revise Funções.

Portanto, como = b2 – 4ac vem, substituindo:

[-2(n+1)]2 – 4.n.(n+1) < 0


Desenvolvendo o primeiro membro da expressão acima, fica:
4(n+1)2 – 4n(n+1) < 0
4(n2 + 2n + 1) – 4n2 – 4n < 0
4n2 + 8n + 4 – 4n2 – 4n < 0
Simplificando, vem:
4n + 4 < 0
4n < - 4
n<-1

Ora, deveremos ter n < -1 e, pela primeira condição, n deve ser maior do que
zero, ou seja: n > 0. Vemos, pois, que estas condições são contraditórias, ou
seja, um número não pode ao mesmo tempo, ser maior do que zero e menor
do que –1. Logo, concluímos que o conjunto das soluções para o valor de n
no problema proposto é um conjunto sem elementos e, portanto, é o conjunto
vazio . A alternativa correta é então a de letra D.
Exercício proposto

PUC – SP – Se A = eB={ } então é correto afirmar:


A) A B
B) A U B =
C) A = B
D) A – B = B
E) B A

Resposta: Pelo que foi visto anteriormente, concluímos facilmente que a única
alternativa verdadeira é a de letra A.

Subconjuntos de um
conjunto finito
Seja A = {a1, a2, a3, a4, ... , an}um conjunto finito com n elementos.Desejamos
determinar o número total de subconjuntos (Ns) que podem ser formados a
partir do conjunto A.
Ora, existem subconjuntos de A formados por um elemento, por dois
elementos, por três elementos, ... , por n elementos e, também por zero
elementos, que corresponde ao conjunto vazio.
Exemplos de subconjuntos possíveis de A:
{a1}, {a2}, ... , {an} – com um elemento.
{a5, a8}, {a1, a6}, etc. – com dois elementos ,etc.

Da Análise Combinatória poderemos escrever:


Número de subconjuntos com zero elementos: Cn,0
Número de subconjuntos com um elemento: Cn,1
Número de subconjuntos com dois elementos: Cn,2
Número de subconjuntos com três elementos: Cn,3
...............................................
...............................................
Número de subconjuntos com n elementos: Cn,n

Assim, o número total de subconjuntos de A, será dado por:


Ns = Cn,0 + Cn,1 + Cn,2 + Cn,3 + ... + Cn,n
Para o cálculo do número Ns acima, o uso da fórmula seria muito trabalhoso,
principalmente para valores elevados de n.

Vamos contornar esta dificuldade, utilizando o Princípio Fundamental da


Contagem, visto na Análise Combinatória :
Observe que para um determinado elemento ai A , onde i pode assumir
valores de 1 a n, existem duas possibilidades: ele poderá pertencer ou não
pertencer a um subconjunto qualquer de A, não existindo outra hipótese.
Assim, pelo Princípio Fundamental da Contagem – PFC o número total de
possibilidades (que neste caso é igual ao número total de subconjuntos de A)
será dado por:
Ns = 2.2.2.2.2.2. ... .2 = 2n (um produto com n fatores iguais).
Logo,
Ns = 2n
onde n é o número de elementos do conjunto A.

Da análise anterior, concluímos então que:

Cn,0 + Cn,1 + Cn,2 + Cn,3 + ... + Cn,n = 2n


Exercícios resolvidos:
1 – Quantos subconjuntos possui o conjunto A = {1,2,3,6,7}?
Solução: Temos n = 5 elementos Ns = 25 = 32 subconjuntos.

2 - Resolva a equação:
Cx,0 + Cx,1 + Cx,2 + ... + Cx, x = 128
Solução: Pelo que foi visto acima, poderemos escrever imediatamente: 2x =
128 = 27 x = 7.

Exercícios propostos:
1 – Quantos subconjuntos possui um conjunto com 10 elementos?
Resposta: 210 = 1024 subconjuntos.

2 – Calcule o valor da soma


C10,0 + C10,1 + C10,2 + ... + C10, 10.
Resposta: 1024

1 + 11 + 111 + 1111 + 11111 +


... + 11...11111
Determine a soma seguinte em função de n:

S = 1 + 11 + 111 + 1111 + 11111 + ... + 11... 11111, onde a última parcela é um


número formado por n algarismos iguais a 1.

Solução:

Observe que poderemos reescrever a igualdade como segue:

S = 1 + (10+1) + (100+10+1) + (1000+100+10+1) + ... + cn


onde cn é a última parcela 11... 11111, um número formado por n algarismos
iguais à unidade.

Observando atentamente o segundo membro da igualdade acima, veremos


que:
Primeiro termo = 1 = 100
Segundo termo = 10 + 1 = 101 + 1
Terceiro termo = 100 + 10 + 1 = 102 + 101 + 1
Quarto termo = 1000 + 100 + 10 + 1 = 103 + 102 + 10 + 1
e assim sucessivamente.

É razoável supor, baseado nas igualdades anteriores, que o termo de ordem n


(n-ésimo termo) cn será dado por:
cn = 10n-1 + 10n-2 + 10n-3 + ... + 10 + 1

A soma S poderá ser reescrita como:

S = 1 + (10+1) + (100+10+1) + (1000+100+10+1) + ... + 10n-1 + 10n-2 + 10n-3 + ... + 10 + 1


contendo n termos, ou seja, a soma dada no enunciado possui sempre n
parcelas.

Ou na forma equivalente:
S = 1 + (10+1) + (102+10+1) + (103+102+10+1) + ... + 10n-1 + 10n-2 + 10n-3 + ... + 10 + 1

Vamos escrever as parcelas acima na forma abaixo, de modo a ajudar na


análise:
1
10 + 1
102 + 10 + 1
103 + 102 + 10 + 1
104 + 103 + 102 + 10 + 1
105 + 104 + 103 + 102 + 10 + 1
106 + 105 + 104 + 103 + 102 + 10 + 1
.................................................................
......................................................................
10n-1 + 10n-2 + 10n-3 + ... + 10 + 1

Verifique que na soma acima:

O número 1 aparece em todos os termos e, portanto, aparece n vezes.


O número 10 aparece em (n –1) termos e, portanto aparece (n – 1) vezes.
O número 100 = 102 aparece (n – 2) vezes
O número 1000 = 103 aparece (n – 3) vezes e assim sucessivamente.

Portanto poderemos escrever:

S = 1.n + 10(n-1) + 102(n-2) + 103(n-3) + ... + 10n-1(n-(n-1))


S = 1.n + 10(n-1) + 102(n-2) + 103(n-3) + ... + 10n-1
S = n + 10(n - 1) + 102(n - 2) + 103(n - 3) + ... + 10n-1
Como S é uma soma de valor positivo, o máximo valor que n poderá
assumir na igualdade acima será igual ao número de parcelas
consideradas. Esta fórmula é a solução do problema proposto, pois expressa
a soma em função de n.

Vejamos alguns exemplos:

n=1 S1 = 1
n=2 S2 = 2 + 10(2-1) = 12 = 1 + 11
n=3 S3 = 3 + 10(3-1) + 102(3-2) = 3 + 20 + 100 = 123 = 1 + 11 + 111
n=4 S4 = 4 + 10(4-1) + 102(4-2) + 103(4-3) = 1234 = 1 + 11 + 111 + 1111
n=5 S5 = 5 + 10(5-1) + 102(5-2) + 103(5-3) + 104(5-4) = 12345 = 1+11+111+1111+11111
e assim sucessivamente.

Vamos formar uma tabela resumo contendo os 10 primeiros resultados da


soma proposta no problema:

n Soma Resultado
1 1 1
2 1 + 11 12
3 1 + 11 + 111 123
4 1 + 11 + 111 + 1111 1234
5 1 + 11 + 111 + 11111 12345
6 1 + 11 + 111 + 1111 + 11111 + 111111 123456
7 1 + 11 + 111 + 1111 + 11111 + 111111 + 1111111 1234567
8 1 + 11 + 111 + 1111 + 11111 + 111111 + 1111111 + 11111111 12345678
9 1 + 11 + 111 + 1111 + 11111 + 111111 + 1111111 + 11111111 + 123456789
111111111
10 1 + 11 + 111 + 1111 + 11111 + 111111 + 1111111 + 11111111 + 1234567900
111111111 + 1111111111

De uma forma geral, conforme já vimos, a soma de n parcelas será dada em


função de n pela expressão:

S = n + 10(n - 1) + 102(n - 2) + 103(n - 3) + ... + 10n-1


Exemplo:

para n = 11, a fórmula acima dará:

S11 = 11 + 10(11 – 1) + 102(11 – 2) + 103(11 – 3) + 104(11 – 4) + 105(11 – 5) +


+ 106(11 – 6) + 107(11 – 7) + 108(11 – 8) + 109(11 – 9) + 1010(11 – 10) =
= 12345679011
Realmente, o valor da soma
1+11+111+1111+11111+111111+1111111+11111111+111111111+1111111111++11
111111111 = 12345679011, obtido efetuando-se a conta pelo método usual,
senão vejamos:

1
11
111
1111
11111
111111
+ 1111111
11111111
111111111
1111111111
11111111111
12345679011

Veja outra solução deste problema

De um jeito mais fácil


Vamos calcular a soma das n parcelas
1 + 11 + 111 + 1111 + ... + 11... 1111, onde o último termo possui n algarismos -
visto no exercício anterior - de um jeito mais tranqüilo e muito mais fácil. O
método anterior, foi também uma viagem importante, porém, muito longa.
Entretanto, o caminho adotado na solução anterior, nos proporcionou reflexão
e conhecimento.
Sigamos em frente, então.
Já vimos que a soma
9 + 99 + 999 + 9999 + ... + 99...9999
é dada pela fórmula

Observando que 9 + 99 + 999 + ... + 99... 999 = 9 (1 + 11 + 111 + ... + 11...111)


e substituindo o primeiro membro pela fórmula acima, fica:

Daí, tirando o valor da soma 1 + 11 + 111 + ... + 11... 111 da igualdade acima,
resulta finalmente:

Uma conclusão interessante tirada da fórmula acima é que para todo n natural
maior ou igual a 1, o número 10n+1 – 9n – 10 é um número divisível por 81. Isto
é óbvio, já que a soma 1 + 11 + 111 + ... + 11... 111 resulta sempre num
número natural.

Agora resolva este:

Calcule a soma S = 3 + 33 + 333 + 3333 + ... + 33... 3333 que contém n


parcelas, onde o último termo possui n algarismos.
Resposta: S = (1/27)(10n+1 – 9n – 10)

Fatorial de 18
Quais os três últimos algarismos do fatorial de 18?
a) 1, 2 e 6
b) 1, 0 e 8
c) 0, 0 e 0
d) 3, 0 e 4
e) nenhuma das respostas anteriores.

Solução:

Já sabemos que o fatorial de um número natural n maior ou igual a 2 é dado


por:
n! = 1.2.3.4.5. ... . n
Exemplos:
2! = 1.2 = 2
3! = 1.2.3 = 6
4! = 1.2.3.4 = 24
5! = 1.2.3.4.5 = 120
6! = 1.2.3.4.5.6 = 720
7! = 1.2.3.4.5.6.7 = 5040
8! = 1.2.3.4.5.6.7.8 = 40320
e assim sucessivamente.

O problema pede para determinar os três últimos algarismos do fatorial de 18


ou seja, de
18! = 1.2.3.4.5.6.7.8.9.10.11.12.13.14.15.16.17.18
Poderíamos saber isto simplesmente efetuando o produto acima, mas isto não
seria nada criativo e também seria muito trabalhoso.

Observe que poderemos escrever o produto acima, na forma fatorada:


18! = 1.2.3.22.5.2.3.7.23. 32.2.5.11.22.3.13.2.7.3.5.24.17.32.2
onde substituímos os números compostos 4, 6, 8, 9, 10,12, 14, 15, 16 e 18 por
seus valores decompostos em fatores primos, a saber:
4 = 22
6 = 2.3
8 = 23
9 = 32
10 = 2.5
12 = 22.3
14 = 2.7
15 = 3.5
16 = 24
18 = 2.32
Observe que no produto acima os números 3, 5, 7, 11, 13 e 17 não podem ser
decompostos pois já são números primos.

Temos então:
18! = 1.2.3.22.5.2.3.7.23. 32.2.5.11.22.3.13.2.7.3.5.24.17.32.2
efetuando os produtos das potencias de mesma base, fica:
18! = 1.216.38.53.72.11.13.17
Poderemos escrever o produto acima na forma:
18! = 1.23.213.38.53.72.11.13.17
Ou ainda,
18! = 1.23. 53.213. 38.72.11.17 = (2.5)3.213.38.53.72.11.13.17
18! = 103. 213.38.53.72.11.13.17 = 1000. 213.38.53.72.11.13.17
Ora, o produto 213.38.53.72.11.13.17 é um número natural n.
Logo, 18! = 1000. n
Já sabemos que todo número natural multiplicado por 1000, termina em 000.
Logo, os últimos três algarismos de 18! serão zeros, o que nos leva
tranqüilamente à alternativa C.

Agora resolva este:

Quais os três algarismos finais de 15! ?


Resposta: 0, 0 e 0

Apenas como curiosidade, apresentamos abaixo, uma tabela contendo o valor


de alguns fatoriais, até 18.
2! = 2
3! = 6
4! = 24
5! = 120
6! = 720
7! = 5040
8! = 40320
9! = 362880
10! = 3 628 800
11! = 39 916 800
12! = 479 001 600
13! = 6 227 020 800
14! = 87 178 291 200
15! = 1 307 674 368 000
16! = 20 922 789 888 000
17! = 355 687 428 096 000
18! = 6 402 373 705 728 000

Somente a título ilustrativo, a população mundial em 2002, é da ordem de 6


bilhões de pessoas, ou seja, um número próximo a
13! = 6 227 020 800.

Calcule o valor de f(x)


Seja f: R R uma função tal que f(1/ 2) = 2 2 e f(x + y) = f(x) . f(y) x, y
R.
Calcule:
a) f(1)
b) f(5/3)
c) f(x)
d) f(-1/3)

Solução:

O símbolo significa qualquer que seja ou para todo .

Veja bem:
Os dados do problema são :

f(1/ 2) = 2 2
f(x + y) = f(x) . f(y) x, y R

Podemos escrever, com base na segunda condição acima:


f(0 + 1) = f(0 ). f(1) f(1) = f(0). f(1)
Ora, se f(1) = f(0).f(1) , então f(0) = f(1) / f(1) = 1, ou seja, f(0) = 1.

O símbolo significa de onde se deduz que .

Vejam que:
1/2 + 1/2 = 1
Daí, poderemos escrever com base no enunciado:

f(1/2 + 1/2) = f(1/2) . f(1/2)


Ora, pelo enunciado da questão, temos que f(1/2) = 2 2
Substituindo, vem:

f(1) = 2 2.2 2 = (2 2)2 = 4.2 = 8

Agora é tranqüilo:
f(2) = f(1 + 1) = f(1) . f(1) = 8.8 = 64 = 26
f(3) = f(2 + 1) = f(2). f(1) = 64. 8 = 512 = 29
f(4) = f(3 + 1) = f(3). f(1) = 512. 8 = 4096 = 212
f(5) = f(4 + 1) = f(4). f(1) = 4096. 8 = 211. 23 = 215
............................................................................

Vamos resumir os resultados obtidos acima?


f(0) = 20
f(1) = 23
f(2) = 26
f(3) = 29
f(4) = 212
f(5) = 215
..................
Podemos concluir observando os resultados acima que f(x) = 23x .
Assim, poderemos determinar o valor da função para qualquer valor de x.
Então:
f(5/3) = 23(5/3) = 25 = 32.
f(-1/3) = 23(-1/3) = 2-1 = 1/2.

Análise do preço de uma torta de maçã


UEFS 2002.2 – Uma torta de maçã de 25 cm de diâmetro é vendida por R$
30,00, e uma de 30 cm, por R$40,00. Sabe-se que esses preços são obtidos pela
equação P = k. d2+D , onde D são as despesas gerais, k uma constante real não
nula, e P, o preço de venda da torta. Se D não varia com o diâmetro da torta,
então o quociente D / k, em reais, é igual a :
A) 100
B) 200
C) 250
D) 300
E) 400

Solução :

Da simples leitura do enunciado, poderemos escrever:


P = k. d 2 + D
Sendo P = R$30,00, o diâmetro da torta mede 25 cm
Sendo P = R$40,00, o diâmetro da torta mede 30 cm

Substituindo, fica:

30 = k.252 + D
40 = k.302 + D

Subtraindo as igualdades, membro a membro, vem:

40 – 30 = k.302 + D – k.252 – D
10 = k.302 – k.252
10 = k.(302 – 252)
Lembrando que a2 – b2 = (a + b) (a – b), podemos simplificar o cálculo acima:

10 = k.(30 + 25)(30 – 25)


10 = k.55.5
10 = k.275
Daí tiramos k = 10 / 275

Para determinar o valor de D, voltemos a uma das igualdades acima:

40 = k.302 + D
Substituindo o valor encontrado para k, teremos:
40 = (10 / 275).900 + D , de onde concluímos que o valor de D será:
D = 40 – (10 / 275).900
D = 40 – 9000 / 275
D = (40.275 / 275) – 9000 / 275
D = 11000 / 275 – 9000 / 275
D = 2000 / 275

Logo, o quociente D / k procurado, será igual a:

D / k = (2000 / 275) / (10 / 275) = 2000 / 10 = 200

Portanto, a alternativa correta é a de letra B.

Agora resolva este:

Qual o preço a ser cobrado por uma torta de 1m de diâmetro, considerando-se os


dados do problema
anterior ?
Resposta : P = k. d 2 + D = (10 / 275).d2 + (2000 / 275)
Faça agora d = 1m = 100 cm e conclua que a torta custaria aproximadamente
R$390,91.

Agora uma pergunta: você pagaria quase R$400,00 por uma torta de um metro
de diâmetro? Provavelmente e usando o bom senso, não! O cálculo entretanto,
está absolutamente correto. O problema é que o modelo adotado (a equação P =
k. d2 + D para o cálculo do preço da torta) , deve funcionar para valores razoáveis
do diâmetro d da torta. Por exemplo, não obstante ser possível usar a fórmula P
= k. d2 + D para o cálculo do preço de uma torta de 10000 metros de diâmetro,
não seria razoável fazê-lo, pois isto iria contrariar o bom senso ! Uma torta de
10000 m de diâmetro não pode existir fisicamente !

De uma forma geral, ao se adotar um modelo matemático para a descrição de


um determinado problema, deve-se sempre atentar para os limites possíveis das
variáveis envolvidas. O conjunto dos valores possíveis para uma variável é
comumente conhecido como campo de definição ou domínio.

Coletânea de exercícios de
funções
1 - Seja f a função real de variável real definida pela sentença
f(x) = x / (x – 1)
a) determine os valores de f(0), f(1/2) e f(1)
b) determine o domínio e o conjunto imagem da função
c) determine a sua inversa

Solução:

a) determine os valores de f(0) , f(1/2) e f(1)

Sendo f(x) = x / (x – 1), f(0) = 0 / (0 – 1) = 0 / (-1) = 0


Analogamente, f(1/2) = (1/2) / (1/2 – 1) = (1/2) / (-1/2) = -1
Quanto a f(1), observe que f(1) = 1 // 1 – 1 = 1 / 0 e como não é
possível a divisão por zero, dizemos que a função f não está
definida para x = 1. Podemos dizer de uma forma equivalente que 1
não pertence ao domínio ou campo de definição da função dada.

b) determine o domínio e o conjunto imagem da função

Ora, sendo o domínio ou campo de definição de uma função


formado pelos valores possíveis para a variável independente x,
deveremos ter como condição de existência para y (variável
dependente),
que x – 1 0 ou x 1, já que não é possível a divisão por zero.
Logo, poderemos escrever o domínio desta função como sendo:
D(f) = {x R; x 1} = R – {1} , onde R é o conjunto dos números
reais.

Para achar o conjunto imagem da função ou seja, os valores


assumidos pela variável dependente y, teremos, lembrando que y =
f(x) :
y = x / (x – 1) multiplicando ambos os membros por x – 1 0
(para eliminar o denominador x – 1, fica:
y (x – 1) = x .
Vamos resolver esta igualdade em relação a x :
y.x – y = x x.y – x = y x(y – 1) = y x = y / (y – 1)
Ora, como y – 1 está no denominador , tem de ser obrigatoriamente
diferente de zero,
ou seja: y – 1 0 y 1.
Portanto, o conjunto imagem da função será dado por:
Im(f) = { y R; y 1} = R – {1}.

c) determine a sua função inversa f -1(x)

Sendo f(x) = y = x / (x – 1), para achar a expressão que define a


sua inversa, basta permutar as variáveis x e y, obtendo: x = y / (y –
1). Vamos resolver esta expressão em relação a y .
Multiplicando ambos os membros por y – 1 0 (para eliminar o
denominador
(y – 1), fica: x (y – 1) = y x.y – x = y x.y – y = x y (x – 1) =
x
y = x / (x – 1) ou seja: f -1(x) = x / (x – 1).
Observe que a expressão que define a função f e a sua inversa f –1
neste caso são iguais. Isto na ampla maioria das vezes não ocorre.

2 - Sobre a função y = x / (x – 1) é correto afirmar:

a) o seu domínio é o conjunto R dos números reais


b) o seu conjunto imagem é o conjunto R dos números reais
c) o seu gráfico é uma parábola de eixo vertical
d) o seu gráfico é uma curva simétrica em relação à reta y = x
e) todas as alternativas anteriores são incorretas
Solução:

As alternativas (a) e (b) são incorretas pois vimos na questão


anterior que o domínio e o conjunto imagem são ambos iguais a R
– {1} e, portanto, diferentes de R.

A alternativa (c ) também é falsa, pois as parábolas de eixo vertical


são representações das funções quadráticas do tipo f(x) = ax2 + bx
+ c com a 0.

Vamos analisar a alternativa (d): vimos no exercício anterior que as


expressões que definem a função f(x) e a sua inversa f –1(x) são
iguais. Ora, já sabemos que o gráfico de uma função e da sua
inversa são curvas simétricas em relação à reta y = x (bissetriz do
primeiro quadrante). Como f e f –1 neste caso são iguais, os seus
gráficos terão que ser necessariamente iguais. Portanto, o gráfico
da função f será uma curva simétrica em relação à reta y = x e a
alternativa (d) é verdadeira.

3 - Sendo f uma função tal que f(x + 10) = x – 10, pede-se


determinar x de modo
que se tenha x + f(x) = 10.

Solução :

Sendo f(x + 10) = x – 10 , façamos x + 10 = u ; isto é conhecido


como mudança de variável.

Se x + 10 = u , teremos que x = u – 10. Substituindo na expressão


inicial vem:
f(u) = (u – 10) – 10 = u – 20 .
Portanto, f(u) = u – 20.
Então poderemos escrever f(x) = x – 20.
O problema pede para determinar x de forma que x + f(x) = 10.
Substituindo, teremos:
x + (x – 20) = 10 2x = 10 + 20 = 30 x = 30 / 2 = 15.
Resposta: x = 15.

4 – Seja F uma função de domínio N = {1, 2, 3, 4, 5, ...} , tal que


F(1) = 2 e
F(n + 1) = [2.F(n) + 1] / 2
Nestas condições, o valor de F(101) é:

a) 49
b) 50
c) 51
d) 52
e) 100

Solução:

Temos: F(n+1) = (2F(n) + 1) / 2


n=1 F(1 + 1) = (2F(1) + 1) / 2
Como é dado que F(1) = 2, vem substituindo:
F(2) = (2.2 + 1) /2 = 5/2
Usando o resultado obtido acima para F(2) vamos determinar F(3):
F(2 + 1) = (2F(2) + 1) / 2 = (2.5/2 + 1) / 2 = 3
F(3) = 3
Analogamente,
F(3 + 1) = (2F(3) + 1) / 2 = (2.3 + 1) / 2 = 7/2
f(4) = 7/2
Vamos resumir os resultados já obtidos:
F(1) = 2 = (1 + 3) / 2
F(2) = 5/2 = (2 + 3) / 2
F(3) = 3 = (3 + 3) / 2
F(4) = 7/2 = (4 + 3) / 2
...........................................................................................................
Da observação dos resultados anteriores é razoável supor que
F(n) = (n + 3) / 2

Portanto, F(101) = (101 + 3) / 2 = 104 / 2 = 52, o que nos leva à


alternativa (d).

5 – Sendo f uma função real de variável real tal que f(x + y) = x +


f(y) e sendo
f(0) = 2 , pede-se determinar f(2000).

Solução:
Observe que fazendo x = 2000 e y = 0, a solução é imediata pois:
f(2000 + 0) = 2000 + f(0) e como é dado que f(0) = 2, vem:
f(2000) = 2000 + 2 = 2002

6 – Seja f uma função real de variável real que satisfaz à condição:


f(x) + 2f(2002/x) = 3x para x > 0.
Nestas condições, o valor de f(2) + f(1001) é:
a) 1003
b) 2003
c) 3003
d) 2000
e) - 997

Solução :

Fazendo x = 2 na expressão dada, vem:

f(2) + 2f(2002 / 2) = 3.2


f(2) + 2f(1001) = 6

Fazendo x = 1001 na expressão dada, fica:

f(1001) + 2f(2002 / 1001) = 3.1001


f(1001) + 2.f(2) = 3003

Temos então o seguinte sistema de duas equações e duas


incógnitas:
f(2) + 2f(1001) = 6
f(1001) + 2.f(2) = 3003
Tirando o valor de f(2) na primeira vem: f(2) = 6 – 2f(1001)
Substituindo na Segunda, fica:
f(1001) + 2(6 – 2f(1001)) = 3003
Desenvolvendo, teremos:
f(1001) + 12 – 4f(1001) = 3003
Resolvendo em relação a f(1001), vem:
-3f(1001) = 3003 – 12
–3f(1001) = 2991
Logo, f(1001) = 2991 / (-3) = - 997

Substituindo o valor de f(1001) na expressão em azul acima, fica:


f(2) = 6 – 2 (– 997) = 6 + 1994 = 2000
Portanto, a soma f(2) + f(1001) é igual a:
f(2) + f(1001) = 2000 + (– 997) = 2000 – 997 = 1003, o que nos leva
à alternativa (a).

7 – Seja f uma função definida para todo x real, satisfazendo às


condições:
f(3) = 2 e f(x + 3) = f(x) . f(3) . Nestas condições, determine f(-3).

Solução:

Fazendo x = 0 vem: f(0 + 3) = f(0).f(3) f(3) = f(0) . f(3) , de onde


concluímos
inevitavelmente que f(0) = f(3) / f(3) = 1.

Fazendo x = -3, fica: f(-3 + 3) = f(-3).f(3) ou, f(0) = f(-3) . f(3)


Como já sabemos que f(0) = 1, vem substituindo:
1 = f(-3) . 2 , pois é dado no enunciado que f(3) = 2.
Portanto, f(-3) = 1 / 2.
Agora resolva estes:
1 – Sendo f uma função tal que f(x + 1) = 2x, determine f(2x).
Resposta: f(2x) = 4x – 2

2 – Seja f uma função real de variável real que satisfaz à condição:


f(x) + 2f(2002/x) = 6x para x > 0.
Nestas condições, calcule o valor de f(2) .
Resposta: f(2) = 4000.

3 – Qual o domínio da função y = (x +1) / (x2 + 1) ?


Resposta: R (conjunto dos números reais).

Argumentos
lógicos

Antes de iniciar, é altamente recomendável que você revise os arquivos:


Lógica I
Lógica II
Lógica III

Dado um conjunto de proposições P1, P2 , P3 , ... , Pn, Q (simples ou


compostas) chama-se ARGUMENTO à proposição composta S : ( P1 P2
P3 ... Pn ) Q .
As proposições P1, P2 , P3 , ... , Pn são denominadas PREMISSAS e a
proposição Q é denominada CONCLUSÃO.
Costuma-se representar um argumento, também da forma simplificada:
P1, P2 , P3 , ... , Pn Q, onde o símbolo significa "logo" ou "de onde se
deduz " .

O argumento S : ( P1 P2 P3 ... Pn ) Q será VÁLIDO se e


somente se a proposição composta
s : ( P1 P2 P3 ... Pn ) Q for uma TAUTOLOGIA, ou seja, a última
coluna da sua TABELA VERDADE só contiver o valor lógico verdadeiro (V).
Caso contrário, o argumento não será válido e será denominado FALÁCIA.

Consideremos o seguinte exemplo de argumento:

Se chove então faz frio.


Não chove,
Logo, não faz frio.
Este argumento é válido? Vejamos:

Sejam as proposições:
p: " chove "
q: " faz frio "
Claro que a proposição "não chove" será ~p (a negação de p) e "não faz frio"
será ~q (a negação de q). Poderemos então escrever o argumento na forma
simbólica indicada acima:

s: [(p q) ~p] ~q

Para saber se o argumento apresentado é válido ou não, teremos que


construir a tabela verdade da proposição composta
s: [(p q) ~p] ~q.

Teremos, com base nos nossos conhecimentos anteriores:

p q ~p ~q p [(p q) s
q ~p
V V F F V F V
V F F V F F V
F V V F V V F
F F V V V V V

Como a proposição composta


s: [(p q) ~p] ~q não é uma Tautologia (apareceu um F na terceira
linha da última coluna), concluímos que o argumento dado não é válido. O
argumento é, portanto, uma FALÁCIA.

Vamos agora considerar o seguinte argumento:

Se chove então faz frio.


Não faz frio.
Logo, não chove.

Este argumento é válido? Vejamos:

Sejam as proposições:
p: " chove "
q: " faz frio "
Claro que a proposição "não chove" será ~p (a negação de p) e "não faz frio"
será ~q (a negação de q). Poderemos então escrever o argumento na forma
simbólica:

s: [(p q) ~q] ~p

Para saber se o argumento apresentado é válido ou não, teremos que


construir a tabela verdade da proposição composta
s: [(p q) ~q] ~p .

Teremos, com base nos nossos conhecimentos anteriores:

p q ~p ~q p [(p q) s
q ~q
V V F F V F V
V F F V F F V
F V V F V F V
F F V V V V V
Como a proposição composta
s: [(p q) ~q] ~p é uma Tautologia (só aparece V na última coluna),
concluímos que o argumento dado é válido.

Este tipo de problema se complica um pouco quando o número de premissas


aumenta, pois com duas premissas, a tabela verdade conterá 22 = 4 linhas,
com três premissas, a tabela verdade conterá 23 = 8 linhas e assim
sucessivamente. Com quatro premissas, a tabela verdade conterá 24 = 16
linhas; imagine 10 premissas!
A tabela verdade conteria 210 = 1024 linhas. Aí, só os computadores
resolveriam ...

Considere outro exemplo, agora com 3 premissas:

Se o jardim não é florido então o gato mia.


Se o jardim é florido então o passarinho não canta.
O passarinho canta.
Logo, o jardim é florido e o gato mia.

Sejam as proposições:
p: “o jardim não é florido"
q: " o gato mia"
r: " o pássaro canta"
Poderemos escrever o argumento na seguinte forma simbólica:

s : [(p q) (~ p ~ r) r] (~p q)

Teremos, com base nos nossos conhecimentos anteriores:

p q r ~ ~p p ~p ~p [(p q) (~p r) s
r q q ~r (~r)
V V V F F V F V F V
V V F V F V F V V F
V F V F F F F V F V
V F F V F F F V F V
F V V F V V V F F V
F V F V V V V V V V
F F V F F V V F F V
F F F V F V V V V F
Como o argumento s não é uma Tautologia (apareceu F na última coluna) , o
argumento não é válido.

Notas:
1 – o entendimento da tabela verdade acima, requer muita atenção.
2 – neste tipo de exercício, não devemos usar a intuição, somente. A
construção da tabela verdade é uma necessidade imperiosa, embora possa
parecer muito trabalhosa.
3 – recomendamos enfaticamente, imprimir o arquivo e analisar
criteriosamente a tabela verdade.

Agora resolva estes:

1 - Se o jardim não é florido então o gato mia.


O gato não mia.
Logo, o jardim é florido.
Resposta: o argumento é válido.

2 - Se o jardim não é florido então o gato não mia.


O jardim é florido.
Logo, o gato mia.
Resposta: o argumento não é válido.

Um problema
interessante
Vejam o e-mail que recebi de um brasileiro deste imenso Brasil:

Olá, Caro professor,

Sou muito grato à tua louvável inspiração em criar um site tão cheio de
conteúdo e possibilidades. Por meio desta possibilidade criada, venho a ti,
muito reverentemente pedir ajuda em esclarecer-me em relação a uma
questão que vem me deixando impaciente.

Eis a questão:

"Com os algarismos x, y e z, formam-se os números de dois algarismos: xy e


yx, cuja soma é o número de três algarismos zxz. Quanto valem x, y e z?"
Desde já agradeço ao emérito professor pela ajuda e se possível pela breve
instrução com relação a esta e outras similares questões serão bem vindas.
Um abraço.

Solução:
Inicialmente quero agradecer-lhe pela visita e elogio ao site
Entretanto, recomendo enfaticamente uma revisão sobre valor posicional de
um algarismo, premissa básica para entender a solução apresentada a seguir.

A solução deste problema pode ser iniciada considerando-se que:

1) o número (xy) pode ser expresso como 10x + y


2) o número (yx) pode ser expresso como 10y + x
3) o número (zxz) pode ser expresso como 100z + 10x + z .

Exemplos:
35 = 10.3 + 5
53 = 10.5 + 3
303 = 100.3 + 10.0 + 3
etc.

Então, como é dito no enunciado que a soma dos números (xy) e (yx) resulta
em (zxz), com base nos argumentos anteriores, é lícito escrever:

(xy) + (yx) = (zxz)


[10x + y] + [10y + x] = [100z +10x +z]

Reduzindo os termos semelhantes, fica:


11x + 11y = 101z + 10x
x + 11y = 101z
(x + 10y) + y = 100z + z
(10y + x) + y = 100z + z
Ocorre que 10y + x = (yx) e 100z + z = 100z + 0z + z = (z0z)

Logo,
(yx) + y = (z0z)

Como x, y e z assumem valores de 1 a 9, a única combinação possível é 92 +


9 = 101 e, portanto,
x = 2, y = 9 e z = 1.

PS.: Como não recebi autorização para publicar o e-mail do gerador do


enunciado da questão, refiro-me simplesmente a um brasileiro, o que nos
parece não ferir susceptibilidades nem sensibilidades.

Domínio e conjunto imagem de uma função


real de variável real
Já vimos no capítulo FUNÇÕES a definição de domínio (ou campo de
definição) de funções reais de variável real.
Falamos acima em função real de variável real. Bem, o que significa
exatamente isto?
Partindo-se da premissa que você já conhece o conceito de FUNÇÃO ,
podemos dizer que uma função
f:A B, definida por y = f (x) é uma função real de variável real , quando os
conjuntos A e B são subconjuntos de R ,
sendo R o conjunto dos números reais.
+++++++++++++++++++++++++++++++++++++++++++++++++++++++++++
+++
Seja a função f: A B; y = f (x).
Nestas condições, temos x Aey B. Os valores de x constituem o
domínio da função f e os valores de y constituem o conjunto imagem da
função f . O conjunto B é chamado contradomínio.

Nestas condições, determine o domínio e o conjunto imagem das seguintes


funções reais de variável real:
1 ) y = 1 + sqrt (x)
Nota: sqrt (x) = square root of x = raiz quadrada de x = x

Observando que a raiz quadrada de x é um número real se e somente se x for


positivo ou nulo, vemos que a condição de existência para y é que x 0.
Portanto, o domínio da função dada será
D = {x R;x 0} = R + (conjunto dos números reais não negativos).
Para determinar o conjunto imagem, temos que achar os valores possíveis
para y.
Teremos então: y – 1 = sqrt (x). Como a raiz quadrada de um número real
positivo ou nulo é outro número positivo ou nulo, deveremos ter y – 1 0 ou
y 1.
Portanto, o conjunto imagem da função é Im = {y R; y 1} = [1, ) ou
seja, o conjunto dos
números reais maiores ou iguais a 1.

2 ) y = 3 + sqrt (x – 5)
Neste caso, a condição de existência para y é que x – 5 0 ou seja, x 5.
O domínio da função dada será
D = {x R;x 5} = [5 , ) , ou seja, o intervalo de todos os números
reais
maiores ou iguais a 5.
Para determinar o conjunto imagem, temos que achar os valores possíveis
para y.
Teremos então: y – 3 = sqrt (x – 5 ). Como a raiz quadrada de um número real
positivo ou nulo é outro número positivo ou nulo, deveremos ter y – 3 0 ou
y 3.
Portanto, o conjunto imagem da função é Im = {y R; y 3} = [3, ) ou
seja, o conjunto dos
números reais maiores ou iguais a 3.
3 ) y = 2 + sqrt (– x )
Neste caso, a condição de existência para y é que – x 0. Multiplicando
ambos os membros dessa desigualdade por – 1 , ela muda de sentido, ou
seja, x 0. Portanto, o domínio da função será
D = {x R;x 0} = R – (conjunto dos números reais não positivos).
Para determinar o conjunto imagem, temos que achar os valores possíveis
para y.
Teremos então: y – 2 = sqrt (– x). Como a raiz quadrada de um número real
positivo ou nulo é outro número positivo ou nulo, deveremos ter y – 2 0 ou
y 2.
Portanto, o conjunto imagem da função é Im = {y R; y 2} = [2, ) ou
seja,
o conjunto dos números reais maiores ou iguais a 2.
4 ) y = x / (2x – 6)
Aqui, a condição para a existência de y é que o denominador 2x – 6 seja
diferente de zero, já que não existe divisão por zero. Portanto, 2x – 6 0 ou
seja, x 3 . Portanto, o domínio desta função é
D = {x R; x 3} = R – {3}, ou seja, o conjunto de todos os números reais
diferentes de 3.
Para determinar o conjunto imagem, temos que achar os valores possíveis
para y.
Vamos então, explicitar x em função de y. Teremos:
De y = x / (2x – 6), poderemos escrever: y(2x – 6) = x . Efetuando as
operações indicadas, vem:
2xy – 6y = x 2xy – x = 6y x(2y – 1) = 6y x = 6y / (2y – 1).
Ora, como não existe divisão por zero, deveremos ter 2y – 1 0 ou seja 2y
1 e, finalmente, y 1/2 . Portanto, o conjunto imagem da função dada é:
Im = {y R; y ½} = R – {1/2}, ou seja, o conjunto de todos os números
reais diferentes de ½.

5 ) y = sqrt (x – 1) + sqrt (5 – x)
Neste caso, as condições para a existência de y são que x – 1 0e5–x
0 . Logo,
x 1e5 x o que é o mesmo que 1 x 5, ou seja, o domínio da
função é
D = {x R; 1 x 5 } = [1, 5] (intervalo fechado dos números reais de 1 a
5).
Para determinar o conjunto imagem, teremos que achar os valores possíveis
para y.
Como x pode variar em R (conjunto dos números reais) de 1 até 5, poderemos
escrever para valores inteiros de x :
Observação: claro que sendo x um número real, ele assume também valores
não inteiros, os quais não utilizaremos aqui, pois complicaria os cálculos,
desnecessariamente.
y = sqrt (x – 1) + sqrt (5 – x) = (x-1) + (5-x)
x=1 y = sqrt (1 – 1) + sqrt (5 – 1) = sqrt 0 + sqrt 4 = 0 + 2 = 2
x=2 y = sqrt (2 – 1) + sqrt (5 – 2) = sqrt 1 + sqrt 3 = 1 + 3 1 + 1,732
2,732
x=3 y = sqrt (3 – 1) + sqrt (5 – 3) = sqrt 2 + sqrt 2 = 2.sqrt2 = 2 2
2.1,414 2,818
x=4 y = sqrt (4 – 1) + sqrt (5 – 4) = sqrt 3 + sqrt 1 = 3+1 1,732 + 1
2,732
x=5 y = sqrt (5 – 1) + sqrt (5 – 5) = sqrt 4 + sqrt 0 = 2 + 0 = 2
Observe que para x inteiro de 1 a 5, y variou de 2 até voltar novamente a a 2,
passando pelo valor máximo 2,818... = 2 2.
Portanto, o conjunto imagem desta função é Im = {y R;2 y 2 2} =
[2, 2 2], ou seja, o intervalo fechado de números reais de 2 a 2 2.
Nota: neste problema, não tentamos explicitar x em função de y por meios
algébricos, pois o caminho seria por demais tortuoso e até um certo ponto
inviável, face ao trabalho imenso necessário. Seria na verdade o que eu
chamo de sacrifício sem glória! Usando derivadas – determinação dos pontos
máximos e mínimos de uma função – seria menos trabalhoso, mas, este
assunto vocês só verão no 1º ano de faculdade, para quem optar por cursos
na área de ciências exatas.
Agora resolva este:
Qual o domínio e o conjunto imagem da função y = sqrt (x – 2) – sqrt (2 – x)?
Resposta: D = {2} e Im = {0}.
Um comentário importante.
Para que exista uma função f , são necessários dois conjuntos A e B e uma lei
y = f (x) que a defina, ou seja:
f: A B ; y = f (x). Portanto, quando nos referimos a y = f (x) como sendo a
própria função, estamos na verdade cometendo um abuso de linguagem, pois
estamos confundindo a função com a lei que a define.
Outro aspecto a ser considerado é que dada uma função, o seu domínio já
existe implicitamente.
Perguntar qual o domínio de uma função, é também um abuso de linguagem.
Contudo, estas formas de expressão são tradicionalmente aceitas pelo uso,
não se constituindo em propriamente um erro.
É conveniente entretanto que se tenha isto em mente.
No entanto, perguntar qual o conjunto imagem de uma função, não se
constitui em abuso de linguagem pois, normalmente, as funções são definidas
por uma lei – f(x) – e por dois conjuntos A e B onde B é o seu contradomínio.
Aliás, é conveniente registrar que quando o conjunto imagem de uma função
coincide com o seu contradomínio, ela recebe a denominação especial de
função sobrejetora.
Num Centro
Esportivo
UFBA 1993 - O diretor de um Centro Esportivo vai cercar um campo de forma
retangular e depois dividi-lo com outra cerca, para formar duas quadras
retangulares. A cerca que vai dividir o campo internamente custa Cr$
10000,00 por metro, e a cerca externa custa Cr$ 25000,00 por metro.
Sabendo-se que o diretor dispõe de Cr$ 4.800.000,00 para realizar a obra,
determine o semiperímetro do campo de maior área cercada possível.

Nota: UFBA = Universidade Federal da Bahia

Solução:

Nota: vemos que em 1993 a moeda oficial era o cruzeiro – Cr$ - que tinha
uma "porrada" de zeros.
Os Cr$ 4.800.000,00 daquela época, valeriam hoje cerca de R$1012,00. Não
calculei isto; usei um programinha de atualização monetária de um CD que
adquiri numa banca de revistas.

Veja a figura a seguir:

onde x, y e z são as medidas expressas em metros (m).


O perímetro (soma das medidas dos lados) do campo de forma retangular é
igual a:
P = x + y + z + x + y + z = 2(x + y + z)
O custo para construir esta cerca será então:
C1 = 2(x + y + z) . Cr$25000,00 = 50000(x + y + z)

A cerca que vai dividir o campo em duas quadras retangulares tem


comprimento z. Logo, o custo para construí-la será C2 = z . Cr$10000,00 =
10000z

O custo total será igual a C = C1 + C2 = 50000(x + y + z) + 10000z


Desenvolvendo a expressão acima, fica:
C = 50000x + 50000y + 50000z + 10000z = 50000x + 50000y + 60000z

Como o diretor dispõe de Cr$4.800.000,00 o custo total deverá ser igual a


esse valor, ou seja:
50000x + 50000y + 60000z = 4800000
Dividindo ambos os membros por 10000 , para eliminar os zeros, vem:
5x + 5y + 6z = 480 (equação 1)
A área do campo de forma retangular será igual a : S = (x + y) . z, conforme
pode-se inferir (deduzir ) da figura acima.
Da equação (1) acima, vem: 5(x + y) + 6z = 480
Daí tiramos x + y = (480 – 6z) / 5
Então, a área S será igual a S = [(480 – 6z) / 5] . z = [(480/5) – (6z/5)].z
Desenvolvendo, fica:
S = (480z / 5) – (6z2 / 5)

Observe que a área S é uma função quadrática de z, ou seja,


S = (- 6/5) z2 + (480/5) z

Temos então uma função quadrática S = f(z) da forma


S = az2 + bz + c onde a = - 6/5, b = 480/5 e c = 0.

Como vocês estão acostumados com a função quadrática na forma y = ax2 +


bx + c, aqui é como se o S fosse o y e o z fosse o x. Porém, isto não muda
nada no problema.

Observe que na função S = (- 6/5) z2 + (480/5) z , temos que a < 0 e, portanto,


a função possui um valor máximo, como já sabemos do estudo das funções
quadráticas ou funções do segundo grau.

Como queremos determinar os valores de x, y e z (veja figura acima) para que


a área seja máxima, vamos começar determinando o valor de z para que S
seja máxima, ou seja, calcular o zv (coordenada do vértice) da função S = (-
6/5) z2 + (480/5) z.

Teremos:

Ora, como z = 40, e x + y = (480 – 6z) / 5 conforme vimos acima, vem,


substituindo o valor de z:
x + y = (480 – 6.40) / 5 = (480 – 240) / 5 = 240 / 5 = 48

Portanto, como x + y = 48 e z = 40, o perímetro P que é a soma das medidas


dos lados será igual a
P = 2(x + y + z) = 2(48 + 40) = 176 metros = 176 m

Mas, o problema pede o semiperímetro, ou seja, a metade do perímetro.


Logo, o semiperímetro p será igual a
p = P/2 = 2(x + y + z) / 2 = 176/2 = 88 = 88 metros = 88 m , que é a resposta
do problema.

Resposta: para que a área seja máxima, o semiperímetro do campo


retangular deverá ser igual a 88 m.

Agora resolva este:

Qual a área máxima do campo de forma retangular do exercício anterior?

DICA: Basta fazer z = 40 na função S = (- 6/5) z2 + (480/5) z , ou calcular a


ordenada do vértice através da conhecida fórmula Sv = yv = - / 4a .
Observe que usei a letra y apenas porque vocês estão acostumados a
escrever a função quadrática como y = ax2 + bx + c. Na verdade não seria
necessário isto.

Resposta: área máxima = 1920 metros quadrados = 1920 m2

Dois problemas, duas soluções e duas


propostas imperdíveis.
1 - Para produzir uma peça, uma indústria gasta $1,20 por unidade. Além
disso, há uma despesa fixa de $4000,00 independente da quantidade
produzida. O preço de venda é de $2,00 por unidade. O número mínimo de
peças que devem ser vendidas para que a indústria tenha lucro é:
a) 3000
b) 3010
c) 4999
d) 4010
e) 5001

Solução:

Sendo C o custo de produção de n peças, pelo enunciado poderemos


escrever:
C = 1,20n + 4000

Sendo V = o preço de venda de n peças, teremos V = 2n .


O lucro L será dado evidentemente por L = V – C .
Para que haja lucro deveremos ter L = V – C > 0.
Substituindo C e V pelos valores conhecidos, ou seja: C = 1,20n + 4000 e V =
2n, vem:
2n – (1,20n + 4000) > 0. Daí vem que 2n – 1,20n – 4000 > 0.
Portanto, 0,80n – 4000 > 0 ou 0,80n > 4000, de onde tiramos n > 4000 / 0,8 ,
ou seja,
n > 4000,0 / 0,8 que é equivalente a n > 40000 / 8 n > 5000.
Observe que para n = 5000 peças, o lucro L será zero, pois teremos: V = 2n =
2.5000 = 10000 e
C = 1,20n + 4000 = 1,20.5000 + 4000 = 10000. Portanto, para que a indústria
tenha lucro, o número mínimo de peças vendidas deve ser 5001, o que nos
leva tranqüilamente à alternativa E.

2 – Seja g(x) uma função cujo domínio é o conjunto dos números inteiros,
assim definida:
g(n) = – 1 se n é par.
g(n) = 3n se n é ímpar.
Nestas condições, dada a soma A = g(1) + g(2) + g(3) + g(4) + g(5) + ...+ g(2k
– 1) + g(2k) para k inteiro, o valor de A + k é igual a:
a) 2k3
b) 3k3
c) 3k2
d) 2k2
e) 3k

Solução:

Temos: A = g(1) + g(2) + g(3) + g(4) + g(5) + ...+ g(2k – 1) + g(2k).


Observe que a soma A possui 2k parcelas. Esta soma A poderá ser reescrita
como:
A = S1 + S2 = [g(2) + g(4) + ... + g(2k)] + [g(1) + g(3) + g(5) + ... g(2k – 1)] ,
onde:
S1 = g(2) + g(4) + ... + g(2k) . Repare que a soma S1 possui k parcelas.
S2 = g(1) + g(3) + g(5) + ... g(2k – 1). Repare que a soma S2 possui também k
parcelas.

Do enunciado, temos que g(2) = g(4) = g(6) = ... = g(2k) = – 1, já que n = 2, 4,


6, ... , 2k são números pares. Portanto, S1 = (-1) + (-1) + ... + (-1) , uma soma
com k parcelas iguais a (– 1), e, portanto igual a k(-1) = – k. Então, S1 = – k .

Também do enunciado, vem que:


g(1) = 3.1 = 3, g(3) = 3.3 = 9, g(5) = 3.5 = 15, ... , g(2k – 1) = 3(2k – 1) = 6k –
3.
A soma S2 é então igual a :
S2 = 3 + 9 + 15 + ... + (6k – 3).
Vemos que a soma S2 é igual à soma dos k primeiros termos (lembre-se que
vimos acima que S2 possui k parcelas) de uma Progressão Aritmética - PA de
primeiro termo a1 = 3, razão r = 6 e último termo
ak = 6k – 3.
Usando a conhecida fórmula da soma dos termos de uma PA : S = [(a1 + an).n]
/ 2, vem:
S2 = 3 + 9 + 15 + ... + 6k – 3 = [(3 + 6k – 3 ) . k] / 2 = [(6k) . k] / 2 = 6k2 / 2 = 3k2

Portanto, a soma A procurada será dada por A = S1 + S2 = – k + 3k2 = 3k2 – k,


e, portanto,
A + k = 3k2 – k + k = 3k2 , o que nos leva à alternativa C.

Agora resolva estes:

a) Para produzir uma peça, uma indústria gasta $1,50 por unidade. Além
disso, há uma despesa fixa de $3000,00 independente da quantidade
produzida. O preço de venda é de $3,00 por unidade. Pede-se determinar o
número mínimo de peças que devem ser vendidas para que a indústria tenha
lucro.
Resposta: 2001

b) Seja f(x) uma função cujo domínio é o conjunto Z (dos números inteiros),
assim definida:
f(n) = – 2 se n é par.
f(n) = 5n se n é ímpar.
Nestas condições, dada a soma B = f(1) + f(2) + f(3) + f(4) + f(5) + ...+ f(2k – 1)
+ f(2k) para k inteiro, o valor de B + 2k é igual a:
Resposta: 5k2

DesenROCK-se

Se 70% de uma certa população gosta de samba, 75% de chorinho,


80% de bolero e 35% de rock, a percentagem mínima dessa população
que gosta de samba, chorinho, bolero e rock é igual a:
A) 5%
B) 10%
C) 15%
D) 1%
E) faltam dados para a resposta
Solução:

Notas:
a) samba = música popular brasileira, de origem africana.
b) bolero = música de origem espanhola.
c) chorinho = muito provavelmente, o primeiro ritmo genuinamente
brasileiro.
d) rock = música de origem nos EUA.

Vi esta questão, proposta numa apostila antiga (década de 90),


informando que teria sido aplicada numa prova da Escola Naval, sem
especificar o ano. A resposta atribuída à questão naquela antiga
apostila (já com sinais de traças) era 10%.

Fundamentado num argumento bastante simples, sugerido por um


visitante ilustre do site - Hélio Fragoso - , vou compartilhar com vocês a
solução simples sugerida pelo ilustre professor, com algumas
adaptações minhas.
Apresentarei inclusive ao final do arquivo, minhas considerações sobre
a questão, evidenciando a veracidade do método simples utilizado. A
resposta 10% da quase antiga apostila está errada.

Sejam os conjuntos:
S = população que gosta de samba
C = população que gosta de chorinho
B = população que gosta de bolero
R = população que gosta de rock
P = população total

Se 80% da população gosta de bolero e 75% gosta de chorinho e como


a soma dessas porcentagens excede 100%, no mínimo, 55% da
população gosta de bolero e chorinho (80 + 75 – 100 = 55).

Nota: lembre-se que a fórmula do número de elementos da união de


dois conjuntos A e B é dada por:
n(AUB) = n(A) + n(B) – n(A B)
Logo, 100%P = 80%P + 75%P – n(A B) n(A B) = 55%P onde
P é a população total.
Fazendo o mesmo raciocínio para as seis combinações possíveis
tomadas duas a duas das quatro formas de música, teremos o quadro
seguinte:
Bolero e 80 + 75 – 100 =
B C Chorinho 55% 55
Samba e 70 + 80 – 100 =
S B Bolero 50% 50
Chorinho e 70 + 75 – 100 =
C S Samba 45% 45
80 + 35 – 100 =
B R Bolero e Rock 15% 15
Chorinho e 75 + 35 – 100 =
C R Rock 10% 10
80 + 35 – 100 =
S R Samba e Rock 5% 5

Lembre-se que pelo enunciado 70% da população gosta de samba,


75% de chorinho, 80% de bolero e 35% de rock.
Ora, como para gostar dos quatro gêneros de música é preciso gostar
de samba e rock e 5% é a mínima porcentagem da população que
gosta de samba e rock, esta é também a mínima porcentagem da
população que gosta dos quatro gêneros de música.
Portanto, a resposta correta é a alternativa A.

Como uma forma de verificar esta solução apresentada pelo Prof.


Fragoso, construí o seguinte raciocínio, aparentemente
complicado mas, inteiramente válido:

Considerando-se que, sendo P a população total, S = {samba}, C =


{chorinho},
B = {bolero} e R = { rock} , é verdadeiro pelo enunciado que:

0,70P = n(S), de onde vem P = n(S) / 0,70


0,75P = n(C), de onde vem P = n(C) / 0,75
0,80P = n(B), de onde vem P = n(B) / 0,80
0,35P = n(R), de onde vem P = n(R) / 0,35

Portanto, poderemos escrever:


P = n(S) / 0,70 = n(C) / 0,75 = n(B) / 0,80 = n(R) / 0,35

Daí, vem por uma das propriedades das proporções, que:

n(S)/0,70 = n(C)/0,75 = n(B)/0,80 = n(R)/0,35 = [(n(S)+ n(C)+ n(B)+


n(R)] / [0,70 + 0,75 + 0,80 + 0,35]
= [(n(S) + n(C) + n(B) + n(R)] / 2,60

Usando a primeira igualdade (realçada em negrito) do conjunto de


igualdades acima:
n(S) / 0,70 = [(n(S) + n(C) + n(B) + n(R)] / 2,60

Daí vem:
2,60 n(S) = 0,70 [(n(S) + n(C) + n(B) + n(R)]
2,60 n(S) = 0,70 n(S) + 0,70 n(C) + 0,70 n(B) + 0,70 n( R)
Igualando a zero:
2,60 n(S) - 0,70 [(n(S) – 0,70 n(C) – 0,70 n(B) – 0,70 n(R) = 0
1,90 n(S) – 0,70 n(C) – 0,70 n(B) – 0,70 n(R) = 0.

Ou, de forma equivalente:


- 1,90 n(S) + 0,70 n(C) + 0,70 n(B) + 0,70 n(R) = 0

Procedendo de forma análoga, usando as igualdades acima uma a


uma, chegaremos ao seguinte sistema linear homogêneo de quatro
equações e quatro incógnitas:

- 1,90n(S) + 0,70n(C) + 0,70n(B) + 0,70n(R) = 0


0,75 n(S) –1,85 n(C) + 0,75n(B) + 0,75n(R) = 0
0,80 n(S) + 0,80n(C) –1,80n(B) + 0,80n(R) = 0
0,35 n(S) + 0,35n(C) + 0,35n(B) – 2,25n(R) = 0

cuja solução geral são as quádruplas do tipo [(2k), (15k / 7), (16k / 7),
(k)] sendo k um número real.

Nota: usei um software (que roda no modo DOS) para resolver o


sistema linear acima.
Manualmente, a solução deste sistema linear seria um verdadeiro
suplício!!!. O responsável pelo suplício são os coeficientes decimais das
equações. O software que usei – LINSYS - , obtive na Internet. Não
coloquei o link porque os links na WEB mudam freqüentemente de
lugar. Porém, digitando linsys num programa de busca, você
certamente encontrará páginas que disponibilizam este software
simples e bastante útil.

Voltando à questão:

Por exemplo:
Fazendo k = 35, teremos a quadra (70, 75, 80, 35) e P = 100.(a
população total é igual a 100% de P)

Para facilitar a construção de conjuntos que atendam ao problema , fiz


k =7, obtendo a quadra
(14, 15, 16, 7) e P = 20, o que mantém os mesmos percentuais
relativos do problema proposto,
pois 14/20 = 0,70, 15/20 = 0,75, 16/20 = 0,80 e 7/20 = 0,35.

Então, construí os seguintes conjuntos, sendo A i uma pessoa i da


população P:

S = {samba} = {A1, A2, A3, ... , A14}


C = {chorinho} = {A1, A2, A3, ... , A14, A15}
B = {bolero} = {A1, A2, ... , A15, A16}
R = {rock} = {A14, A15, A16, A17, A18, A19, A20}

Observe que:
S C B R = {A14} (um elemento) e
S U C U B U R = {A1, A2, ... , A20} (20 elementos).

Ora, se a interseção dos quatro conjuntos possui apenas um elemento


e a união dos quatro conjuntos possui vinte elementos, então 1/20 =
0,05 = 5%.

Se a resposta correta fosse "no mínimo 10%" (como sugerida na


apostila antiga), teríamos
10% de 20 = 2., que conflita inapelavelmente com a quantidade 1,
encontrada logicamente acima para a interseção.
Logo, a resposta 10% não pode estar correta.

O problema proposto não é tão simples assim como parece à primeira


vista!

Nota: o título atribuído ao arquivo – DesenROCK-se – é de uma música


do grande cantor e compositor de Irará – BA – do Brasil e do Mundo,
TOM ZÉ. Uma singela homenagem a Tom Zé, um gênio da música!. .

Contando elementos I
UFBA 1986) Uma pesquisa realizada com um grupo de pessoas
revelou a seguinte preferência pelas revistas A, B e C:
109 lêem a revista A ;
203 lêem a revista B ;
162 lêem a revista C ;
41 lêem as revistas B e C;
28 lêem as revistas A e C;
5 lêem as três revistas
115 não lêem revista.
Das informações conclui-se:

(01) 500 pessoas foram consultadas.


(02) 51 pessoas lêem somente a revista A.
(04) 176 pessoas não lêem as revistas B ou C.
(08) 94 pessoas lêem pelo menos duas revistas.
(16) 223 pessoas lêem as revistas A ou B e não lêem a revista C.

Solução:

Esta modalidade de questão na UFBA – Universidade Federal da


Bahia - consiste em identificar as respostas corretas e somar os
valores associados a cada uma delas, marcando o resultado na
Folha de Respostas.
Veja a figura a seguir, onde x, y, z, w, p, q, r e s representam
número de elementos:

Do enunciado poderemos concluir o que segue:

x + y + w + p = 109 (número de pessoas que lêem a revista A).


y + z + q + w = 203 (número de pessoas que lêem a revista B).
p + w + q + r = 162 (número de pessoas que lêem a revista C)
y + w = 25 (número de pessoas que lêem as revistas A e B)
w + q = 41 (número de pessoas que lêem as revistas B e C).
w + p = 28 (número de pessoas que lêem as revistas A e C).
w = 5 (número de pessoas que lêem as revistas A, B e C).
s = 115 (número de pessoas que não lêem revista).

De w = 5 e w + p = 28, vem que 5 + p = 28 p = 23.

De w = 5 e w + q = 41, vem que 5 + q = 41 q = 36.

De w = 5 e y + w = 25, vem que 5 + y = 25 y = 20.

Substituindo os valores de p, q, w e y nas igualdades anteriores,


fica:
x + y + 5 + 23 = 109 x + y = 81
20 + z + 36 + 5 = 203 z = 142
23 + 5 + 36 + r = 162 r = 98

Vamos analisar cada alternativa, utilizando os valores já


encontrados acima:
(01) 500 pessoas foram consultadas.
Observe pela figura que o número total de pessoas consultadas
será igual à soma
x + y + z + w + p + q + r + s = (x + y) + z + w + p + q + r + s
Substituindo os valores conhecidos, vem:
81 + 142 + 5 + 23 + 36 + 98 + 115 = 500
Portanto, a alternativa (01) é correta.

(02) 51 pessoas lêem somente a revista A.


Observe na figura acima que o número de pessoas que lêem
somente a revista A é exatamente igual a x. Vimos acima que x + y
= 81 e que y = 20.
Logo, x + 20 = 81 x = 61.
Portanto a alternativa (02) é falsa.

(04) 176 pessoas não lêem as revistas B ou C.


Observe que o número de pessoas que não lêem as revistas B ou
C, é igual à soma x + s ou seja como x = 61 e s = 115, vem: x + s =
61 + 115 = 176
Portanto a alternativa (04) é verdadeira.

(08) 94 pessoas lêem pelo menos duas revistas.


A expressão “lêem pelo menos duas revistas” significa “lêem duas
ou três revistas”

Olhando a figura vemos que o número de pessoas que lêem pelo


menos duas revistas é igual a y + p + q + w = 20 + 23 + 36 + 5 = 84
Portanto a alternativa (08) é falsa.

(16) 223 pessoas lêem as revistas A ou B e não lêem a revista


C.
Observe que o número de pessoas que lêem as revistas A ou B e
não lêem a revista C é igual x + y + z = 61 + 20 + 142 = 223
Portanto, a alternativa (16) é verdadeira.

Somando os valores das alternativas verdadeiras: 01 + 04 + 16 =


21, número que deveria ser marcado na Folha de Respostas.

Agora resolva este:

Foi feita uma pesquisa sobre a preferência de um grupo de pessoas


por determinado tipo de música, e concluiu-se que:
43 pessoas gostavam de SAMBA.
6 pessoas gostavam apenas de ROCK.
15 pessoas gostavam de SAMBA e ROCK.
8 pessoas gostavam de ROCK e JAZZ.
13 pessoas gostavam de SAMBA e JAZZ.
3 pessoas gostavam apenas de ROCK e JAZZ.
40 pessoas não gostavam de ROCK.
Ninguém gostava apenas de JAZZ.
Podemos concluir que:

(01) 86 pessoas foram consultadas.


(02) 24 pessoas gostavam de ROCK.
(04) 36 pessoas gostavam de SAMBA ou ROCK, mas não
gostavam de JAZZ.
(08) 21 pessoas gostavam de pelo menos dois tipos de música.
(16) 12 pessoas não gostavam de nenhum dos três tipos de música
citadas.

Resposta: Somente as afirmativas (02), (04) e (16) são


verdadeiras. Logo, deveria ser marcado na Folha de Respostas o
número 02 + 04 + 16 = 22

Contando elementos II
Foi feita uma pesquisa sobre a preferência de um grupo de pessoas
por determinado tipo de música, e concluiu-se que:
43 pessoas gostavam de SAMBA.
6 pessoas gostavam apenas de ROCK.
15 pessoas gostavam de SAMBA e ROCK.
8 pessoas gostavam de ROCK e JAZZ.
13 pessoas gostavam de SAMBA e JAZZ.
3 pessoas gostavam apenas de ROCK e JAZZ.
40 pessoas não gostavam de ROCK.
Ninguém gostava apenas de JAZZ.

Podemos concluir que:

(01) 86 pessoas foram consultadas.


(02) 24 pessoas gostavam de ROCK.
(04) 36 pessoas gostavam de SAMBA ou ROCK, mas não
gostavam de JAZZ.
(08) 21 pessoas gostavam de pelo menos dois tipos de música.
(16) 12 pessoas não gostavam de nenhum dos três tipos de música
citadas.

Solução:

Sejam os conjuntos A = {gostam de SAMBA}, B = {gostam de


ROCK} e C = {gostam de JAZZ}.
Pelo enunciado e observando a figura acima, poderemos escrever:

x + y + w + p = 43 (número de pessoas que gostam de SAMBA).


z = 6 (número de pessoas que gostam apenas de ROCK)
y + w = 15 (número de pessoas que gostam de SAMBA e ROCK).
w + q = 8 (número de pessoas que gostam de ROCK e JAZZ).
p + w = 13 (número de pessoas que gostam de SAMBA e JAZZ).
q = 3 (número de pessoas que gostam de ROCK e JAZZ).
x + p + r + s = 40 (número de pessoas que não gostam de ROCK).
r = 0 (ninguém gostava apenas de JAZZ).

Substituindo os valores conhecidos nas igualdades acima, vem:

Como q = 3, vem de w + q = 8 que w = 5.

Como y + w = 15 e w = 5, vem que y = 10.

Como w = 5 e p + w = 13, vem que p = 8.

Ora, como y = 10, w = 5 e p = 8, vem de x + y + w + p = 43 que:


x + 10 + 5 + 8 = 43 x = 20.

Substituindo os valores conhecidos na igualdade x + p + r + s = 40,


vem:
20 + 8 + 0 + s = 40 s = 12.

Resumindo os valores encontrados:


x = 20, y = 10, z = 6, w = 5, p = 8, q = 3, r = 0 e s = 12.
Vamos analisar cada alternativa, utilizando os valores conhecidos
acima:

(01) 86 pessoas foram consultadas.


O número total de pessoas consultadas será igual a x + y + z + w +
p + q + r + s.
Substituindo os valores conhecidos, vem:
20 + 10 + 6 + 5 + 8 + 3 + 0 + 12 = 64
Portanto, a alternativa (01) é falsa.

(02) 24 pessoas gostavam de ROCK.


Sendo B = {gostam de ROCK} , o número total de pessoas que
gostam de ROCK será igual a
y + z + q + w = 10 + 6 + 3 + 5 = 24
Portanto, a alternativa (02) é verdadeira.

(04) 36 pessoas gostavam de SAMBA ou ROCK, mas não


gostavam de JAZZ.
Sendo A = {gostam de SAMBA}, B = {gostam de ROCK} e C =
{gostam de JAZZ}, vemos pela figura anterior que o número de
pessoas que gostam de SAMBA ou ROCK mas não gostam de
JAZZ será igual a x + y + z = 20 + 10 + 6 = 36
Portanto, a alternativa (04) é verdadeira.

(08) 21 pessoas gostavam de pelo menos dois tipos de música.


A expressão “gostavam de pelo menos dois tipos de música”
significa neste caso: “gostam de duas ou três tipos de música”.
Olhando a figura acima, concluímos que este número é igual a
y + p + q + w = 10 + 8 + 3 + 5 = 26
Portanto, a alternativa (08) é falsa.

(16) 12 pessoas não gostavam de nenhum dos três tipos de


música citadas.
Pela figura acima, vemos imediatamente que o número de pessoas
que não gostavam de nenhum dos 3 tipos de música é igual a s.
Já vimos acima que s = 12.
Portanto, a alternativa (16) é verdadeira.

Resposta: 02 + 04 + 16 = 22

No reino da bicharada e
outros problemas
1 No livro “Elementos de Álgebra, publicado em 1770, o matemático
suíço Leonhard Euler, 1707 - 1783 – propôs o seguinte problema:
Uma lebre está 50 pulos à frente de um cachorro, o qual dá 3 pulos
no tempo que ela leva para dar 4. Sabendo que 2 pulos do cachorro
valem 3 da lebre, quantos pulos ele deve dar para pegá-la?

Solução:

Acompanhem o seguinte raciocínio simples:


Se 2 pulos do cachorro equivalem a 3 pulos da lebre então é óbvio
que 1 pulo do cachorro será equivalente a 1,5 pulos da lebre.
Logo, 3 pulos do cachorro será equivalente a 3 x 1,5 = 4,5 pulos da
lebre.

Então, a cada seqüência de 3 pulos do cachorro, ele se aproxima


4,5 – 4 = 0,5 pulo (da lebre).
Como a distancia inicial entre eles é igual a 50 pulos (da lebre), o
cachorro para vencer a distância deverá dar 50 / 0,5 = 100
seqüências
de 3 pulos(do cachorro), ou seja 100 x 3 = 300 pulos.
Resposta: o cachorro deverá dar 300 pulos.

2 Um rato sai correndo e quando deu 200 pulos o gato parte ao seu
encalço. Enquanto o gato dá 3 pulos, o rato dá 11 pulos, porém 2
pulos do gato valem 9 do rato. Quantos pulos o gato deverá dar
para alcançar o rato?

Solução:

Acompanhem o seguinte raciocínio simples, similar ao anterior:


2 pulos do gato = 9 pulos do rato. Daí, é claro que:
1 pulo do gato = 4,5 pulos do rato e,
3 pulos do gato = 3 x 4,5 = 13,5 pulos do rato
Em cada seqüência de 3 pulos, o gato se aproxima 13,5 – 11 = 2,5
pulos (do rato).
Como a distancia entre eles é de 200 pulos (do rato), o gato, para
vencer a distancia, deverá dar 200/2,5 = 80 seqüências de 3 pulos,
ou seja: 80 x 3 = 240 pulos.
Resposta: o gato deverá dar 240 pulos.
Agora resolva este:
Um gato persegue um rato; enquanto o rato dá 5 pulos, o gato dá 3,
porém 1 pulo do gato equivale a 2 pulos do rato. O rato leva uma
dianteira equivalente a 50 pulos do gato. Quantos pulos o gato
deverá dar para alcançar o rato?
Resposta: o gato deverá dar 300 pulos.

3 Se 0 < k < 1, então o valor da expressão.

é igual a:
a) 0
b) 1
c) 2
d) 2/k
e) k
Solução:
Observe que pela condição dada para k, ele é um número real
entre 0 e 1, e por conseqüência k + 1 > 0 e k – 1 < 0.
Observe também que k2 + 2k + 1 = (k + 1)2 e k2 – 2k + 1 = (k –
1)2

Substituindo na expressão dada fica:

<>
Se necessário, verifique a definição de módulo de um número real.
Ora, da definição de módulo infere-se que, como k + 1 é positivo |k
+ 1| = k + 1 e como k – 1 é negativo, |k – 1| = – (k – 1) = – k + 1.
Substituindo novamente, fica:
Notas:

1 – observe que k está situado entre 0 e 1 e, portanto, é diferente


de zero, o que torna possível dizer que 2k / k = 2. Portanto, a
alternativa correta é a de letra C.
2 – se k fosse igual a zero, teríamos 2k / k = 0 / 0 e não poderíamos
neste caso afirmar que o resultado seria 2, pois 0/0 poderia assumir
qualquer valor. O símbolo 0/0 é conhecido como uma
indeterminação matemática.
3 – esta questão simples foi enviada por um estudante que achou a
solução 2/k por não ter observado corretamente as propriedades da
função módulo.

Agora resolva este:


Se k < –1, então o valor da expressão.

é igual a:
a) 0 b) 1 *c) -2/k d) 2/k e) k

4 UESB 2007.1 – Um cliente pagou 40 % de uma dívida de x reais.


Sabendo-se que R$300,00 correspondem a 20 % do restante a ser
pago, é correto afirmar que o valor de x é igual a
(01) 3750
(02) 3000
(03) 2750
(04) 2500
(05) 2050
Nota: UESB – Universidade Estadual do Sudoeste Baiano

Solução:

Sendo x o valor total da dívida, como foi pago 40% de x, restou


(100% - 40% ) de x ou seja: 60% de x = 60% . x = 0,60 x.
Pelo enunciado, 20% do restante é igual a 300, ou seja: 20 % de
60% de x = 0,20 . 0,60. x = 0,12x = 300. Logo,
x = 300 / 0,12 = 30000 / 12 = 15000 / 6 = 5000 / 2 = 2500.

5 UESB 2007.1 –Em uma campanha de Natal foram distribuidos


entre algumas famílias de uma comunidade, 144 brinquedos, 192
pares de sapatos e 216 camisas. A distribuição foi feita de modo
que o maior número possível de famílias fossem contempladas e
todas recebessem o mesmo número de brinquedos, o mesmo
número de pares de sapatos e o mesmo número de camisas.
Considerando-se que cada família recebeu x brinquedos e y pares
de sapatos, pode-se afirmar que o valor de x + y é igual a:
01) 24
02) 14
03) 12
04) 8
05) 6

Solução:
Trata-se de um problema típico de aplicação do MDC - Máximo
Divisor Comum. Ora, MDC (144, 192, 216) = 24. Então, serão
144/24 = 6 brinquedos, 192/24 = 8 pares de sapatos e 216/24 = 9
camisas para cada família. Do enunciado da questão infere-se
então x = 6 e y = 8 e, portanto, x + y será igual a 14, o que nos leva
à alternativa (02). Se você quiser revisar MDC, visite o arquivo
Máximo Divisor Comum. Para retornar a esta página, clique em
VOLTAR no seu navegador.

6 – Um rato está 30 metros à frente de um gato que o persegue.


Enquanto o rato corre 8 metros, o gato corre 11 metros. Qual a
distância que o gato terá de percorrer para alcançá-lo?

Solução:
Seja g à distância percorrida pelo gato num tempo t e seja r a
distância percorrida pelo rato no mesmo intervalo de tempo.
Poderemos escrever: g = 11t e r = 30 + 8t. O gato encontrará o rato
quando g = r, ou seja: 11 t = 30 + 8 t. Daí vem 11 t – 8 t = 30, ou
seja, 3 t = 30, de onde tiramos t = 10. Substituindo em g = 11 t vem
finalmente: g = 11.10 = 110 metros.
Resposta: o gato alcançará o rato a 110 metros do ponto de partida.

7 – Um gato está 72 metros à frente de um cachorro que o


persegue. Enquanto o gato corre 7 metros, o cachorro corre 9
metros. Quantos metros o cachorro deverá percorrer até ficar a 12
metros do gato?

Solução:
Sendo g à distância percorrida pelo gato num tempo t, poderemos
escrever: g = 72 + 7t. Analogamente, sendo c a distância percorrida
pelo cachorro no mesmo tempo t, vem: c = 9t. Ora, o cachorro
estará a 12 metros do gato quando g - c = 12, ou seja: (72 + 7t) - 9t
= 12. Daí vem 72 - 2 t = 12 ou 2 t = 60; portanto t = 60/2 = 30.
Substituindo este valor em c = 9 t vem finalmente: c = 9.30 = 270.
Resposta: o cachorro estará a 12 metros do gato após ter
percorrido 270 metros. Faça as contas para saber que o cachorro
alcançará o pobre gato após ter percorrido 324 metros. Aí será
briga na certa!
SURPRESA ESPECIAL PARA VOCÊ!

Gostaria de participar do sorteio de 1 Kindle Paperwhite


+ 1 ano grátis de Kindle Unlimited, podendo ler quantos
ebooks, de vários autores da Amazon, quiser?
É muito fácil! A gente te ajuda nessa! Escreva uma resenha
para gente na página deste ebook na Amazon e pronto!
Automaticamente você estará concorrendo, pegaremos seus
dados de contato e caso você seja sorteado informaremos
via email. O próximo sorteio será dia 21/04/2019, e os
demais toda quinta-feira até junho de 2019. Corra e nos
envie sua resenha, não perca essa grande oportunidade!
Confira alguns de nossos ebooks abaixo com valores
exclusivos, a partir de R$ 1,99:
Obrigado pela leitura!

Você também pode gostar